56
Matematický korespondenční seminář Korespondenční seminář KAM MFF UK Malostranské náměstí 25 118 00 Praha 1 Milý příteli ! Pokud se Ti o Vánocích neukázalo zlaté prasátko, nezoufej, naše Pra- Sátko Ti to určitě mnohokrát vynahradí. Tentokrát Ti jako opož- děný dárek přináší hned tři série plné úloh – Cestování a bloudění, (Od)mocniny a druhou seriálovou. K ní patří text, nazvaný tentokrát Pevné základy, který najdeš dále v komentářích. Jestliže jsi měl(a) podzim příliš hektický a pořád ses seriálem Do nekonečna a ještě dál neprokousal(a), rozhodně to nevzdávej. Začátek tohoto dílu totiž na předchozí část nenavazuje, takže se do něj může pustit kdokoli. Po- kud ho ale chceš pochopit celý a přečíst si i pohádku o nespočetném gamblerovi, znalosti z prvního dílu budeš potřebovat. Podzimní část už skončila a co nevidět budou známé její konečné výsledky. Nejlepší řešitelé budou jako vždy pozvaní na jarní soustře- dění, které se tentokrát uskuteční od 12. do 20. března na zatím uta- jeném místě. Ale ať už pojedeš, nebo ne, určitě se snaž i v jarní části – podzimní soustředění bývá taky super, na ukázku z toho posledního se můžeš podívat níže. Za ostatní organizátory zdraví a krásný nový rok se spoustou vyřeše- ných úloh přeje Bára Kociánová Co je dále v komentářích? Vzorová řešení 2. a 3. podzimní série Vzorové řešení 1. seriálové série Seriál – Do nekonečna a ještě dál II. Výsledkové listiny Příloha: Zadání 1. a 2. jarní série a 2. seriálové série Vzpomínka na podzimní soustředění Soustředění tentokrát proběhlo ve Skleném na Vysočině. Účastníci tam přijeli 24. října jakožto branci a branky a podstoupili začátek vojenského výcviku. Nicméně kvůli (ne)schopnosti své a generálově se propadli do pohádkové země, kde ušli nepočítaně mil, spali sto let a zničili jednoho děda Vševěda. Nakonec se jim ale podařilo dostat zpět do našeho světa a jako skoro ostřílení vojáci a řešitelé zkušenější o mnoho matematických přednášek odjeli 1. listopadu domů.

Matematický korespondenční seminář Milýpříteli! · těžiště trojúhelníka KNP. Poměr, v kterém těžiště dělí těžnici, je přitom dobře znám – kýžený výsledek

  • Upload
    others

  • View
    7

  • Download
    0

Embed Size (px)

Citation preview

Page 1: Matematický korespondenční seminář Milýpříteli! · těžiště trojúhelníka KNP. Poměr, v kterém těžiště dělí těžnici, je přitom dobře znám – kýžený výsledek

Matematickýkorespondenční

seminář

Korespondenční seminář

KAM MFF UK

Malostranské náměstí 25

118 00 Praha 1

Milý příteli !

Pokud se Ti o Vánocích neukázalo zlaté prasátko, nezoufej, naše Pra-Sátko Ti to určitě mnohokrát vynahradí. Tentokrát Ti jako opož-děný dárek přináší hned tři série plné úloh – Cestování a bloudění,(Od)mocniny a druhou seriálovou. K ní patří text, nazvaný tentokrátPevné základy, který najdeš dále v komentářích. Jestliže jsi měl(a)podzim příliš hektický a pořád ses seriálem Do nekonečna a ještě dálneprokousal(a), rozhodně to nevzdávej. Začátek tohoto dílu totiž napředchozí část nenavazuje, takže se do něj může pustit kdokoli. Po-kud ho ale chceš pochopit celý a přečíst si i pohádku o nespočetnémgamblerovi, znalosti z prvního dílu budeš potřebovat.Podzimní část už skončila a co nevidět budou známé její konečné

výsledky. Nejlepší řešitelé budou jako vždy pozvaní na jarní soustře-dění, které se tentokrát uskuteční od 12. do 20. března na zatím uta-jeném místě. Ale ať už pojedeš, nebo ne, určitě se snaž i v jarní části– podzimní soustředění bývá taky super, na ukázku z toho posledníhose můžeš podívat níže.

Za ostatní organizátory zdraví a krásný nový rok se spoustou vyřeše-ných úloh přeje

Bára Kociánová

Co je dále v komentářích?

• Vzorová řešení 2. a 3. podzimní série• Vzorové řešení 1. seriálové série• Seriál – Do nekonečna a ještě dál II.• Výsledkové listiny

• Příloha: Zadání 1. a 2. jarní série a 2. seriálové série

Vzpomínka na podzimní soustředění

Soustředění tentokrát proběhlo ve Skleném na Vysočině. Účastnícitam přijeli 24. října jakožto branci a branky a podstoupili začátekvojenského výcviku. Nicméně kvůli (ne)schopnosti své a generálověse propadli do pohádkové země, kde ušli nepočítaně mil, spali sto leta zničili jednoho děda Vševěda. Nakonec se jim ale podařilo dostatzpět do našeho světa a jako skoro ostřílení vojáci a řešitelé zkušenějšío mnoho matematických přednášek odjeli 1. listopadu domů.

Page 2: Matematický korespondenční seminář Milýpříteli! · těžiště trojúhelníka KNP. Poměr, v kterém těžiště dělí těžnici, je přitom dobře znám – kýžený výsledek

Matematický korespondenční seminář 35. ročník (2015/2016), 2. komentáře

Jarní výlet

V sobotu 9. dubna se vypravíme na tradiční jarní výlet za krásami Středočeského kraje. Pokud užPraSátka znáš, určitě se tam potkáš s nějakými svými kamarády z řad účastníků a organizátorů.Pokud jsi naopak v našem semináři nováčkem, je to pro Tebe jedinečná příležitost se s námiseznámit. Bližší informace, jako například místo srazu, se dozvíš v příštích komentářích. Prozatímprozradíme, že se vydáme do takřka legendárních míst, kde vznikalo naše turistické značení, a kteráuž vlastně ani neexistují.

Náboj

Do kalendáře si také poznamenej datum 15. dubna, kdy se uskuteční mezinárodní matematickásoutěž Náboj. Uteče to, tak už raději mezi spolužáky začni hledat spolunadšence do týmu!

2

Page 3: Matematický korespondenční seminář Milýpříteli! · těžiště trojúhelníka KNP. Poměr, v kterém těžiště dělí těžnici, je přitom dobře znám – kýžený výsledek

Poměry2. podzimní série Vzorové řešení

Úloha 1. (193; 191; 2,97; 3,0)Jana, Dana a Hana organizují spolu s pěti hochy Korespondenční seminář z poměrů. Vyšlo najevo,že Jana i Dana měly v minulosti poměr se třemi organizátory a Hana dokonce se čtyřmi. Musí užnutně existovat organizátor, který měl poměr se všemi třemi organizátorkami?

(Alexander „Olinÿ Slávik)

Řešení:Organizátorky měly dohromady celkem deset poměrů, které můžeme rozdělit mezi pět organizátorůtak, aby každý měl právě dva. Organizátory si označíme čísly 1 až 5. Jana měla poměr s organizátory1, 2, 3, Dana s organizátory 4, 5, 1 a Hana s organizátory 2, 3, 4, 5. Takto měl každý organizátorpoměr právě se dvěma organizátorkami. Tudíž tvrzení, že vždy nutně existuje organizátor, kterýměl poměr se všemi třemi organizátorkami, neplatí.

Poznámky:S první úlohou si hravě poradili téměř všichni řešitelé. K vyvrácení zadaného tvrzení stačilo najítjedinou situaci, kdy žádný organizátor neměl poměr se všemi organizátorkami. To se v podoběgrafů, tabulek nebo jen barvitých líčení podařilo všem, kteří správně pochopili zadání.

(Karolína Kuchyňová)

Úloha 2. (170; 166; 2,89; 3,0)Tenista počítá svoji úspěšnost tak, že vydělí počet vyhraných zápasů počtem všech odehranýchzápasů. Před začátkem turnaje byla jeho úspěšnost přesně 1/2. Během turnaje odehrál čtyři zápasy,ze kterých tři vyhrál a jeden prohrál. Po turnaji jeho úspěšnost přesáhla 0,503. Jaký nejvyšší početzápasů mohl před turnajem vyhrát? (Lucia Magurová)

Řešení:Počet zápasů, které tenista vyhrál před turnajem, si označíme písmenem n. Potom víme, že předturnajem byla jeho úspěšnost 0,5 = n

2n, takže celkový počet zápasů, které odehrál před turnajem,

je 2n. Po turnaji přibyly čtyři zápasy, z toho tři vyhrané; také však víme, že úspěšnost přesáhlahodnotu 0,503. To si zapíšeme nerovnicí, kterou následně vyřešíme:

n+ 3

2n+ 4> 0,503,

n+ 3 > 1,006n+ 2,012,

0,988 > 0,006n,

164,6̄ > n.

Poznamenejme, že ve druhém kroku jsme násobili výrazem 2n+4, kterým násobit můžeme, protožeto je jistě kladné číslo. Nejvyšší celočíselné n splňující danou nerovnost je 164, a protože provedenéúpravy byly ekvivalentní, je to také řešení naší úlohy.

3

Page 4: Matematický korespondenční seminář Milýpříteli! · těžiště trojúhelníka KNP. Poměr, v kterém těžiště dělí těžnici, je přitom dobře znám – kýžený výsledek

Matematický korespondenční seminář 35. ročník (2015/2016), 2. komentáře

Poznámky:

Úloha byla přímočará a skoro všechna řešení se ubírala stejnou cestou jako to vzorové. Těm několikařešitelům, kteří si sestavili chybnou nerovnici (zajímavé je, že každý jinou) a tu poté vyřešili, jsemstrhnul jeden bod. (Václav Rozhoň)

Úloha 3. (144; 129; 2,74; 3,0)

Buď ABCD rovnoběžník. V jakém poměru rozdělují přímky procházející vrcholem A a středy stranBC resp. CD úhlopříčku BD?

(Míša Hubatová)

Řešení:

Označme si středy stran BC, resp. CD, jako S1, resp. S2. Dále si označme průsečík úhlopříček ACa BD jako bod S a průsečíky BD s AS1, resp. AS2, jako X, resp. Y .

Úhlopříčky se v rovnoběžníku půlí, proto |AS| = |SC| a |BS| = |SD|. V trojúhelníku ABCje proto BS těžnicí na stranu AC. Zároveň je AS1 těžnicí na stranu CB, takže bod X je těžištětrojúhelníku ABC. Z toho plyne |BX| = 2|XS|, neboli |BX| = 2

3|BS| = 1

3|BD|.

Analogicky z trojúhelníku CDA plyne |DY | = 2|Y S|, což implikuje |DY | = 23|DS| = 1

3|BD|.

Body X a Y jsou tedy ve třetinách úhlopříčky BD, takže |DY | : |Y X| : |XB| = 1 : 1 : 1.

A B

CD

SS1

X

Y

S2

Jiné řešení:

Definujme si body X, Y , S1 a S2 stejně jako v minulém řešení. Posunutím rovnoběžníku ABCDvytvoříme síť shodných rovnoběžníků jako na obrázku. Jako I, J ,K a L si označme body zvýrazněnéna obrázku. Bod B se ve stejnolehlosti se středem A a koeficientem 3 zobrazí na bod J a bod Dna K. Naším cílem bude ukázat, že se také zobrazí bod X na L (a analogicky také bod Y na I).Jednak je úsečka JL rovnoběžná s BD, a tedy i s BX. Dále z toho, že ABLC je rovnoběžník,plyne, že jeho úhlopříčka AL půlí druhou úhlopříčku BC. Proto S1 leží na AL, a tedy i X leží naAL. A protože obraz X je průnikem AX a rovnoběžky s BX vedené skrze obraz B, je L skutečněobrazem X.

Protože KI, IL a LJ jsou úhlopříčky ve shodných rovnoběžnících, je |KI| = |IL| = |LJ |. Poopětovném použití stejnolehlosti získáváme |DY | = |Y X| = |XB| = 1

3|BD|. Hledaný poměr tedy

je 1 : 1 : 1.

4

Page 5: Matematický korespondenční seminář Milýpříteli! · těžiště trojúhelníka KNP. Poměr, v kterém těžiště dělí těžnici, je přitom dobře znám – kýžený výsledek

Korespondenční seminář, KAM MFF UK, Malostranské náměstí 25, 118 00 Praha 1

A B

CD

X

YS1

S2

J

L

I

K

Ještě jiné řešení:Označme si body stejně jako v prvním řešení. Z rovnoběžnosti přímek DC a AB plyne rovnost|∢ABY | = |∢S2DY |, protože tyto dva úhly jsou střídavé. Dále platí |∢DY S2| = |∢BY A|, protožese jedná o vrcholové úhly. Navíc platí |DS2| = 1

2|DC| = 1

2|BA|. Proto podle věty uu dostáváme

△AY B ∼ △S2Y D s koeficientem podobnosti 12 . Z toho plyne |DY | =12|Y B|. Analogicky dosta-

neme i |BX| = 12|XD|. Z toho plyne |DY | = |Y X| = |XB|, takže kýžený poměr je 1 : 1 : 1.

Poznámky:Úloha byla celkem jednoduchá, o čemž svědčí počet správných řešení. Někdy se vyskytovala řešenípro nějaké konkrétní rovnoběžníky – čtverec atd. Proto připomínám, že pokud není v zadání spe-cifikováno jinak, myslí se obecný rovnoběžník a je nutno dokazovat vše obecně. Rozhodně se takéza důkaz nepovažuje objekt narýsovaný v geogebře nebo v ruce! Pokud se dalo řešení jednodušezobecnit, dával jsem dva body, v opačném případě nebo v případech, kdy chyběly důkazy či jejichpodstatné části, jsem dával jeden.Dalším nešvarem, vyskytujícím se u poměrně velkého množství řešitelů, je nedbat při zapisování

podobnosti trojúhelníků na pořadí vrcholů. To je nutné, neboť nám pak toto pořadí udává, jakéstrany jsou v jakém poměru. Při nedodržení značení získávám poměr jiných stran, než chci nebopoužívám v důkazu. (Honza Kadlec)

Úloha 4. (78; 70; 4,21; 5,0)Jsou dány tři kružnicové oblouky se společnými koncovými body A a B. Z bodu B vedeme dvěpolopřímky tak, že obě leží ve stejné polorovině vzhledem k přímce AB. První polopřímka obloukyprotne postupně v bodech M1, M2, M3, druhá pak postupně v bodech N1, N2, N3. Dokažte, že

|M1M2||M2M3|

=|N1N2||N2N3|

.

(Martin Töpfer)

5

Page 6: Matematický korespondenční seminář Milýpříteli! · těžiště trojúhelníka KNP. Poměr, v kterém těžiště dělí těžnici, je přitom dobře znám – kýžený výsledek

Matematický korespondenční seminář 35. ročník (2015/2016), 2. komentáře

Řešení:

A B

M1

M2

M3

N1

N2

N3

Všimneme si, že úhly AM1B a AN1B jsou shodné, neboť jsou to obvodové úhly oblouku AB.Obdobně jsou shodné dvojice AM2B a AN2B i AM3B a AN3B. Trojúhelníky AM1M2 a AN1N2jsou podobné podle věty uu (už jsme ukázali |∢AM2B| = |∢AN2B| a úhly AM1M2 a AN1N2 jsoudoplňkové do 180◦ k úhlům AM1B, resp. AN1B, jejichž shodnost jsme si též již ukázali). Obdobněsi jsou podobné i trojúhelníky AM2M3 a AN2N3. Z první podobnosti vyplývá

|M1M2||N1N2|

=|AM2||AN2|

a z druhé

|M2M3||N2N3|

=|AM2||AN2|

.

Pravé strany jsou si rovny, takže se musí rovnat i levé, tedy

|M1M2||N1N2|

=|M2M3||N2N3|

,

což už lze jednoduše upravit na dokazovanou rovnost.

Poznámky:

Téměř všechna řešení, která jsem opravoval, byla správně a téměř všechna správná řešení vypadalajako vzorové. Nejčastější chybou bylo, že se při dokazování shodnosti úhlů ∢AM1B a ∢AN1Břešitelé místo shodnosti obvodových úhlů snažili používat Thaletovu větu. Ta ale lze použít pouzev případě, kdy by AB byl průměr daného oblouku. V řešení, kde jsem objevil alespoň náznaktoho, že autor ví, že je Thaletovu větu pro použití v tomto případě nutno nějak „zobecnitÿ, jsemale za tuto chybu body nestrhával. Dalším častým problémem bylo, že mnozí nijak nedokázali adokonce ani nezmínili, že koeficienty podobností mezi trojúhelníky jsou stejné (ve vzoráku dokázánoporovnáváním s poměrem |AM2| : |AN2|), za což jsem jeden bod strhával. (Viki Němeček)

6

Page 7: Matematický korespondenční seminář Milýpříteli! · těžiště trojúhelníka KNP. Poměr, v kterém těžiště dělí těžnici, je přitom dobře znám – kýžený výsledek

Korespondenční seminář, KAM MFF UK, Malostranské náměstí 25, 118 00 Praha 1

Úloha 5. (101; 91; 4,28; 5,0)Tečny ke kružnici k se středem S se jí dotýkají v bodech K, L a protínají se v bodě M . Bod N ležína k tak, že KN je její průměr. Označme P průsečík přímek LN a KM a Q průsečík PS a MN .Vypočtěte |MQ| / |QN |.

(Anh Dung „Tondaÿ Le)

Řešení:

M

K

L N

S

P

Q

Tečny z bodu M ke kružnici k mají stejnou délku, tudíž |MK| = |ML|. Dále |SK| = |SL|, protožeobě úsečky jsou poloměry kružnice k. Trojúhelníky SMK a SML jsou tedy shodné a spolu tvořídeltoid, jehož úhlopříčka SM tvoří osu druhé úhlopříčky KL.Nyní se podívejme na trojúhelník KLP , který má pravý úhel u vrcholu L, neboť KLN je úhel

nad průměrem KN v kružnici k, a proto je pravý. Střed kružnice opsané trojúhelníku KLP je středúhlopříčky KP , který také náleží osám odvěsen. Zmíněný střed kružnice opsané musí být tedy M ,protože leží na KP a ose KL. Z toho plyne, že M je střed KP . Vyšlo tak najevo, že bod Q jetěžiště trojúhelníka KNP . Poměr, v kterém těžiště dělí těžnici, je přitom dobře znám – kýženývýsledek je |MQ|/|QN | = 1/2.

Poznámky:Většina došlých řešení byla správná a skoro všechna spočívala v důkazu, že Q je těžiště trojúhelníkaPKN jako ve vzorovém řešení, nebo že MS je rovnoběžná s PN a hledaný poměr se pak dávypočítat z podobných trojúhelníků MQS a NQP . Dále chci upozornit, že se nesmí zapomenoutna slovní definici přikreslených bodů nebo úhlů, které se nevyskytly v zadání úlohy.

(Anh Dung „Tondaÿ Le)

Úloha 6. (55; 24; 1,91; 0,0)Pětiprvkovou množinu nenulových reálných čísel nazveme vykutálenou, pokud platí, že pro libovolnátři různá čísla x, y, z ležící v této množině je xy + yz + zx racionální číslo. Ukažte, že poměrlibovolných dvou čísel z jedné vykutálené množiny je racionální. (Rado Švarc)

Řešení:Budeme používat skutečnost, že součet, rozdíl, součin i podíl nenulových racionálních čísel jsoutaké racionální. To se ukáže lehce – racionální čísla x a y zapíšeme jako x = p/q a y = r/s, kde p,q, r a s jsou nenulová celá čísla. Potom dostáváme

x± y = p

q± r

s=ps± qrqs

∈ Q,

xy =p

q· rs=pr

qs∈ Q,

x

y=

p

q

rs

=ps

qr∈ Q,

což jsme chtěli.

7

Page 8: Matematický korespondenční seminář Milýpříteli! · těžiště trojúhelníka KNP. Poměr, v kterém těžiště dělí těžnici, je přitom dobře znám – kýžený výsledek

Matematický korespondenční seminář 35. ročník (2015/2016), 2. komentáře

Nechť a a b jsou libovolné dva různé prvky vykutálené množinyM . Ukážeme, že a/b je racionální.Zbylé prvky M budeme označovat jako c, d a e. Platí ab+ bc+ ca ∈ Q a ab+ bd+ da ∈ Q. Potomi rozdíl těchto dvou čísel je racionální, takže

(ab+ bc+ ca)− (ab+ bd+ da) = (a+ b)(c− d) ∈ Q.

Analogicky dostaneme i vztahy

(b+ e)(c− d) ∈ Q a (a+ e)(c− d) ∈ Q.

Pak ale musí být i

(a+ b)(c− d) + (a+ e)(c− d)− (b+ e)(c− d) = 2a(c− d) ∈ Q.

Analogicky dostaneme

2b(c− d) ∈ Q.

Nyní si uvědomíme, že a i b jsou ze zadání nenulová. Navíc c a d jsou různé prvky, takžec− d 6= 0. Proto 2a(c− d) 6= 0 a 2b(c− d) 6= 0. A protože poměr dvou nenulových racionálních číselje racionální, dostáváme, že

2a(c− d)2b(c− d) =

a

b∈ Q,

což je přesně to, co jsme chtěli dokázat.

Poznámky:

Řešení se sešlo vcelku mnoho, většina z nich ovšem byla špatně. Velká část řešitelů prohlásila, žepokud je součet tří čísel racionální, potom všechna tato čísla jsou taktéž racionální. To ale zjevněneplatí, například pro trojici

√2,√3 a −(

√2 +√3).

Většina úspěšných řešení používala stejný postup jako řešení vzorové – různým sčítáním, odčí-táním a dělením čísel, o nichž je známo, že jsou racionální, dostávat další racionální čísla. Zajímavýpostup, který vlastně objasňoval, proč řešení ostatních vážně funguje, zvolila Vendula Kuchyňová,která použila substituci x1 = ab, x2 = ac, . . . , x10 = de a prohlásila, že ze zadání plyne, žex1 + x2 + x5 = q1 pro nějaké racionální q1 a analogicky pro další součty. Tím dostala soustavudeseti lineárních rovnic o deseti neznámých x1, x2, . . . , x10 s racionálními parametry q1, q2, . . . ,q10. Následně prohlásila, že tuto soustavu umíme vyřešit a tím každé xi vyjádřit jako nějakou raci-onální kombinaci q1 až q10. V tu chvíli už víme, že součin každých dvou čísel z vykutálené množinyje racionální, a protože ac

bc= a

b, jsme hotovi. Kvůli nejasnostem v tom, proč má daná soustava

vždy jednoznačné řešení, se tak dopracovala ke kurióznímu ohodnocení 4 + i. (Rado Švarc)

Úloha 7. (67; 24; 2,34; 1,0)V trojúhelníku ABC má úhel při vrcholu A velikost 60◦. Uvnitř trojúhelníka se nachází bod K,pro který |∢AKB| = |∢BKC| = |∢CKA| = 120◦. Označíme-li střed strany BC jako M , dokažterovnost

|KA|+ |KB|+ |KC||AM | = 2.

(Martin Töpfer)

8

Page 9: Matematický korespondenční seminář Milýpříteli! · těžiště trojúhelníka KNP. Poměr, v kterém těžiště dělí těžnici, je přitom dobře znám – kýžený výsledek

Korespondenční seminář, KAM MFF UK, Malostranské náměstí 25, 118 00 Praha 1

Řešení:Označíme si x = |AK|, y = |BK|, z = |CK| a a = |BC|, b = |AC|, c = |AB| a |∢BAK| = ϕ.Z toho plyne |∢CAK| = 60◦ − ϕ. Dopočítáním do 180◦ v trojúhelníku ABK dostaneme

|∢ABK| = 180◦ − 120◦ − ϕ = 60◦ − ϕ,takže △ABK ∼ △CAK podle věty uu. Tedy x/y = z/x, tedy x2 = yz. Nyní použijeme kosinovouvětu, která tvrdí, že v libovolném trojúhelníku XYZ platí

|XZ|2 = |XY |2 + |Y Z|2 − 2 · |XY | · |Y Z| · cos |∢XY Z|.Z rovnosti cos 120◦ = − 1

2plyne, že použitím kosinové věty pro trojúhelníky AKB, AKC a

BKC získáme vztahyb2 = x2 + z2 + xz,

c2 = x2 + y2 + xy,

a2 = z2 + y2 + zy.

Nyní použijeme kosinovou větu na trojúhelníky ABM a ACM . Tím dostaneme

b2 = |AM |2 + a2

4− a · |AM | · cos |∢CMA|,

c2 = |AM |2 + a2

4− a · |AM | · cos |∢BMA|.

Jelikož |∢BMA| = 180◦−|∢CMA|, tak díky vztahu cosφ = − cos(180◦−φ) dostaneme po sečtenítěchto rovností b2 + c2 = 2|AM |2 + a2/2. Tedy 4|AM |2 = 2b2 + 2c2 − a2. Nyní můžeme dosadit.

4|AM |2 = 2b2 + 2c2 − a2

= 2(x2 + z2 + xz) + 2(x2 + y2 + xy)− (z2 + y2 + zy)= 4x2 + y2 + z2 + 2xy + 2xz − yz.

Ze vztahu x2 = yz získáme rovnost 3x2 = 3yz. Tu dosadíme a dostaneme

4|AM |2 = x2 + y2 + z2 + 2xy + 2yz + 2xz = (x+ y + z)2.

Obě strany můžeme odmocnit, protože jsou nezporné, a dostaneme

2|AM | = |AK|+ |BK|+ |CK|,což jsme chtěli dokázat.

Druhé řešení (podle Tomáše Konečného):Uvažujme nový trojúhelník A1BC1 vzniklý rotací trojúhelníku ABC kolem bodu B o 60◦ směremod bodu C k bodu A. Nechť K1 je obraz K v této rotaci. Protože otočení je shodné zobrazení, platí|A1K1| = |AK|. Navíc z |BK| = |BK1| a |∢KBK1| = 60◦ plyne, že △BKK1 je rovnostranný,takže speciálně platí |KK1| = |BK|. Potom platí

|AK|+ |BK|+ |CK| = |A1K1|+ |K1K|+ |KC|.Jelikož |∢CKA| = 120◦, po otočení AK o 60◦ přímky CK a K1A1 svírají úhel 180◦, tedy A, K1,K a C leží na jedné přímce. Z toho také plyne

|A1K1|+ |K1K|+ |KC| = |A1C|,takže vlastně chceme dokázat, že |A1C| = 2|AM |.Přímky AC a BA1 jsou rovnoběžné, jelikož |∢A1BA| = |∢BAC| = 60◦. Střed A1B nazvěme

X. Trojúhelník A1AB je rovnostranný a AX je jeho těžnice. Proto je zároveň jeho výškou, takže|∢AXB| = 90◦. Potom BXAC je lichoběžník s pravým úhlem u vrcholu X. Střed XA nazvěme Y .Potom YM je střední příčka lichoběžníka AXBC a YM je kolmá na XA. Ale jelikož |XY | = |AY |,je △YMX shodný s △YMA, tedy |AM | = |XM |.V trojúhelníku BA1C je však XM střední příčka, z čehož dostáváme 2|XM | = |A1C|. Proto je

i 2|AM | = |A1C|, což jsme chtěli dokázat.

9

Page 10: Matematický korespondenční seminář Milýpříteli! · těžiště trojúhelníka KNP. Poměr, v kterém těžiště dělí těžnici, je přitom dobře znám – kýžený výsledek

Matematický korespondenční seminář 35. ročník (2015/2016), 2. komentáře

Poznámky:Většina řešení obsahovala tvrzení, že když máme trojúhelník splňující zadání, potom už musí býtrovnostranný. To samozřejmě není pravda, bod K má každý trojúhelník. Říká se mu Fermatův,nebo též Torriceliho bod a platí pro něj spousta zajímavých věcí. Správná řešení většinou používalavzorec pro výpočet těžnice kombinovaný se vztahem |AK|2 = |BK| · |CK|.Dvě řešení zase použila zobrazení, ze kterého už bylo řešení vidět. Tyto řešení jsem odměnil +i,

jelikož byly výrazně elegantnější než algerbaické počítání. (Jakub Svoboda)

Úloha 8. (58; 15; 1,07; 0,0)Pro přirozené číslo n označme jeho ciferný součet symbolem S(n). Najděte největší možnou hodnotupoměru S(n) /S(16n). (Martin Töpfer)

Řešení:Označme cifry přirozených čísel a a b tak, že a =

∑ki=0 10

iai a obdobně b =∑lj=0 10

jbj . Nyníukážeme několik základních tvrzení, která platí o ciferném součtu.

(1) S(10na) = S(a): Vynásobením a mocninou desítky jen připíšeme n nul na konec čísla,ciferný součet se tedy nezmění.

(2) S(a) ≤ a.(3) S(a + b) ≤ S(a) + S(b): Podívejme se, jak se sčítají čísla a a b pod sebou. Na každémřádu je buď součet příslušných cifer z a a b, nebo dojde k přenosu jedničky. Pak se číslona tomto řádu sníží o 10 a číslo řádu o jedna vyššího se zvýší o jedna, ciferný součet setedy celkově sníží o 9. Proto platí dokazovaná nerovnost.

(4) S(a ·b) ≤ S(a) ·S(b): Násobení a a b rozepíšeme jako násobení jednotlivých cifer a následněvýraz odhadneme pomocí předchozích tvrzení:

S(a · b) = S

k∑

i=0

l∑

j=0

aibj10i+j

≤k

i=0

l∑

j=0

S(

aibj10i+j

)

=

=k

i=0

l∑

j=0

S (aibj) ≤k

i=0

l∑

j=0

aibj = S(a) · S(b).

S použitím těchto pozorování již snadno odhadneme hodnotu zadaného poměru

S(n)

S(16n)=S(10000n)

S(16n)≤ S(625) · S(16n)

S(16n)= 13.

Poměr může nabývat hodnoty nejvýše 13. Této hodnoty dosáhneme např. pro n = 625.

Poznámky:Sešlo se nebývalé množství řešení osmičky, ale bohužel většina řešení sice našla maximální hodnotu,ale chyběla hlavní část úlohy – dokázat, že jde skutečně o maximum. Nejčastější chybný argumentbyl, že pro maximalizaci hodnoty zlomku stačí minimalizovat hodnotu jmenovatele. To není pravdauž jen proto, že pro libovolně velký jmenovatel dokážeme nalézt n, aby hodnota zlomku byla 13(n-krát za sebe napíšeme 625). Dalším jednoduchým příkladem, proč taková úvaha nebude správná,je maximalizace hodnoty nějakého jiného zlomku – například n2/n. Podle předchozího argumentuby takový zlomek nabýval hodnoty nejvýše 1 (pro n = 1). V některých řešeních byl i pokus orozebrání jiných hodnot n než 625, ale většinou i tato řešení rozebrala jen některé hodnoty n, okterých řekla (bez zdůvodnění), že budou nejlepší.Obecně u podobných úloh je úvaha založená na myšlence, že nějaký konkrétní postup (zde

S(n) = 1) bude nejlepší, většinou nevede k cíli. Často je totiž popsaný postup založený na nějakélokální optimalizaci nebo na pouhé intuici řešitele. Ani jedna z těchto věcí ale nezaručuje optimálnívýsledek a u opravovatele neprojde.

10

Page 11: Matematický korespondenční seminář Milýpříteli! · těžiště trojúhelníka KNP. Poměr, v kterém těžiště dělí těžnici, je přitom dobře znám – kýžený výsledek

Korespondenční seminář, KAM MFF UK, Malostranské náměstí 25, 118 00 Praha 1

Úlohu jsem se rozhodl bodovat velmi přísně a dost pravděpodobně jde o úlohu s nejnižším bo-dovým průměrem za poslední dobu. Za pouhé najití n, pro které má zlomek hodnotu 13, jsemneuděloval žádný bod. Stejně tak si body nevysloužily ani řešení založená na nějaké variaci ar-gumentu zmíněného v poznámkách, protože podobný postup by nebylo možné nějak jednodušerozšířit tak, aby šlo o řešení úlohy. U lehčí úlohy bych zřejmě i za částečné výsledky nějaké bodyuděloval, ale u osmé (tj. teoreticky nejtěžší) úlohy v sérii mi přijde, že by mělo být vyžadovánoúplné řešení. (Martin Töpfer)

11

Page 12: Matematický korespondenční seminář Milýpříteli! · těžiště trojúhelníka KNP. Poměr, v kterém těžiště dělí těžnici, je přitom dobře znám – kýžený výsledek

Úlohy na šachovnici3. podzimní série Vzorové řešení

Úloha 1. (141; 135; 2,90; 3,0)

Rozmístěte na šachovnici 6 × 6 čtyři tchýně 1 tak, aby se navzájem neohrožovaly a právě jednovolné pole zůstalo neohrožené. (Martin Töpfer)

Řešení:

V této úloze bylo úkolem najít vyhovující rozestavení tchyní. Možností, jak nějaké takové najít,existuje mnoho. My si ukážeme, jaké úvahy nám v hledání mohou pomoci.

Začneme pozorováním, že žádné dvě tchyně se nemohou vyskytovat ve stejném řádku ani sloupci,protože by se jinak ohrožovaly tahem věže. Právě čtyři řádky a čtyři sloupce tedy budou obsazenytchyní. Dva neobsazené řádky a sloupce nám pak určují čtyři políčka, která tchyně nebudou ohrožo-vat tahem věže. Víme, že jedno z těchto polí musí zůstat neohrožené a zbylá tři musí být tchyněmiohrožena tahem koně.

Nyní se můžeme zaměřit na ta rozestavení, kde jsou neobsazené právě dva prostřední sloupcea dva prostřední řádky. Po chvíli zkoušení pak dojdeme k jednomu z rozestavení na prvních dvouobrázcích (či k rozestavení, které z těchto dvou vznikne tak, že šachovnici otočíme, či překlopímepodle jedné z jejích os symetrie).

✻ 0Z0ZNZ✺ Z0Z0ZN✹ 0Z0Z0Z✸ Z0Z0Z0✷ 0M0Z0Z✶ M0Z0Z0

❛ ❜ ❝ ❞ ❡ ❢

1Tchýně je figura pohybující se po šachovnici pomocí tahů koně i věže.

12

Page 13: Matematický korespondenční seminář Milýpříteli! · těžiště trojúhelníka KNP. Poměr, v kterém těžiště dělí těžnici, je přitom dobře znám – kýžený výsledek

Korespondenční seminář, KAM MFF UK, Malostranské náměstí 25, 118 00 Praha 1

✻ NZ0Z0Z✺ Z0Z0ZN✹ 0Z0Z0Z✸ Z0Z0Z0✷ 0M0Z0Z✶ Z0Z0M0

❛ ❜ ❝ ❞ ❡ ❢

✻ NZ0Z0Z✺ ZNZ0Z0✹ 0ZNZ0Z✸ Z0Z0Z0✷ 0Z0Z0M✶ Z0Z0Z0

❛ ❜ ❝ ❞ ❡ ❢

Jinou možností je začít s rozestavením čtyř tchyní na hlavní diagonálu šachovnice. Toto ještěnení správné řešení, neboť zbývají dvě neohrožená políčka. Přesunutím poslední tchyně však jižzískáme validní pozici na třetím obrázku.

Poznámky:

Sešlo se mnoho různých rozestavení a drtivá většina z nich byla správně. Jejich tvůrci si tak zasloužiliplný počet bodů. Jak si také někteří řešitelé povšimli, správných řešení je hodně2. To je mimo jinézpůsobeno tím, že otočením či překlopením šachovnice ze správného řešení vyrobíme zase správnéřešení.

Na závěr opravme jednu gramatickou chybu, ke které nedopatřením v zadání došlo a kterouněkteří z Vás postřehli. Slovo tchyně je odvozeno od podstatného jména tchán pomocí přípony-yně, a není tedy důvod psát jej s dlouhým ý, čehož jsme se v zadání dopustili. Omlouváme se adoufáme, že Vás tato nepřesnost při řešení natolik závažného problému, jakým bezpečné rozmístěnítchyní jistě je, příliš nerušila. (Václav Rozhoň)

Úloha 2. (98; 76; 2,07; 2,0)

V rozích šachovnice 3×3 stojí dokola postupně Šemík, Rosinanta, Stínovlas a Trojský kůň. Všichnise mohou pohybovat jako šachoví koně a nesmí stát dva na stejném poli. Je možné, aby se za těchtopodmínek Šemík s Rosinantou prohodili a ostatní se vrátili na svá původní místa?

(Anh Dung „Tondaÿ Le)

2přesněji 340 :)

13

Page 14: Matematický korespondenční seminář Milýpříteli! · těžiště trojúhelníka KNP. Poměr, v kterém těžiště dělí těžnici, je přitom dobře znám – kýžený výsledek

Matematický korespondenční seminář 35. ročník (2015/2016), 2. komentáře

Řešení:

Troj Stín

Šem Ros

Troj Stín

Ros Šem

Uvažme tahy, kterými se koně mohou pohybovat mezi poli. Sestrojíme graf, jehož vrcholy odpovídajípolím šachovnice a hrana spojuje dvě políčka taková, že se z jednoho dá jedním tahem skočitna druhé. Výsledný graf je cyklus a jeden izolovaný vrchol (viz obrázek), ve kterém tahy koníodpovídají pohybu na sousední políčko.

Trojský k��

St���vlas

R���nanta

Šemík

ŠemíkTrojský kůň

S��vlas

�� nanta

Ani v tomto grafu nemohou dva koně stát ve stejném vrcholu. Podíváme-li se, kde stojí Šemík,a půjdeme-li po cyklu po směru hodinových ručiček, nalezneme Rosinantu, Stínovlase a nakonecTrojského koně. Uvedené tahy toto pořadí zachovávají. Rozestavení, do kterého chceme koně roz-místit, má ovšem jiné pořadí, a proto není možné prohodit Šemíka s Rosinantou tak, aby se ostatnívrátili na původní místa.

Poznámky:Všimněte si, že můžeme vypustit předpoklad o vracení na původní místa a stále to nepůjde. MeziŠemíkem a Rosinantou je v cyklu pouze jedno políčko, kde nemohou stát oba Stínovlas a Trojskýkůň. Nebo můžeme ze šachovnice odstranit Trojského koně, a přesto hledaná posloupnost tahů ne-bude existovat. Rozmyslete si, že to dokazuje stejný argument s pořadím koňů v cyklu. Kdybychomuvažovali pouze tři koně a nevyžadovali návrat na původní políčko, existovala by posloupnost tahů,při níž by si Šemík s Rosinantou vyměnili místa.Zhruba třetina řešení se podobala tomu autorskému. Většina těch ostatních se pokoušela o rozbor

případů. To není úplně marná cesta, protože na takto malé šachovnici existuje poměrně málorůzných situací a většina z nich je navíc v jistém smyslu symetrická. Postup je to ale velmi zdlouhavýa velmi náchylný na chyby. Mnoho řešitelů si snažilo ušetřit práci tím, že vybrali vždy ten nejlepšínebo nejrozumnější tah, ale bez argumentu s cyklem vůbec není jasné, který tah to je. Navíc nenívůbec jasné, proč nemá smysl, aby kůň skočil zpět, odkud přišel (dokud se nezmíní rozestavení nakružnici). A jak vlastně může být nějaký tah lepší než jiný, když ani jeden z nich nevede ke zdárnémucíli? Při rozboru případů je nezbytné pozorně prozkoumat všechny možnosti a nezavrhnout nějakoujenom proto, že „nevypadá slibně.ÿ (Filip Hlásek)

14

Page 15: Matematický korespondenční seminář Milýpříteli! · těžiště trojúhelníka KNP. Poměr, v kterém těžiště dělí těžnici, je přitom dobře znám – kýžený výsledek

Korespondenční seminář, KAM MFF UK, Malostranské náměstí 25, 118 00 Praha 1

Úloha 3. (132; 105; 2,39; 3,0)Obarvěte sedm polí šachovnice 4× 4 tak, aby na ní po odebrání libovolných dvou sloupců a dvouřádků zůstalo alespoň jedno obarvené pole. (Rado Švarc)

Řešení:Nejprve si uvědomíme, že po vyškrtnutí libovolných dvou řádků a sloupců nám z šachovnicezbude čtveřice polí, která původně tvořila rohy obdélníku3 se stranami rovnoběžnými s okrajemšachovnice.Pro začátek obarvěme horní levý roh šachovnice a přeškrtněme všechna pole ležící na úhlopříčce,

v řádku a v sloupci, které obsahují tento roh (viz obrázek).

Všimneme si, že žádná čtveřice přeškrtnutých polí netvoří rohy obdélníku. Tudíž všechny obdél-níky, které nemají v rohu již obarvené pole, musejí mít v alespoň jednom z rohů některé nepřeškrt-nuté pole. Těch už je ale jen 6, tudíž je můžeme obarvit všechna a získáme tak jedno z možnýchřešení:

Poznámky:Nejprve bych rád zmínil, že až na proházení řádků a sloupců měla úloha jediné řešení, celkem jichtedy bylo 96. Vzhledem k malým rozměrům zadané šachovnice bylo častým řešením této úlohyzkoušení několika různých obarvení, dokud nebyla splněna zadaná podmínka. Tento postup seovšem poměrně velkému počtu řešitelů vymstil, jelikož naprostá většina chybných řešení opomnělajeden či dva nepokryté obdélníky. Špatné řešení bylo obvykle podobné jednomu z následujícíchdvou obarvení:

(Tomáš Novotný)

3Čtverec je speciálním případem obdélníku.

15

Page 16: Matematický korespondenční seminář Milýpříteli! · těžiště trojúhelníka KNP. Poměr, v kterém těžiště dělí těžnici, je přitom dobře znám – kýžený výsledek

Matematický korespondenční seminář 35. ročník (2015/2016), 2. komentáře

Úloha 4. (89; 80; 3,35; 3,0)

Najděte všechna přirozená n, pro která lze rozdělit šachovnici n× n na lichý počet čtverců 2× 2 aněkolik4 tetromin tvaru T. (Rado Švarc)

Řešení:

Nejprve si dokážeme, že n musí být sudé: Čtverec 2×2 i T-tetromino jsou složené ze čtyř čtverečků.Proto 4 | n2, a tedy 2 | n.Dále oddělíme dva případy, n = 4k + 2 a n = 4k, kde k je nějaké celé nezáporné číslo.

Pro n = 4k+2 je možné pokrýt šachovnici lichým počtem čtverců 2×2 bez použití T-tetromina.Zjevně jich můžeme n/2 = 2k+1 položit vedle sebe na spodní okraj mřížky. Pak totéž uděláme vezbylých 2k dvojřádcích a dohromady budeme mít (2k + 1)2 = 4k2 + 4k + 1 čtverců 2 × 2, což jelichý počet.

Pro n = 4k obarvíme šachovnici černobíle klasickým způsobem. Protože je n sudé a barvy sepravidelně střídají, je na šachovnici stejně černých i bílých čtverečků. Každý čtverec 2 × 2 zřejmězabírá dvě bílá a dvě černá políčka. Naopak T-tetromino pokryje vždy buď jeden bílý a tři černé(nazveme ho černé), nebo jeden černý a tři bílé čtverečky (nazveme ho bílé). Abychom našimi útvarymohli pokrýt celý čtverec n × n, určitě potřebujeme zakrýt stejný počet bílých i černých políček.Proto ke každému černému T-tetrominu musí být na šachovnici jedno bílé. Neboli je potřeba použítsudý počet T-tetromin, označme tedy jejich počet jako 2l pro l nezáporné celé číslo.

Platí n = 4k, neboli n2 = 16k2. A odečteme-li od celkového počtu polí počet polí zabranýchT-tetrominy, dostaneme 16k2 − 4 · (2l) = 8(2k2 − l). Každý čtverec 2 × 2 zabírá čtyři políčka, dozbytku šachovnice se jich tedy vejde 8(2k2 − l)/4 = 2(2k2 − l), což je sudé číslo. Pro n dělitelnéčtyřmi proto nelze šachovnici pokrýt podle zadání.

Poznámky:

Úloha nedopadla moc dobře, ačkoli ke správné odpovědi dospěli snad všichni, kteří si správněpřečetli zadání. Více než polovina řešitelů ale potom nedokázala, že pro n dělitelné čtyřmi sešachovnice pokrýt nedá. Většinou vágně tvrdili, že „z T-tetromin se nedá sestavit jiný rozumnýtvar než čtverce 4 × 4ÿ, což zaprvé není pravda (lze jimi vyplnit například dvoučtverečkový okrajšachovnice pro n = 4k + 2) a zadruhé (což je mnohem důležitější) to opravdu nemůžeme tvrditjen proto, že jsme nenašli žádné jiné jejich uspořádání. Pro čtyři nebo pět T-tetromin snad ještěmůžeme vyzkoušet všechny možnosti, ale pro tisícové počty by to šlo už opravdu těžko.

Stejně tak je potřeba ukázat, že pro n = 4k + 2 řešení opravdu existuje. Snadno si lze totižpředstavit útvar, který na šachovnici zabere n2/4 bílých a n2/4 černých čtverečků, a přesto sevedle něj už nevejde druhý, který by pokryl zbytek šachovnice.

Na závěr bych ještě poznamenala, jaké problémy působilo slovo tetromino. Řešitelé ho tak půlnapůl používali ve středním a mužském rodě, někteří z něj udělali třeba tetramín nebo dokoncetriomino a vůbec jim nevadilo, že tak se běžně nazývá útvar o čtvereček menší.

(Bára Kociánová)

Úloha 5. (76; 63; 4,11; 5,0)

Kuba a Bára spolu hrají hru. Na začátku mají šachovnici 2015 × 2015, kde jsou všechna políčkabílá. Kuba v každém svém tahu přebarví nějaký bílý čtverec 2 × 2 na černo, Bára vždy přebarvínějaká tři bílá políčka tvořící jakkoliv orientované L. Pravidelně se střídají v tazích, přičemž Kubazačíná. Prohrává ten, kdo jako první nemůže táhnout. Který z nich má vyhrávající strategii?

(Kuba Krásenský)

4Nemusí být použito žádné.

16

Page 17: Matematický korespondenční seminář Milýpříteli! · těžiště trojúhelníka KNP. Poměr, v kterém těžiště dělí těžnici, je přitom dobře znám – kýžený výsledek

Korespondenční seminář, KAM MFF UK, Malostranské náměstí 25, 118 00 Praha 1

Řešení:Vyhrávající strategii má Bára. Ve svém prvním tahu si vybere některý z prázdných rohů šachovnice(takový určitě existuje, protože Kuba zatím stihl vybarvit jen jeden čtverec 2× 2) a zahraje do nějnásledovně:

Vzniknou tak tři rezervní políčka (v obrázku vyznačena šedě) ve tvaru L taková, že ani jednoz nich nemůže Kuba svým tahem vybarvit. Potom bude hra pokračovat. Bára bude nadále hráttak, že nebude vybarvovat žádné z rezervních políček. Jelikož v každém tahu ubude bílých políček,tak časem nastane situace, že na šachovnici už nebude žádný bílý čtverec 2 × 2. Pokud je v tétosituaci na tahu Kuba, prohrál, protože nemá co zabarvit. Pokud je na tahu Bára, může zabarvittři rezervní políčka. Po tomto jejím tahu Kuba nemá co zabarvit, takže také prohrál.

Poznámky:Úloha byla na pětku celkem jednoduchá a pětibodovými řešeními se to jen hemžilo. Drtivá většinaz nich využívala stejnou myšlenku jako to vzorové. Několik řešitelů navrhlo pro Báru strategiitakovou, že bude hrát vždy na pozici středově symterickou s předchozím Kubovým tahem – tatostrategie funguje také, jen je potřeba dát si pozor na pár technických detailů. (Tonda Češík)

Úloha 6. (71; 52; 3,37; 5,0)Dva kamarádi, Plusík a Mínusík, našli šachovnici 3 × 3 vyplněnou v nějakém pořadí čísly 1 až 9.Plusík umí ke všem číslům v libovolném čtverci 2 × 2 přičíst jedničku, Mínusík umí analogickyodčítat. Poté, co si s šachovnicí chvíli takto hráli, objevilo se ve všech jejích políčkách stejné číslo.Kolik to mohlo být? (Rado Švarc)

Řešení:Na začátku je součet čísel na šachovnici rovný 45. Označme p počet tahů Plusíka a m počet tahůMínusíka. Položme k = p−m. Plusík i Mínusík ovlivňují každým svým tahem 4 políčka. Na koncijejich hry tedy bude součet čísel na šachovnici rovný 45 + 4k.Dále označme e hodnotu, která se původně nacházela ve středu šachovnice. Každý tah Plusíka

i Mínusíka toto číslo mění, a tak na konci bude hodnota na prostředním políčku rovna e + k.Na konci však má být na všech polích šachovnice stejná hodnota, tudíž jejich součet bude rovendevítinásobku čísla na prostředním políčku. Můžeme tedy psát, že

45 + 4k = 9(e+ k),

45 = 9e+ 5k.

To však znamená, že e je dělitelné 5. Jediné číslo, které bylo na počátku napsáno na šachovnici abylo dělitelné pěti, je 5. Tedy e = 5. Z toho již snadno dopočteme, že 5k = 45− 9e = 0, tedy k = 0.Hodnota, která nakonec na šachovnici zůstane, je e + k = 5. Příkladem šachovnice, kde je možnévšechny hodnoty změnit na 5, může být

17

Page 18: Matematický korespondenční seminář Milýpříteli! · těžiště trojúhelníka KNP. Poměr, v kterém těžiště dělí těžnici, je přitom dobře znám – kýžený výsledek

Matematický korespondenční seminář 35. ročník (2015/2016), 2. komentáře

4 1 2

7 5 3

8 9 6

Plusík jednou zvýší hodnoty v levém horním čtverci a třikrát v pravém horním. Mínusík jednousníží čísla v pravém dolním a třikrát v levém dolním. Jediné číslo, které mohlo být na všech políčkáchšachovnice zároveň je tedy 5.

Poznámky:Úlohu bylo možno dokázat mnoha různými postupy. Ti, kteří předvedli rychlé a elegantní řešení,byli odměněni imaginárním bodem. Část řešitelů mylně předpokládala, že se Plusík a Mínusík vesvých tazích musí pravidelně střídat. (Martin Hora)

Úloha 7. (67; 41; 3,12; 4,0)Na šachovnici 2015 × 2015 stálo 2015 věží, z nichž se žádné dvě neohrožovaly. Náhle se všechnyproměnily v tchýně, udělaly jeden tah jako koně a proměnily se zpět ve věže. Dokažte, že nyní senutně nějaké dvě z nich ohrožují. (Anh Dung „Tondaÿ Le)

Řešení:Zavedeme si souřadnice políček tak, že souřadnice levého horního políčka je (1, 1) a souřadnicepravého spodního políčka je (2015, 2015). Pokud se žádné dvě věže neohrožují, musí být každáv řádku i sloupci sama, a protože je věží stejně jako řádků a sloupců, je v každém řádku i sloupciprávě jedna věž.Uvažujme nyní součet x-ových a y-ových souřadnic všech věží. Na začátku se žádné dvě věže

neohrožovaly, proto se každý řádek i každý sloupec vyskytl v součtu právě jednou. Součet byl tudížroven

1 + 1 + 2 + 2 + · · ·+ 2015 + 2015 = 2 · 2015 · 20162

= 2015 · 2016.

Předpokládejme, že se ani po tahu žádné dvě věže neohrožují. Potom je součet jejich souřadnic zase2015 · 2016, takže se nezměnil.Ale každá věž se pohnula dohromady o tři políčka, změnila tedy součet souřadnic o liché číslo.

A protože věží je 2015, dohromady také změnily součet souřadnic o liché číslo, což je spor.

Šachovnicové řešení (podle Jana Šorma):

Lemma. Mějme šachovnici (2k−1)×(2k−1), kde k je přirozené, a na ní 2k−1 věží rozmístěnýchtak, aby se neohrožovaly. Pokud šachovnici obarvíme šachovnicově tak, aby levé horní políčko byločerné, pak je na černých políčkách lichý počet věží.

Důkaz. Indukcí podle k.Pro šachovnici 1× 1 tvrzení platí. (Je tam jedno černé políčko a na něm jedna věž.)Předpokládejme, že tvrzení platí pro 2k−1, a mějme šachovnici (2k+1)×(2k+1) a na ní 2k+1

věží rozmístěných tak, aby se neohrožovaly. Každá věž je sama v řádku i ve sloupci. Rozeberemedvě možnosti:

(i) Pokud je na černých políčkách pouze jedna věž, tvrzení platí.

(ii) Jinak jsou na černých políčkách alespoň dvě věže. Nějaké takové dvě vybereme a odeberemeje i s jejich řádky a sloupci. Tím jsme dostali vyhovující šachovnici (2k − 1) × (2k − 1),na kterou použijeme indukční předpoklad, tedy že tam je lichý počet věží na černýchpolíčkách. A pak zpátky přidáme ty dvě, čímž se parita nezmění.

Nyní je úloha triviální, protože skokem koně změní figura barvu svého políčka. Protože je věží2015, můžeme použít lemma. Na začátku se neohrožují, je jich lichý počet na černých, a tedy sudý

18

Page 19: Matematický korespondenční seminář Milýpříteli! · těžiště trojúhelníka KNP. Poměr, v kterém těžiště dělí těžnici, je přitom dobře znám – kýžený výsledek

Korespondenční seminář, KAM MFF UK, Malostranské náměstí 25, 118 00 Praha 1

počet na bílých políčkách. Skokem koně změní svou barvu, takže bude na černých políčkách sudýpočet věží. Proto se nějaké dvě budou ohrožovat.

Řešení pomocí cyklů v permutacích (podle Adama Španěla):Opět si uvědomíme, že když se věže neohrožují, je v každém řádku i v každém sloupci právě jedna.Nyní si celou šachovnici promítneme na osu x (tzn. máme jeden řádek dlouhý 2015 políček). Na

každém políčku je právě jedna věž. Předpokládejme pro spor, že se věže po skoku neohrožují. Toznamená, že je opět každé políčko zabrané. Nechť první věž skočila na políčko i. Věž, která stálapřed skokem na i, musela skočit na nějaké jiné políčko. Takto pokračujeme, ale protože je políčekjen 2015, musela někdy nějaká věž skočit na políčko 1, čímž nám vznikl cyklus. A my si přesouvánívěží rozdělíme na takové cykly.5

Každá věž mohla skočit o ±2 nebo o ±1 políčko. Aby se cyklus uzavřel, musí být součet skoků0, což je sudé číslo. Takže počet skoků o 1 musel být sudý. A to platí pro každý cyklus, tedycelkový počet skoků o 1 ve všech řádcích musel být sudý. Proto skoků o 2 byl lichý počet (protožedohromady jich bylo 2015).Ale tutéž úvahu můžeme udělat, pokud si šachovnici promítneme na osu y. Každý kůň skočí

o 2 ve směru jedné osy a o 1 ve směru druhé, neboli pokud ve směru x skočil o 2, skočí ve směru yo 1 a opačně. Podle předchozí úvahy ve směru y skočil lichý počet koňů o 1. Ale to je spor, protožeaby se neohrožovali, musel by jich o 1 skočit sudý počet.

Poznámky:Úloha byla na sedmičku poměrně jednoduchá a tomu také odpovídá počet došlých řešení. Několikřešitelů tvrdilo, že jediné možné vyhovující rozmístění je diagonální, což není pravda (dokonceexistuje 2015! vyhovujících rozmístění).6 Mnoho řešení se snažilo ukázat, že pro nějaké konkrétnírozložení věží se je přesunout nepovede, ale úloha chtěla dokázat, že pro každé vyhovující rozloženívěží a každou variantu jejich skoků se nakonec budou nějaké dvě ohrožovat. To je častá chybav chápání úlohy, dávejte si na to pozor, zbytečně pak ztrácíte body. (Matěj Konečný)

Úloha 8. (24; 15; 2,42; 3,0)Kouzelníci Štěpán a David si pro Rada připravili trik s šachovnicí n × n. Nejprve David odešelpryč, aby nic neviděl ani neslyšel. Poté Štěpán Radovi nakázal, ať na každé políčko položí dle svévůle buď bílý, nebo černý knoflík. Následně ho nechal, aby zvolil libovolné políčko A a sdělil mu,které to je. Nato si Štěpán vybral políčko B (ne nutně různé od A) a změnil barvu knoflíku, kterýna B ležel. Když potom přišel David, byl schopný pouze z pohledu na šachovnici uhodnout, kterépolíčko A si Rado vybral. Pro která n je tento trik proveditelný? (Rado Švarc)

Řešení:Nejprve předpokládejme, že je možné trik uskutečnit pro šachovnici n×n. Jakýkoliv způsob, jakýmje možné rozmístit knoflíky na šachovnici, budeme nazývat konfigurací.David je schopen z pouhého pohledu na šachovnici zjistit, které políčko si Rado vybral. Nechť

SP je množina všech konfigurací, podle kterých David pozná, že si Rado vybral políčko P . ProtožeDavid je schopný se jednoznačně rozhodnout, jsou všechny tyto množiny disjunktní.NechťM je to políčko šachovnice, které má nejmenší množinu SM . Protože políček je n2, existuje

2n2

možných konfigurací. Protože žádná konfigurace neleží ve dvou množinách, každá množina mávelikost alespoň |SM | a celkem je jich n2, platí

n2 · |SM | ≤ 2n2

.

Pro každou konfiguraci z SM existuje n2 konfigurací, které se od ní liší jen v barvě jednohoknoflíku. Takže celkově je maximálně n2 · |SM | konfigurací, které se liší jen o jedna od nějaké

5Skoky věží odpovídají nějaké permutaci a my pracujeme s jejím rozkladem na cykly.6n! se čte n faktoriál a značí to součin 1 · 2 · . . . · n.

19

Page 20: Matematický korespondenční seminář Milýpříteli! · těžiště trojúhelníka KNP. Poměr, v kterém těžiště dělí těžnici, je přitom dobře znám – kýžený výsledek

Matematický korespondenční seminář 35. ročník (2015/2016), 2. komentáře

konfigurace z SM . Ovšem aby byl trik uskutečnitelný i v případě, že si Rado vybere M , musí prokaždou konfiguraci existovat konfigurace z SM , která se od ní liší jen v barvě jednoho knoflíku.Protože konfigurací je 2n

2

, dostáváme vztah

n2 · |SM | ≥ 2n2

.

Porovnáním těchto dvou nerovností dostáváme n2 · |SM | = 2n2

, takže n | 2n2 . To znamená, žen musí být mocnina dvou.Nyní ukážeme, že pokud n = 2k pro nějaké nezáporné celé k, je trik už proveditelný. Pro n = 1

je to lehké – David ví, že si Rado zvolil to jediné políčko, které na šachovnici je. Předpokládejmedále, že n > 1. Ukážeme si dva různé způsoby, jak řešení dokončit.

Řešení XORem:Budeme používat takzvaný XOR. Nechť a a b jsou nezáporná celá čísla, která se v binární soustavězapíšou jako a1a2 . . . ak a b1b2 . . . bk (pokud nemají stejný počet cifer, tak to menší doplníme zlevanulami). Pro každé i od jedné do k nechť ci = 0, pokud ai = bi, a ci = 1 v opačném případě. PotomXOR a a b zadefinujeme jako takové číslo a⊕ b, které se v binární soustavě zapíše jako c1c2 . . . ck.Například 12 ⊕ 5 = 11002 ⊕ 01012 = 10012 = 9. Povšimněme si, že a ⊕ a = 0, a pokud a ⊕ b = c,pak a⊕ c = b.Políčka na šachovnici si označíme čísly od 0 do 22k − 1. Potom Štěpán s Davidem postupují

následovně: Nechť x je XOR čísel všech políček, na které Rado položil černý knoflík a nechť a ječíslo políčka A. Potom Štěpán změní barvu knoflíku na políčku s číslem x ⊕ a (takové existuje,protože na šachovnici jsou právě všechna políčka, jejichž čísla mají 2k nebo méně cifer). Po tétozměně bude XOR čísel všech políček s černým knoflíkem roven x ⊕ (x ⊕ a) = a. Proto si Davidpouze spočítá XOR všech políček s černým knoflíkem a dostane A.

Řešení půlením intervalů (podle Františka Coufa):Všechna políčka šachovnice si pomyslně poskládáme za sebe a vytvoříme pole délky n2. To rozdělímena dvě souvislé části, přičemž levou nazveme M1. Následně si každou z částí rozdělíme na dvě alevé půlky obou vložíme do množiny M2. Poté opět každý interval rozdělíme na dvě poloviny a tylevé vložíme doM3. Postupujeme dál a dál, dokud intervaly nejsou velikosti 1. K těm se dostanemepo 2k − 1 krocích.Níže uvedený obrázek prezentuje dělení pro šachovnici o šestnácti políčkách, přičemž Mi je

tvořena všemi šedými částmi v příslušném řádku.

M2

M3

M4

M1

David po příchodu k šachovnici bude postupovat následujícím algoritmem: pokud je v Mi sudýpočet černých knoflíků, přesune se v i-tém kroku do levé poloviny intervalu, ve kterém právě je,jinak se přesune do pravé. Na příkladu na obrázku vidíme, že v M1, M2, M3 a M4 jsou postupně4, 5, 4 a 4 černé knoflíky, a proto se David přesouvá doleva, doprava, doleva a doleva. Za odpověďzvolí to políčko, na kterém skončil.Stačí nám tedy ukázat, že Štěpán umí změnit jeden knoflík tak, aby David skončil přesně na tom

políčku, které Rado zvolil. Štěpán ví, jakou posloupnost příkazů „dolevaÿ a „dopravaÿ musí Davidvykonat. Proto se podívá, zda by ho v i-tém kroku současná konfigurace posílala na správnou, nebo

20

Page 21: Matematický korespondenční seminář Milýpříteli! · těžiště trojúhelníka KNP. Poměr, v kterém těžiště dělí těžnici, je přitom dobře znám – kýžený výsledek

Korespondenční seminář, KAM MFF UK, Malostranské náměstí 25, 118 00 Praha 1

špatnou stranu. Pokud na špatnou, zapamatuje si množinu Mi, jinak si zapamatuje její doplněk.Chce, aby knoflík, který změní, byl ve všech zapamatovaných množinách.Ovšem pomocí jednoduché indukce se lehce ukáže, že průnik prvních i zapamatovaných množin

má průnik právě v jednom intervalu délky 22k−i. Skutečně, pro i = 1 toto platí, a pokud tvrzeníplatí pro i, pak průnik prvních i+1 zapamatovaných množin je buď levá, nebo pravá půlka průnikuprvních i množin. To znamená, že průnik všech množin je interval délky 1, a tudíž Štěpán skutečněnajde knoflík, jehož přebarvením se změní parita počtu černých knoflíků právě těch intervalů, kteréposílaly Davida na špatnou stranu. Díky tomu David najde správné políčko.

Poznámky:Úloha byla na osmičku spíše lehká, a protože sestávala ze dvou částí, mnoho řešitelů skutečně tulehčí vyřešila. Často se ale neshodli na tom, která to je.Krom dvou nastíněných řešení (rozmyslete si, že jde vlastně o to samé řešení, jen jinak zapsané),

se objevilo ještě třetí, které úlohu převedlo na obarvování grafu hyperkrychle Q2k , což dořešiloindukcí. (Rado Švarc)

21

Page 22: Matematický korespondenční seminář Milýpříteli! · těžiště trojúhelníka KNP. Poměr, v kterém těžiště dělí těžnici, je přitom dobře znám – kýžený výsledek

Do nekonečna a ještě dál I1. seriálová série Vzorové řešení

Úloha 1. (33; 23; 3,39; 5,0)Na nějakém celém čísle na číselné ose sedí neviditelná blecha. V každém skoku skočí o pevně danénenulové přirozené číslo n doleva nebo doprava (při každém skoku si může znovu vybrat směr).Mirek se snaží blechu chytit tak, že po každém skoku blechy položí na některé celé číslo past. Blechuchytí, pokud položí past na blechu nebo blecha ve svém skoku skočí do již dříve položené pasti.Ukažte, že Mirek může pasti pokládat tak, aby blechu po konečně mnoha skocích zaručeně chytil, ikdyž nezná počáteční pozici blechy, konstantu n ani směry, kterými blecha skáče. (Mirek Olšák)

Řešení:Nejprve dokážeme, že možných dvojic (m,n), kdem je počáteční pozice a n délka skoku, je spočetněmnoho. Možných výchozích pozic je stejně jako celých čísel, a těch je stejně jako přirozených. Délkaskoku je přirozené číslo, takže dvojic, které nás zajímají, je stejně jako dvojic přirozených čísel.A těch je stejně jako přirozených čísel.7

To ale znamená, že tyto dvojice je možné očíslovat přirozenými čísly a jednu po druhé projít.V nějakém kroku budeme například předpokládat, že parametry blechy mají konkrétní hodnoty(mk, nk). Než jsme se dostali k tomuto předpokladu, uplynulo již i tahů (rozdělených do k − 1skupin, během nichž jsme postupně lovili blechy s parametry (m1, n1) až (mk−1, nk−1)). Pakmůžeme postupovat následovně: Položíme past na číslo mk + nk · i (pokud tam již není) a pak namk−nk · (i+1). Tím je blecha uvězněna na konečném úseku přímky. Následně v libovolném pořadíprojdeme všechny pozice, které uvězněné bleše zbyly – tedy čísla tvarumk+lnk pro l ∈ Z∩〈−i, i−1〉.Takto jsme prošli postupně všechny možnosti, kde blecha mohla začínat a o kolik mohla skákat.

Vyřešení každé z nich nám zabralo konečně mnoho tahů, takže jsme ji bez ohledu na počátečnípolohu a délku skoku po konečném počtu tahů chytili.

Alternativní řešení:Tahy si rozdělíme do trojic. V prvním tahu k-té trojice položíme past na pole k! + k, v druhémna pole −(k! + k) a ve třetím na pole (případně jedno z polí), na kterém ještě není past a které jez takových polí nejblíže nule.Protože faktoriál roste rychleji než libovolná lineární funkce, dosáhneme za určitou dobu toho, že

políčka, která budeme zabírat v prvních a druhých tazích, budou od nuly dále než blecha. Všimněmesi, že se blecha pohybuje pouze po číslech, která dávají po dělení n zbytek m. Kdyby tedy k! + knejen bylo dost velké, ale zároveň dávalo po dělení n zbytek m, podařilo by se nám v k-tém tahuuvěznit blechu na konečném úseku přímky.Pro k ≥ n platí n | k!. To ale znamená, že k!+k se modulo n mění při každém zvýšení k o jedna.

Proto projde během každých n po sobě jdoucích tahů každou možnou zbytkovou třídu modulo n.Tím tedy bude blecha uvězěna v nějakém konečném intervalu. Pomocí třetího kroku naší trojicecelý tento interval časem vyplníme pastmi, a tak blechu chytíme.

7Toto tvrzení je dokázáno v textu seriálu.

22

Page 23: Matematický korespondenční seminář Milýpříteli! · těžiště trojúhelníka KNP. Poměr, v kterém těžiště dělí těžnici, je přitom dobře znám – kýžený výsledek

Korespondenční seminář, KAM MFF UK, Malostranské náměstí 25, 118 00 Praha 1

Alternativní řešení 2:Zvolíme si funkci f :N → N, která roste rychleji než lineárně (například x2 nebo xx), a budemepostupovat následovně: Pokaždé, když budou pasti právě na všech číslech mezi nejpravější a nej-levější pastí, zvolíme si k = f(i), kde i je počet již položených pastí. Následně budeme pokládatpasti popořadě na k, −k, k− 1, −k+1 a tak dále, dokud opět nebudou pasti na všech číslech mezik a −k.Budiž nyní opět m počáteční poloha blechy a n délka skoku. Maximální vzdálenost blechy od

nuly po jejím l-tém skoku můžeme vyjádřit jako ||m|+ l ·n|. V některém z tahů, ve kterém položímepopořadě i-tou past na f(i)-té pole, bude splněna nerovnost

∣|m|+ in+ 2n2∣

∣ < f(i)− n.

Bude platit díky tomu, že levá strana roste lineárně (jediná proměnná je i, zbytek jsou konstanty),kdežto pravá strana roste rychleji. Nyní se podíváme, co tato nerovnost vyjadřuje.Výraz

∣|m|+ in+ 2n2∣

∣ je maximální vzdálenost blechy od nuly po i+2n skocích, kdežto f(i)−nvyjadřuje vzdálenost od nuly, v jaké se nám podařilo (v (i+ 2n)-tém kroku) položit na n po sobějdoucích čísel pasti (a to stejný počet symetricky jak na kladné, tak na záporné části osy). Podařilose nám tedy vytvořit bariéru, kterou blecha neumí přeskočit, a rozhodně se k ní nedostala dříve,než jsme ji dostavěli. Nyní je tedy blecha lapena mezi dvěma barikádami, a následným zaplněnímprostoru mezi nimi ji nutně chytíme.

Poznámky:Téměř každé řešení, které dorazilo, bylo originál, ať už volbou funkce f u řešitelů postupujícíchpodle poslední verze řešení, nebo popisem seřazení dvojic počáteční polohy blechy a délky jejíhoskoku při řešení prvního typu.Určitě se hodí poznamenat, že první řešení má oproti zbývajícím dvěma jednu výhodu: dalo by se

úplně stejně aplikovat i v případě, že by blecha skákala například po racionálních číslech o racionálníhodnotu, či v jakémkoli jiném případě, kdy je možno všechny možné počáteční konfigurace zapsatpomocí nějaké n-tice přirozených čísel a zároveň platí, že umíme blechu v konečném počtu krokůchytit, známe-li tento počáteční stav a počet kroků, který od začátku hry uběhl.Pozitivně mě překvapilo, že většina řešitelů vyřešila úlohu správně nebo skoro správně, a i většina

špatných řešení obsahovala alespoň správnou myšlenku. Jestliže někde byla chyba, tak zpravidlav tom, že některá tvrzení nebyla dokázána, nebo v tom, že řešiteli uniklo, že mu z jeho konstrukcemůže umět blecha pro nějaká m, n vždy utéct. (Viki Němeček)

Úloha 2. (32; 19; 2,97; 3,0)Nechť X je množina všech bijekcí R → R. Ukažte |X| ≥ |P(R+)|, kde symbol R+ značí množinuvšech kladných reálných čísel. (Mirek Olšák)

Řešení:Abychom dokázali, že |X| ≥ |P(R+)|, stačí najít prosté zobrazení g z P(R+) do X. Podmnožiněkladných reálných čísel M přiřadíme následující reálnou funkci f :

f(x) =

{

−x, pokud x ∈Mx, jinak.

Tímto způsobem jsme definovali zobrazení g, můžeme tedy psát g(M) = f .Je funkce f bijekcí? Pokud mají dvě čísla a 6= b různé absolutní hodnoty, zobrazí se na různá

čísla. Pokud je mají stejné, jedná se o opačná čísla, a ta se zobrazí na vzájemně opačné hodnoty– buď se prohodí, nebo obě zůstanou na místě. Proto je f prostá. Dále je funkce f také na, neboťpro reálné číslo a se −a zobrazí na a, pokud |a| ∈M , a jinak se a zobrazí na samo na sebe. Z toho,že f je prostá a na, plyne, že je skutečně bijekcí.

23

Page 24: Matematický korespondenční seminář Milýpříteli! · těžiště trojúhelníka KNP. Poměr, v kterém těžiště dělí těžnici, je přitom dobře znám – kýžený výsledek

Matematický korespondenční seminář 35. ročník (2015/2016), 2. komentáře

Víme tedy, že g zobrazuje z P(R+) do X. Nyní stačí ukázat, že g je prosté. Nechť N je podmno-žina kladných reálných čísel různá od M . Určitě existuje takové kladné reálné x, že x leží v právějedné z množin M , N . Pak se hodnota f(x) pro tyto dvě podmnožiny liší, neboť v jednom případědostaneme x a v druhém −x.

Poznámky:Potěšilo mě, že se navzdory obtížnosti a abstraktnímu tématu seriálu sešlo mnoho správných řešení.Řešitelům, kteří postupovali jako ve vzorovém řešení, jsem udělil +i. Častou chybou byla konstrukcefunkce z P(R+) do X, která využívala očíslování podmnožiny M přirozenými čísly. PodmnožinaM může být i nespočetná, a proto takové očíslování nemusí existovat. (Anh Dung „Tondaÿ Le)

Úloha 3. (12; 3; 1,25; 0,0)Najděte takové dvě nekonečné DUMy A, B, aby platilo A · B ≃ B. Zdůvodněte, proč se jednáo DUMy, a popište příslušnou rostoucí bijekci. (Mirek Olšák)

Řešení:Uvažme množinu všech konečných posloupností přirozených čísel, do které ještě přidáme prázdnouposloupnost ∅. Tuto množinu uspořádáme primárně podle délky a sekundárně standardně lexiko-graficky odzadu. Máme tedy

∅ < (0) < (1) < (2) < · · · < (0, 0) < (1, 0) < (2, 0) < · · · < (42, 42, 42) < (43, 42, 42) < · · · .

Toto uspořádání je dobré, protože je lineární a každá podmnožina má nejmenší prvek – nejprvez podmnožiny vezmeme nejkratší posloupnosti a mezi nimi pak najdeme tu nejmenší v lexikografic-kém uspořádání. Popsaná množina posloupností je tedy DUM, označme ji jako X. Nyní definujmezobrazení f :ω ·X → X jako

(1) f(

(0, ∅))

= ∅,(2) f

(

(n, ∅))

= (n− 1) pro n > 0,(3) f

(

n, (x1, x2, . . . , xk))

= (n, x1, . . . , xk) pro k ≥ 1.Tato funkce je definovaná pro každou dvojici (n, x), kde n ∈ ω a x ∈ X, a je zřejmě prostá a na– body (1) a (2) pokryjí nejvýše jednoprvkové posloupnosti a bod (3) zbytek. Funkce f je tedybijekce. Je rostoucí? Pokud (m,x) < (n, y) pro (m,x), (n, y) ∈ ω ·X, znamená to, že nastává některáz těchto možností:

(i) x = y = ∅. Pak musí být m < n, a tedy i f(

(m,x)) < f((n, y))

vzhledem k bodům (1) a(2).

(ii) ∅ = x < y. f(

(m,x))

je nejvýše jednoprvková, zatímco f(

(n, y))

je alespoň dvouprvková.

(iii) ∅ < x < y. Posloupnost f(

(m,x))

bez prvního členu je x a posloupnost f(

(n, y))

bezprvního členu je y; protože x < y a uspořádání uvažujeme „odzaduÿ, dostáváme takf(

(m,x))

< f(

(n, y))

.

(iv) ∅ < x = y. Zde musí být m < n. Vztah f(

(m,x))

< f(

(n, y))

plyne z porovnání prvníchprvků těchto posloupností.

Našli jsme rostoucí bijekci a dokázali tak ω ·X ≃ X, takže DUMy A = ω a B = X vyhovují zadání.

Poznámky:Jak se na řešení dalo přijít? To se v první části těchto poněkud delších poznámek pokusím popsat.Součin A · B si můžeme představit tak, že postupně procházíme8 prvky B (rostoucím způsobem

8Můžete namítnout, že jiné nekonečné DUMy než ω takto celé projít neumíme, a budete mítpravdu. Nicméně je tento pohled stále užitečný, takže si pojďme pojďme představit „procházeníÿlibovolně dlouhých DUM. Koneckonců přesně to umí transfinitní indukce a rekurze.

24

Page 25: Matematický korespondenční seminář Milýpříteli! · těžiště trojúhelníka KNP. Poměr, v kterém těžiště dělí těžnici, je přitom dobře znám – kýžený výsledek

Korespondenční seminář, KAM MFF UK, Malostranské náměstí 25, 118 00 Praha 1

vzhledem k jejímu uspořádání), ale místo každého prvku projdeme kopii DUMy A, opět vzestupněvzhledem k uspořádání A. DUMa A musí být nekonečná, ale zároveň tím, že budeme v B brátmísto každého prvku kopii A, chceme dostat stejný typ. Dává tedy smysl volit A co nejmenší, tedyjako ω. Za B chceme naopak zvolit co největší DUMu, aby ji násobení moc nezměnilo. V seriáluje uvedeno, že 2 · ω ≃ ω, obdobně to platí i pro n · ω. Kdyby tedy mohla být množina A konečná,je úloha vyřešena. Bohužel ale ω · ω 6≃ ω a podobně ani ω · (ω · n) 6≃ ω · n. Pro nekonečné A jetedy potřeba volit větší B. Podívejme se, jaké větší DUMy známe. Co takhle zkusit ω · ω? Podledefinice násobení je to množina všech dvojic přirozených čísel s porovnáváním primárně podle druhésložky. A co ω · (ω · ω)? Bude to vlastně to samé jako (ω · ω) · ω? Opět použitím pouhé definicenásobení si snadno rozmyslíme, že obojí odpovídá trojprvkovým posloupnostem přirozených čísels lexikografickým uspořádáním – v prvním případě přidáváme k dvojicím číslo na tu nejménědůležitou – první – pozici a ve druhém před číslo přidáváme méně důležitou dvojici. Ze stejnéhodůvodu dává smysl definovat pro n > 1 DUMu ωn = ω · ωn−1 = ωn−1 · ω jako lexikografickyuspořádané posloupnosti n přirozených čísel. Žádnou takovou DUMu nemůžeme použít jako B(pro A = ω), protože vynásobení ω zvýší exponent o jedničku.Mohli bychom použít množinu nekonečných posloupností? Tu by podle výše použitých úvah

přenásobení ω určitě nezměnilo, ale máme jiný problém – nekonečné posloupnosti s lexikografickýmuspořádáním netvoří ani spočetnou, ani dobře uspořádanou množinu! Pro nespočetnost viz seriála nekonečnou klesající posloupnost pro vyvrácení dobrého uspořádání jistě zvládnete najít sami.Abychom se existenci této posloupnosti vyhnuli, nezbývá už než zkusit konstrukci popsanou výše.:-)Úloha byla obtížná a obdrželi jsme pouze tři správná řešení, všechna myšlenkově shodná se

vzorovým. Nejčastějším chybou těch ostatních byla volba A = B = ω s bijekcí ω ·ω → ω popsanouv kapitole seriálu věnované Hilbertovu hotelu. Tato bijekce je dobrým zdůvodněním toho, že jsouobě příslušné množiny stejně velké, ale není rostoucí vzhledem k uspořádání DUMy ω · ω. Dáse sice říct, že bijekce ω → M čísluje prvky množiny M , takže je „rostoucíÿ, to jsme ale na Mpřenesli uspořádání z ω, takže o jejím vlastním uspořádání (pokud to byla DUM) z toho nic usouditnemůžeme. DUM X ze vzorového řešení se obvykle nazývá ωω a jistou představu o ní lze získatstudiem zajímavého obrázku9. (David Hruška)

9https://upload.wikimedia.org/wikipedia/commons/e/e6/Omega-exp-omega-labeled.svg

25

Page 26: Matematický korespondenční seminář Milýpříteli! · těžiště trojúhelníka KNP. Poměr, v kterém těžiště dělí těžnici, je přitom dobře znám – kýžený výsledek

Matematický korespondenční seminář 35. ročník (2015/2016), 2. komentáře

Do nekonečna a ještě dál

Na počátku bylo slovo, a to slovo bylo od Matematika, a to slovo bylo „množinaÿ.

Díl druhý – pevné základy

V tomto díle si ukážeme, jak se matematika asi před sto dvaceti lety rozbila, a především to, jak jinásledně opravili. Nenavážeme tedy na první díl hned, ale až zhruba v polovině seriálu.Možná totiž v prvním díle nebylo jasné, proč při snaze sestrojit obrovské množiny používáme tak

rafinované konstrukce namísto toho, abychom prostě úplně všechny množiny naházeli do jednohopytle. To už bude určitě největší množina. Nic většího nevyrobíme. Jenže v tom je právě ta potíž,vždyť jsme si ukázali, že vždycky můžeme sestrojit větší množinu pomocí potence. Jak to? Když sivzpomeneme na důvod, proč je potence větší než původní množina, dostáváme Russelův paradox:Russel: „Existuje množina všech množin?ÿIntuice: „Ovšem že ano. Proč by měla neexistovat?ÿRussel: „A obsahuje tato množina sama sebe?ÿIntuice: „Jasně, obsahuje přece všechny množiny.ÿRussel: „A existuje množina všech množin, které neobsahují samy sebe?ÿIntuice: „Jasně, stačí z množiny všech množin vyhodit ty prvky, které samy sebe obsahují.ÿRussel: „A obsahuje tato množina sama sebe?ÿIntuice: „Jejda.ÿProblém spočívá v tom, že z definice množiny všech množin neobsahujících samu sebe vyplývá,

že tato množina se obsahuje právě tehdy, když se neobsahuje. Obě varianty vedou ke sporu.Matematika ale nesmí vést sama o sobě ke sporu. Když jsme dostali spor, znamená to, že jsme

ji špatně vybudovali. Po objevu Russelova paradoxu matematici bádali nad tím, jak zařídit, abyfungovala veškerá dosud vybudovaná matematika, ale již nikoli Russelův paradox.S řešením přišli pánové Zermelo a Fraenkel. Sestavili pro matematiku sadu axiomů – to jsou

tvrzení, která se bez důkazu považují za pravdivá, jedná se tedy o základní kameny matematiky.Podle těchto axiomů není množinou jen tak ledaco, axiomy dávají pro tvorbu množin striktnípravidla. Nemůžeme si tedy jen tak vzít množinu všech množin. Naopak Russelův paradox sporemdokazuje, že žádná množina všech množin ve skutečnosti neexistuje.

Jazyk teorie množin

Napřed si představíme formální jazyk, ve kterém jsou axiomy psané. Jedná se o jazyk hovořícío množinách, žádné jiné matematické objekty v něm zastoupeny nejsou. Možná si říkáš: „No mo-ment, a co prvky těch množin? O těch se mluvit nedá? Co přirozená čísla? Co body v rovině?Co posloupnosti? Co funkce?ÿ Inu, vše tohle budou zase množiny.10 Časem si definujeme, která

10Se situací, kdy prvky množiny byly opět množiny, ses v minulém díle setkal(a) napříkladu potence P(X) – prvky potence jsou podmnožiny X, tedy množiny.26

Page 27: Matematický korespondenční seminář Milýpříteli! · těžiště trojúhelníka KNP. Poměr, v kterém těžiště dělí těžnici, je přitom dobře znám – kýžený výsledek

Korespondenční seminář, KAM MFF UK, Malostranské náměstí 25, 118 00 Praha 1

množina je přirozeným číslem, která bodem v rovině, a tak podobně. V základním jazyku teoriemnožin si však vystačíme s následujícími symboly.

(i) Závorky a klasické logické spojky11: ¬ (není pravda, že), ∧ (a zároveň), ∨ (nebo), ⇒(implikuje), ⇔ (právě tehdy, když).

(ii) Proměnné – písmenka, která zastupují nějakou množinu.(iii) Kvantifikátory: Symbol (∀x) (pro všechna x) znamená, že následující výrok platí, ať za x

dosadíme kteroukoli množinu, a symbol (∃x) (existuje x) znamená, že je možné v násle-dujícím výroku dosadit za x nějakou množinu tak, aby byl splněn.

(iv) Rovnítko x = y značí, že x a y jsou tytéž množiny.(v) Náležítko x ∈ y značí, že množina x je prvkem množiny y. Pozor! Prvek množiny apodmnožina množiny jsou zásadně odlišné pojmy. Prvky množiny často nebývají jejímipodmnožinami. Je rozdíl mezi množinou x a jednoprvkovou množinou obsahující x jakoprvek.

Z koncepce tohoto jazyka je patrné, proč se o množinách říká, že „Prvky množiny jsou neuspo-řádané a nemohou se opakovatÿ. Samotný jazyk totiž neumožňuje zjistit, v jakém pořadí prvkyv množině jsou a kolikrát. Jediné, na co se lze ptát, je, zda x ∈ y, či nikoli.Pomocí těchto symbolů pak lze skládat takzvané formule – to je něco jako výroky. Formule jsou

takové nápisy, které jdou smysluplně přečíst. Formulí je třeba (x = y) ∧ (∃z)(z ∈ x), což se přečtejako „x je stejná množina jako y a zároveň existuje z, které je prvkem množiny x.ÿ Zato například∃¬) =∈ x formulí není, protože je to zkrátka blbost. Exaktně můžeme formuli definovat jako to, covznikne opakovaným použitím následujících pravidel:

(i) Formulemi jsou x ∈ y a x = y (případně s jinými proměnnými).(ii) Nechť ϕ je formule.12 Pak přidáním kvantifikace na začátek vznikne opět formule. Tedy(∀x)(ϕ) a (∃x)(ϕ) jsou formule.

(iii) Aplikováním logických spojek na formule opět vzniknou formule. Tedy z formulí ϕ, ψvzniknou například formule ¬(ϕ) nebo (ϕ)⇒ (ψ).

Pokud nedojde k nejednoznačnosti, lze závorky v zápisu vynechat. Ke kvantifikátoru v takovémpřípadě náleží jen to, co po něm bezprostředně následuje: (∃x)(ϕ) ∧ (ψ) znamená

(

(∃x)(ϕ))

∧ (ψ).Příklad. Ukážeme konstrukci formule (x = y)⇒ (∀z)(x ∈ z ⇔ y ∈ z). Z bodu (i) jsou formulemiformální nápisy x = y, x ∈ z a y ∈ z. Aplikováním bodu (iii) získáme formuli x ∈ z ⇔ y ∈ z. Dález bodu (ii) je formulí (∀z)(x ∈ z ⇔ y ∈ z) a nakonec opět z bodu (iii) dostaneme formuli

x = y ⇒ (∀z)(x ∈ z ⇔ y ∈ z).Ještě přeložme tuto formuli do češtiny: „Pokud jsou x a y stejné objekty (x = y), tak pro libovolnoumnožinu z platí, že x je prvkem množiny z právě tehdy, když y je prvkem množiny z.ÿ Říká tedyjen to, že když jsou x, y stejné prvky, tak leží ve stejných množinách. To platí v každém případě,čili tato formule platí obecně, pro libovolnou volbu13 x, y. Zato formule, které jsme vytvořili cestou(například x ∈ z) nejsou obecně pravdivé.Příklad. Chceme napsat formuli „Množina x je jednoprvková.ÿ Formule se může opírat pouzeo to, které prvky leží v této množině, tedy „Existuje jeden prvek y množiny x takový, že každýprvek z množiny x musí být roven onomu jednomu prvku y.ÿ To lze říci ještě formálněji: „Existujey takové, že y je prvkem x a zároveň pro každé z platí, že pokud je z prvkem x, tak je z rovno y.ÿToto již lze přímočaře přepsat do řeči symbolů:

(∃y)(y ∈ x ∧ (∀z)(z ∈ x⇒ z = y)).

11Logické spojky jsou popsány například na http://www.matematika.cz/vyroky.12Řecké písmenko ϕ označující nějakou formuli se čte „fíÿ. Dále budeme pro formule používat

písmenko „psíÿ ψ.13Pokud zvolíme x, y různé, tak vyjde formule pravdivá z toho důvodu, že není splněn předpo-

klad implikace.

27

Page 28: Matematický korespondenční seminář Milýpříteli! · těžiště trojúhelníka KNP. Poměr, v kterém těžiště dělí těžnici, je přitom dobře znám – kýžený výsledek

Matematický korespondenční seminář 35. ročník (2015/2016), 2. komentáře

O něco stručnější způsob zápisu by mohl být: „Existuje objekt y takový, že pro každý objekt z jez prvkem x právě tehdy, když je z roven y.ÿ Formule vypadá takto:

(∃y)(

∀z)(z ∈ x⇔ z = y)

.

Cvičení 1. Napiš formuli, která říká: „Množina x′ obsahuje stejné prvky jako x a ještě jedennavíc.ÿ

Abychom nemuseli všude psát samá náležítka, zavedeme ještě běžně používané zkratky.

(i) x 6∈ y resp. x 6= y znamená ¬(x ∈ y) resp. ¬(x = y).(ii) x ⊂ y (x je podmnožinou množiny y) znamená (∀z)(z ∈ x⇒ z ∈ y).(iii) Pokud za ∀x okamžitě následuje požadavek na x, například (∀x ∈ y)(. . .), jedná se

o zkratku za to, že příslušný požadavek je předpokladem implikace, tedy (∀x)(

(x ∈ y)⇒(. . .)

)

.(iv) Pokud za ∃x okamžitě následuje požadavek na x, například (∃x ⊂ y)(. . .), jedná se

o zkratku za to, že příslušný požadavek je současně vyžadován, tedy (∃x)(

(x ⊂ y)∧ (. . .))

.(v) Zápis (∀x, y, z) je jen zkratka za sled kvantifikátorů (∀x)(∀y)(∀z), obdobně pro existenčníkvantifikátor.

(vi) Zápis y = {x : ϕ(x)} (množina daná předpisem), kde ϕ(x) je formule,14 značí, že y jemnožina těch prvků x, které splňují formuli ϕ. Jedná se tedy o formuli (∀x)(x ∈ y ⇔ ϕ(x)).

Pozastavíme se u posledního bodu. Ten dává návod, jak interpretovat y = {x : ϕ(x)}, alejiž neříká, jak do základního jazyka přeložit samotnou množinu danou předpisem, tedy samotné{x : ϕ(x)}. Pokud se ve formuli vyskytne taková množina, přeložíme ji do jazyka teorie množin tak,že založíme novou proměnnou y, která se ve formuli dosud nevyskytuje. Tou nahradíme (jeden)výskyt {x : ϕ(x)} a před vzorec s tímto výskytem připíšeme (∃y = {x : ϕ(x)}).Příklad. Přepíšeme do základního jazyka formuli

{x : x ∈ A ∧ x ∈ B} = {x : x ∈ C ∧ x ∈ D}.

Nejprve nahradíme první výskyt do tvaru

(∃y = {x : x ∈ A ∧ x ∈ B})(y = {x : x ∈ C ∧ x ∈ D})

neboli(∃y)

(

(y = {x : x ∈ A ∧ x ∈ B}) ∧ (y = {x : x ∈ C ∧ x ∈ D}))

.

Nyní již můžeme nahradit výrazy podle bodu (vi):

(∃y)(

(∀x)(

x ∈ y ⇔ (x ∈ A ∧ x ∈ B))

∧ (∀x)(

x ∈ y ⇔ (x ∈ C ∧ x ∈ D))

)

.

Poznámka. Skutečnost, že můžeme množinu danou předpisem napsat, ještě nezaručuje její exis-tenci a jednoznačnost. Jednoznačnost vyplyne z axiomu extensionality, existence v některých pří-padech nebude možná vůbec (jak ukazuje Russelův paradox). Pokud si tedy například formálněrozepíšeme tvrzení a ∈ {x : x = x}, zjistíme, že jsme dostali nepravdivou formuli (kde nepravdivostvyplývá z neexistence množiny všech množin, která vyplyne až z axiomu vydělení).

Za často používané typy množin daných předpisem zavedeme další zkratky. V těchto případechz axiomů vyplyne dokonce existence.

(i) {x0, . . . , xn} (množina zadaná výčtem prvků) značí {z : z = x0 ∨ · · · ∨ z = xn}.(ii) x ∩ y (průnik x a y) značí množinu {z : z ∈ x ∧ z ∈ y}.(iii) x \ y (množinový rozdíl x minus y) značí množinu {z : z ∈ x ∧ z 6∈ y}.(iv) x ∪ y (sjednocení x a y) značí množinu {z : z ∈ x ∨ z ∈ y}.(v) P(x) (potence x) značí množinu {y : y ⊂ x}.

14To, že se tato formule jmenuje ϕ(x), a ne jen ϕ, naznačuje, že se x v této formuli nejspíšebude vyskytovat, a že tedy na x závisí její platnost.

28

Page 29: Matematický korespondenční seminář Milýpříteli! · těžiště trojúhelníka KNP. Poměr, v kterém těžiště dělí těžnici, je přitom dobře znám – kýžený výsledek

Korespondenční seminář, KAM MFF UK, Malostranské náměstí 25, 118 00 Praha 1

Příklad. Formule a ∈ {c, {d}} se do základního jazyka přepíše takto:

(∃y0)(∃y1)(

a ∈ y1 ∧ (∀z)(z ∈ y0 ⇔ z = d) ∧ (∀z)(

z ∈ y1 ⇔ (z = c ∨ z = y0))

)

.

Pokud před dvojtečku napíšeme složitější výraz než jednu proměnnou, značíme množinu všechmožných hodnot takového výrazu. Například pokud máme zafixované množiny x a y, značí {{a, b} :a ∈ x, b ∈ y} množinu {c : (∃a ∈ x)(∃b ∈ y)(c = {a, b})}.Nyní již se takřka můžeme pustit do axiomů, tedy formulí, jejichž platnost se v teorii množin

automaticky předpokládá. Poslední věc, kterou je třeba o těchto axiomech pochopit, je, že se nejednáo axiomy logiky. K logice budeme přistupovat intuitivně – například chápeme, že když x = y, takmůžeme ve formuli nahradit x za y a nezmění se tím její platnost. Nebo že když platí ϕ ∨ ψ aneplatí ϕ, tak platí ψ. Popsat a vysvětlit axiomy logiky a formální dedukci by bylo na mnohemdelší povídání. Axiomy teorie množin jsou od toho, aby jasně definovaly, jak se chová náležítko, atedy, co přesně můžeme dělat s množinami.

Axiomy

Za každým axiomem je vysvětleno, co říká a k čemu slouží. Ale v principu by tento doprovodnýtext nemusel být třeba. Jestli si chceš popřemýšlet, zkus nejprve pokaždé pochopit axiom bez něj.(0) Axiom existence:

(∃x)(∀y)(y 6∈ x).

Česky řečeno: Existuje množina x taková, že pro žádnou15 množinu y není y prvkem množiny x.Jinými slovy x nemá žádný prvek. Tuto množinu x budeme nazývat prázdná množina a budeme jiznačit ∅.(1) Axiom extensionality:

(∀z)(z ∈ x⇔ z ∈ y)⇒ (x = y).

Tento axiom říká, že množina není dána ničím jiným než svými prvky. Tedy například prázdnámnožina, průnik, sjednocení, potence, množina zadaná výčtem prvků či obecná množina zadanápředpisem je (pokud existuje) dána jednoznačně. Opačná implikace – že stejné množiny obsahujístejné prvky – také platí. Ta vyplývá přímo z axiomů logiky, považujeme ji tedy za ještě samozřej-mější než axiomy teorie množin.(2) Axiom dvojice:

(∀x, y)(∃d)(d = {x, y})

neboli pro každé dvě zadané množiny existuje množina, která obsahuje právě je. Tento axiom mimojiné zaručuje i existenci jednoprvkových množin {x}, protože {x} = {x, x}.Cvičení 2. Dokaž „opak extensionalityÿ: (∀z)(x ∈ z ⇔ y ∈ z)⇒ (x = y).

(3) Schéma axiomů vydělení: To, že se jedná o schéma, znamená, že za jistý kus axiomu jemožné dosadit skoro jakoukoli formuli a dá se říci, že takto vlastně vyrobíme nekonečně mnohoaxiomů. V tomto případě je možné za ϕ(z) dosadit formuli, která v sobě neobsahuje y. Pak jeaxiomem

(∀x)(∃y)(∀z)(

z ∈ y ⇔ (z ∈ x ∧ ϕ(z)))

.

Jinými slovy toto schéma zaručuje existenci množiny y dané předpisem {z : z ∈ x∧ϕ(z)}. Jedná seo vydělení těch prvků z množiny x, které splňují formuli ϕ(x), výslednou množinu budeme značiti {z ∈ x : ϕ(z)}. Použitím axiomu vznikne vždy podmnožina nějaké existující množiny, takže kekonstrukci množiny všech množin tento axiom použít nelze.

15V češtině má slovo „žádnáÿ takřka stejný význam jako slovo „každáÿ. Sloveso rozhoduje, kteréz těchto dvou slov se má použít. Divně se zde chová čeština, nikoli formální jazyk.

29

Page 30: Matematický korespondenční seminář Milýpříteli! · těžiště trojúhelníka KNP. Poměr, v kterém těžiště dělí těžnici, je přitom dobře znám – kýžený výsledek

Matematický korespondenční seminář 35. ročník (2015/2016), 2. komentáře

Příklad. Axiom vydělení je možné použít i ke konstrukci průniku a∩ b. Za formuli ϕ(z) zvolímez ∈ b. Dále za x dosadíme a a použijeme axiom. Dostáváme y splňující (∀z)

(

z ∈ y ⇔ (z ∈ a∧z ∈ b))

.Tedy y = a ∩ b.Cvičení 3. Odvoď pomocí axiomu vydělení existenci množiny a \ b.(4) Axiom sjednocení:

(∀x)(∃s)(∀z)(

z ∈ s⇔ (∃y ∈ x)(z ∈ y))

.

Tento axiom říká, že kdykoli máme množinu x, můžeme sjednotit všechny množiny, které v x leží.Toto sjednocení s značíme

x. Použití axiomu dvojice a následně axiomu sjednocení zaručujeexistenci sjednocení dvou množin x ∪ y.

x⋃

x

Navíc můžeme pokračovat a sestavit pomocí tohoto axiomu libovolně velkou konečnou množinudanou výčtem {x0, x1, . . . , xn−1}. Existují totiž jednoprvkové množiny {x0}, {x1}, . . . , {xn−1} apostupným aplikováním sjednocení dvou množin z nich dostáváme {x0, x1}, {x0, x1, x2}, . . ., ažnakonec získáme {x0, x1, . . . , xn−1}.(5) Axiom potence:

(∀x)(∃y)(y = P(x)).Cvičení 4. Přepiš axiom potence (tedy tvrzení „existuje potence xÿ) jen pomocí základníhojazyka teorie množin.

(6) Schéma axiomů nahrazení: Mějme formuli ψ(x, y), která v sobě neobsahuje b, y0 ani y1.Symbolem ψ(x, y0) resp. ψ(x, y1) značíme úpravu formule ψ(x, y), ve které nahradíme proměnnouy za proměnnou y0 resp. y1. Pak je axiomem

(∀x, y0, y1)(

(ψ(x, y0) ∧ ψ(x, y1))⇒ (y0 = y1))

⇒ (∀a)(∃b)(∀y)(

y ∈ b⇔ (∃x ∈ a)(ψ(x, y)))

.

I když je tohle schéma na první pohled opravdu nestravitelné, překvapivě to s ním není až takhrozné. Nechť f je zobrazení, které množině x přiřadí množinu y, aby platilo ψ(x, y). Celá prvnízávorka je jen podmínka požadující, aby f bylo jednoznačně určené zobrazení, tedy aby bylo k jed-nomu x přiřazeno jen jedno y. Axiom pak říká, že obrazy prvků množiny a (přesněji té její části,která leží v definičním oboru) v zobrazení f opět tvoří množinu.Pokud budeme chápat zápis f(x) = y jako formální ψ(x, y) můžeme axiom přepsat do tvaru

(∀x, y0, y1)(

((f(x) = y0) ∧ (f(x) = y1))⇒ (y0 = y1))

⇒ (∀a)(∃b)(b = {f(x) : x ∈ a}).

(7) Axiom fundovanosti (nebo též regularity):

(∀x 6= ∅)(∃y ∈ x)(∀z ∈ y)(z 6∈ x).

Tento axiom požaduje, aby každá neprázdná množina obsahovala prvek, který je s ní disjunktní16.Není to intuitivní požadavek a my nebudeme tento axiom příliš potřebovat. Smyslem tohoto axiomuje vyloučit existenci divných množin.

16Slovo disjunktní znamená neprotínající se, tedy že dané množiny mají prázdný průnik.

30

Page 31: Matematický korespondenční seminář Milýpříteli! · těžiště trojúhelníka KNP. Poměr, v kterém těžiště dělí těžnici, je přitom dobře znám – kýžený výsledek

Korespondenční seminář, KAM MFF UK, Malostranské náměstí 25, 118 00 Praha 1

Cvičení 5.

(i) Ukaž, že nemůže existovat množina a splňující a ∈ a.(ii) Ukaž, že nemohou existovat množiny a, b takové, že (a ∈ b) ∧ (b ∈ a).

(8) Axiom nekonečna:

(∃m)(

∅ ∈ m ∧ (∀x ∈ m)(x ∪ {x} ∈ m))

.

Tento axiom zaručuje existenci nekonečné množiny m jistou konkrétní konstrukcí, ačkoli jedno-značně množina m určená není. Podstata tohoto axiomu spočívá v sestrojení alespoň nějaké ucho-pitelné nekonečné množiny. Se vzniklou množinou se setkáme při konstrukci množiny všech přiro-zených čísel.

Cvičení 6. Přepiš axiom nekonečna jen pomocí základního jazyka teorie množin.

(9) Axiom výběru:

(∀a)(

(∀x ∈ a, y ∈ a)(x 6= y ⇔ x ∩ y = ∅)⇒ (∃b)(∀x ∈ a)(∃y)(x ∩ b = {y}))

.

Tento axiom říká, že pokud máme množinu a neprázdných disjunktních množin, tak je možnéz každé množiny vybrat po jednom prvku a sestavit z těchto prvků množinu b. Intuitivně tentoaxiom říká, že je možné provést nekonečně mnoho nahodilých výběrů najednou. Podrobněji se mubudeme věnovat v příštím díle.

Tato (obvykle používaná) sada axiomů trochu překvapivě není minimální – tři axiomy by bylomožné vyškrtnout, aniž bychom o jejich platnost přišli, jak ukazuje následující cvičení.

Cvičení 7.

(i) Uvědom si, že platnost axiomu existence plyne z axiomu nekonečna.(ii) Odvoď schéma axiomů vydělení ze schématu axiomů nahrazení.(iii) Odvoď axiom dvojice z axiomů existence, potence a nahrazení.

To, že se uvádějí i nadbytečné axiomy, je dáno historickými a pedagogickými důvody – přecijen je snazší pochopit axiom dvojice než axiom nahrazení. Dalším důvodem je, že se občas uvažujíjiné teorie množin bez axiomu potence či bez axiomu nahrazení, ale těmito verzemi se zabývatnebudeme.

Dvojice a kartézský součin

Nyní si ukážeme, jak pomocí množin vytvořit některé známé struktury, které se od běžných množinliší.Neuspořádaná dvojice obsahující prvky a a b je množina {a, b}. Obecně se tedy jedná o dvou-

prvkovou nebo jednoprvkovou množinu.Uspořádaná dvojice (a, b) je množina {{a}, {a, b}}.

Cvičení 8. Ukaž, že uspořádaná dvojice (∅, {∅}) je prvkem P(P(P(P(∅)))).Následující cvičení říká, že uspořádané dvojice se chovají tak, jak bychom chtěli, tedy že jedno-

značně určují svůj první i druhý prvek.

Cvičení 9. Ověř, že pokud (a0, b0) = (a1, b1), tak už nutně a0 = a1 a b0 = b1.

Kartézský součin A × B je množina obsahující všechny uspořádané dvojice (a, b), kde a ∈ A,b ∈ B, lze jej tedy zapsat jako {(a, b) : a ∈ A, b ∈ B}.Příklad. Ukážeme z axiomů, že pro libovolné dvě množiny A, B existuje kartézský součin A×B ={(a, b) : a ∈ A, b ∈ B}. Pro každé a, b plyne existence dvojice (a, b) z trojnásobného použití axiomu

31

Page 32: Matematický korespondenční seminář Milýpříteli! · těžiště trojúhelníka KNP. Poměr, v kterém těžiště dělí těžnici, je přitom dobře znám – kýžený výsledek

Matematický korespondenční seminář 35. ročník (2015/2016), 2. komentáře

dvojice.17 Dále zafixujeme a a definujeme formuli ψ(x, y) jako y = (a, x). Díky axiomu nahrazeníexistuje množina obrazů všech b ∈ B v tomto zobrazení, tedy množina Xa = {(a, b) : b ∈ B}.Tím už jsme skoro hotovi, stačí sjednotit množiny Xa pro a ∈ A. Množina Xa existuje, ať zvolímekterékoli a. Uvažme jinou formuli ψ(x, y), a to y = {(x, b) : b ∈ B} (neboli y = Xx). Použitímaxiomu nahrazení na množinu A dostáváme množinu {Xa : a ∈ A}. Aplikováním axiomu sjednocenína tuto množinu dostáváme A×B.

B

A

Xa{Xa : a ∈ A} A×B

Cvičení 10. Dokaž existenci kartézského součinu bez použití axiomu nahrazení.

Třídy a dvojí pohled na zobrazení

V prvním díle jsme definovali zobrazení (neboli funkci) jako předpis, jak přeměnit jeden typ objektůna jiný. Toto lze formálně přepsat pomocí formule jako v axiomu nahrazení: Uvažme formuli ψ(x, y),která neobsahuje proměnné y0, y1, a navíc pro ni platí

(∀x, y0, y1)(

(ψ(x, y0) ∧ ψ(x, y1))⇒ (y0 = y1))

.

Pak taková formule určuje třídovou funkci fψ , kde zápisem fψ(x) rozumíme ten jediný prvek y,který splňuje ψ(x, y) (existuje-li). Takto popsaná funkce je formulí, nikoli množinou. Další formuleale mohou mluvit jen o množinách. Nelze tedy například napsat formuli, která by znamenala „exis-tuje zobrazeníÿ, což je například pro porovnávání mohutností množin celkem podstatný problém.Proto definujeme funkci coby množinu.Množinovou funkcí f :A → B, kde A, B jsou množiny, myslíme nějakou podmnožinu součinu

f ⊂ A×B, takovou, že pro každé a ∈ A existuje právě jedno b ∈ B splňující (a, b) ∈ f . I zde píšemef(a) = b.Jakmile máme takovou množinu f , můžeme pomocí ní napsat odpovídající formuli ψ(x, y) coby

(x, y) ∈ f . Takto jsme převedli množinové zobrazení f na třídové zobrazení fψ , problém je, žeobráceně to nemusí být možné provést. Pro lepší uchopení si objasníme pojem třídy.Vraťme se k Russelovu paradoxu. Ten vzešel z toho, že jsme s množinami začali zacházet způso-

bem, na který původně nebyly stavěny. Pokud bychom zůstali u množin coby „souhrnného označenípro nějaký typ bodů v rovině, časových okamžiků, lidí na Zeměkouli, . . .ÿ, tak bychom Russelůvparadox nedostali. Problém nastal, když jsme množiny opět prohlásili za matematické objekty azačali strkat množiny do množin. V takovém okamžiku se ukázalo, že za množinu nemůžeme pro-hlásit jen tak něco, nýbrž jen to, co sestrojíme pomocí axiomů. Občas se ale hodí používat opětmnožiny v jejich původním významu, tedy jen coby souhrnné označení pro nějaký typ objektů.Jenže slovo množina je již zabrané. Proto budeme souhrnnému označení pro nějaký typ množinříkat třída.Formálně je třída T vždy reprezentovaná nějakou formulí ϕ(x). Za prvky třídy T pak prohlásíme

přesně ty množiny x, pro které ϕ(x) platí.

17Jedno použití vytvoří množinu {a}, druhé množinu {a, b} a poslední žádanou množinu.32

Page 33: Matematický korespondenční seminář Milýpříteli! · těžiště trojúhelníka KNP. Poměr, v kterém těžiště dělí těžnici, je přitom dobře znám – kýžený výsledek

Korespondenční seminář, KAM MFF UK, Malostranské náměstí 25, 118 00 Praha 1

Příklad.

(i) Třída všech množin je reprezentovaná formulí x = x. Z Russelova paradoxu plyne, žetato třída neodpovídá žádné množině. Třídy, které neodpovídají žádné množině, budemenazývat vlastní.

(ii) Pro libovolnou množinu A můžeme popsat třídu se stejnými prvky formulí x ∈ A. Každámnožina je tedy i třídou. Axiom vydělení navíc říká, že kdykoli jsou všechny prvky třídyT obsaženy v jedné množině, je tato třída opět množinou. Proto si lze třídy představovatjako „příliš velké na to, aby byly množinamiÿ.

(iii) Třída všech jednoprvkových množin je reprezentovaná formulí (∃y)(x = {y}).

Cvičení 11. Dokaž, že třída všech jednoprvkových množin je vlastní.

V některých případech se třídy chovají oproti množinám benevolentněji, v některých stejně,v jiných zas striktněji. Stejně jako v množinách můžeme pro libovolné dvě třídy T , U reprezentovanéformulemi ϕ, ψ napsat

(i) T ∪ U (třída reprezentovaná formulí ϕ(x) ∨ ψ(x)),(ii) T ∩ U (třída reprezentovaná formulí ϕ(x) ∧ ψ(x)),(iii) T ⊂ U (neboli (∀x)(ϕ(x)⇒ ψ(x))).

Na rozdíl od množin ale

(i) vlastní třída nemůže být prvkem množiny ani třídy,(ii) třídy nelze kvantifikovat (tedy ptát se, zda existuje třída či zda něco platí pro každoutřídu). Ve výjimečných případech, kdy se tak děje (například ve schématech vydělení anahrazení), se nejedná o formuli, ale pouze o „metatvrzeníÿ, těmito nuancemi se všaknebudeme příliš zabývat.

Nyní je zřejmé, proč zobrazení určené formulí nazýváme třídovým. Platí ještě jedno metatvrzenídávající do souvislosti třídy a zobrazení.

Tvrzení. Třídové zobrazení lze zapsat množinově právě tehdy, když jeho definiční obor je mno-žina.

Důkaz. Pokud lze zobrazení napsat množinově, je jeho definiční obor množina z definice mno-žinového zobrazení. Naopak předpokládejme, že máme třídové zobrazení, jehož definiční obor jemnožina A. Z axiomu nahrazení je tak i jeho obor hodnot množina, označme ji B. Hledané množi-nové zobrazení dostaneme vydělením z kartézského součinu A×B. �

Příklad. Potence je třídová funkce, kterou nelze vyjádřit množinou, protože je definovaná navšech množinách.

Třídová a množinová zobrazení nebudeme důsledně rozlišovat, nicméně pokud to bude možné,budeme chápat zobrazení jako množinová.

Ordinální čísla – typy DUM

Podobně jako jsme zavedli funkci f coby množinu, můžeme zavést DUMu. DUM coby objekt v soběmusí mít uloženou nejenom nosnou množinu, ale i informaci o tom, jak se prvky nosné množinyporovnávají. Formálně proto definujeme DUMu jako dvojici (D,U), kde D je nosná množina aU ⊂ D × D je uspořádání na ní. Uspořádání U pak funguje tak, že značením d0 < d1 rozumíme(d0, d1) ∈ U . Aby (D,U) byla DUM, požadujeme po U vlastnosti dobrého uspořádání.Jak se ukázalo v minulém díle, není pro DUMu důležitá ani tak její nosná množina, jako spíš jen

její „typÿ. Prozatím si typ představujeme naivně jako „zapomenutí konkrétních prvkůÿ, ale dalekovhodnější bude jej definovat jako konkrétní množinu. To se standardně provede takto:

Definice. Mějme DUMu (D,U). Transfinitní rekurzí definujeme na D zobrazení f předpisemf(x) = {f(d) : d < x}, speciálně pro nulový prvek d0 ∈ D vyjde f(d0) = ∅. Nakonec definujeme

33

Page 34: Matematický korespondenční seminář Milýpříteli! · těžiště trojúhelníka KNP. Poměr, v kterém těžiště dělí těžnici, je přitom dobře znám – kýžený výsledek

Matematický korespondenční seminář 35. ročník (2015/2016), 2. komentáře

typ této DUMy jako typ(D,U) = {f(d) : d ∈ D}. Množiny, které lze získat jako typ(D,U) nějakéDUMy (D,U), nazveme ordinálními čísly (stručně ordinály).

Příklad. Pro libovolnou trojprvkovou DUMu (D,U) na nosné množině {d0, d1, d2}, kde d0 <d1 < d2, se chová definice typu takto:

f(d0) = ∅, f(d1) = {∅}, f(d2) ={

∅, {∅}}

, typ(D,U) ={

∅, {∅},{

∅, {∅}}

}

.

Ještě dodejme, v jaké formě jsme použili transfinitní rekurzi. Rekurzivní předpis je třídovézobrazení definované na všech množinových funkcích, jejichž definičním oborem je D←x pro nějakéx ∈ D. Výsledná funkce pak je opět množinová. Jelikož někdy neumíme předem omezit třídu všechmožných částečných množinových funkcí množinou, není tvrzení o existenci funkce dané transfinitnírekurzí tvrzením popsatelným formulí, ale metatvrzením, do kterého teprve můžeme dosazovatrůzné rekurzivní předpisy.

Cvičení 12. Projdi důkaz existence funkce dané transfinitní rekurzí (z minulého dílu) a rozmyslisi, které axiomy jsou potřeba pro tvrzení, že každá DUM má typ.

Nyní si uvědomíme, jak vypadají typy DUM. Každý z postupně přidávaných prvků f(x) ={f(d) : d < x} obsahuje všechny prvky f(d) pro d < x. A také naopak žádný z těchto prvkůf(d) prvek f(x) neobsahuje – to plyne z axiomu fundovanosti.18 Celkově dostáváme, že funkce fzobrazila nosnou množinu DUMy (D,U) na α = typ(D,U) takovým způsobem, že

(d0 < d1)⇔ (f(d0) ∈ f(d1)).

Pokud se budeme dívat na náležítko coby na znaménko uspořádání, vidíme, že f je rostoucí bijekcímezi DUMami (D,U) a (α,∈), platí tedy (D,U) ≃ (α,∈).Ano, používání náležítka k porovnávání zprvu vypadá jako nehorázná zhůvěřilost, k tomu přeci

vůbec nebylo stavěné. Na druhou stranu je náležítko to nejjednodušší, co v jazyce teorie množinmáme, a ukáže se být zcela postačujícím a dokonce praktickým. Na prvcích ordinálního čísla budeproto náležítko plnit roli menšítka <.Další věc, které si můžeme všimnout rovnou z definice funkce typ, je f(x) = typ(D←x, U).

Prvky ordinálního čísla jsou tedy opět ordinály – typy všech menších DUM. Tato skutečnost usnad-ňuje jejich porovnávání. Konkrétně pro ordinály α, β platí

(i) (α,∈) ≃ (β,∈) právě tehdy, když α = β,(ii) (α,∈) < (β,∈) právě tehdy, když α ∈ β,(iii) (α,∈) ≤ (β,∈) právě tehdy, když α ⊂ β.

Definice. Označme On třídu všech ordinálních čísel.

Později ukážeme, že třída On nemůže být množinou. Chová se však jako „největší ordinálÿ,konkrétně:

(i) (∀α ∈ On)(α ⊂ On).(ii) On je dobře uspořádaná náležítkem. Dokonce v tom smyslu, že kdykoli popíšeme neprázd-nou podtřídu T ⊂ On, tak tato podtřída má nejmenší prvek. Proč? Uvažme libovolnýordinál α ∈ T . Jedná-li se o nejmenší prvek T , jsme hotovi. V opačném případě je mno-žina α∩T neprázdná a najdeme nejmenší prvek tohoto průniku díky dobrému uspořádánína α.

(iii) Na On lze definovat třídová zobrazení pomocí transfinitní rekurze. Rekurzivní předpisv takovém případě musí být třídové zobrazení, které je definované na všech množinovýchfunkcích f s definičním oborem α ∈ On. Transfinitní rekurze pak určuje třídovou funkci Fdefinovanou na všech ordinálech. Formálně napsané Ti to možná zní děsivě, ale jak uvidíšna příkladu ωα na konci seriálu, samotné použití je docela přirozené.

18Případně to lze ukázat i bez něj volbou nejmenšího x, pro které se tato vlastnost porušila.

34

Page 35: Matematický korespondenční seminář Milýpříteli! · těžiště trojúhelníka KNP. Poměr, v kterém těžiště dělí těžnici, je přitom dobře znám – kýžený výsledek

Korespondenční seminář, KAM MFF UK, Malostranské náměstí 25, 118 00 Praha 1

Zatím jsme si ukázali, že každé ordinální číslo je množina všech menších ordinálů. Následujícítvrzení říká, že to platí i obráceně.

Tvrzení. Nechť X ⊂ On je množina taková, že kdykoli obsahuje ordinální číslo α, tak obsahujei všechna ordinální čísla β < α. Pak X je ordinál.

Důkaz. Z dobrého uspořádání třídy On (bod (ii)) plyne, že i X je dobře uspořádaná náležítkem.Navíc dle zadání splňuje pro každý prvek α ∈ X podmínku α = {β ∈ X : β < α}. To znamená, žefunkce f(α) = α splňuje rekurzivní předpis pro definici typu DUMy (X,∈), takže typ(X,∈) = X,čili X je ordinální číslo. �

Důsledek. Pro libovolnou množinu X ⊂ On je⋃

X ordinální číslo.

Důsledek. On je vlastní třída.

Důkaz. Kdyby On byla množina, tak by byla ordinálem. To by ale znamenalo On ∈ On, což jesporu s fundovaností.19 �

Cvičení 13. Ukaž, že množina X je ordinální číslo právě tehdy, když platí následující dvě pod-mínky.

(i) Pro každé dva různé prvky x, y ∈ X platí x ∈ y nebo y ∈ x.(ii) Každý prvek x ∈ X je podmnožinou X.

Definice. Ordinály dělíme na nulový, izolované a limitní podle toho, jaké prvky reprezentujív uspořádané třídě On. Konkrétně:

(i) Nulový ordinál je jen prázdná množina ∅.(ii) Izolovaný ordinál α je takový, který má největší prvek β ∈ α v uspořádání náležítkem.(iii) Limitní ordinály jsou ty, které nejsou nulové ani izolované.

Přirozená a reálná čísla

Nyní definujeme přirozená čísla coby množiny. Přirozená čísla jsou od toho, aby udávala velikostikonečných množin. Je proto vhodné, aby každé přirozené číslo mělo tolik prvků, kolik je ono samo.Tedy 0 bude prázdná množina, 1 bude jednoprvková množina, 2 bude dvouprvková, a tak dále.Prázdná množina je určena jednoznačně 0 = ∅, ale které prvky dát do dalších přirozených čísel?Musí se jednat o již dříve definované množiny, tak dává smysl, aby to byla přímo předchozí přirozenáčísla:

1 = {0}, 2 = {0, 1}, 3 = {0, 1, 2}, 4 = {0, 1, 2, 3}, . . .

Přesně takto se ale staví i ordinální čísla. Přirozená čísla jsou tedy jen počátečními ordinálnímičísly – těmi konečnými. Formálně popíšeme, které ordinály považujeme za konečné.

Definice. Konečné ordinální číslo (nebo též přirozené číslo) α je takové, které není limitní a anižádný menší ordinál není limitní. Symbolem ω značíme množinu všech konečných ordinálních čísel.

Z toho snadno plyne, že kdykoli je ordinál n konečný, jsou i všechny ordinály k < n konečné.Množina ω je tak současně ordinálem, a to nejmenším nekonečným – všechny menší jsou konečnéa ω nemůže obsahovat samu sebe.Musíme ale dokázat, že ω existuje (tedy že se nejedná o vlastní třídu). Zvolme libovolnou

množinu m z axiomu nekonečna a vydělme z ní množinu ω′ = {n ∈ m : n je konečný ordinál}.Dokážeme, že ω′ obsahuje všechny konečné ordinály, a je tak hledanou ω. Zvolme pro spor nejmenšíkonečný ordinál n 6∈ ω′. Nulový ordinál v ω′ leží, takže je n izolovaný a existuje jeho předchůdcen′ ∈ ω′. Jenže n = n′ ∪ {n′}, a tak n ∈ m z podmínky pro m. Proto i n ∈ ω′.

19Kdybychom nechtěli použít axiom fundovanosti (který se občas vypouští), mohli bychomargumentovat i tím, že pro DUMu nemůže nastat On < On.

35

Page 36: Matematický korespondenční seminář Milýpříteli! · těžiště trojúhelníka KNP. Poměr, v kterém těžiště dělí těžnici, je přitom dobře znám – kýžený výsledek

Matematický korespondenční seminář 35. ročník (2015/2016), 2. komentáře

Na ordinálech, a tudíž i na přirozených číslech, zavedeme operace sčítání a násobení pomocísčítání a násobení na DUMách. Pouze s tím rozdílem, že na výsledek aplikujeme zobrazení typ,abychom získali opět ordinální číslo, tedy α · β = typ

(

(α,∈) · (β,∈))

.

Cvičení. Rozmysli si, že součet i součin přirozených čísel je opět přirozené číslo.

Na základě přirozených čísel definujeme celá čísla. Množinou celých čísel Z bude ({0, 1} × ω) \{(1, 0)}. Celé číslo pak je vždy dvojice (z, n), kde n je přirozené číslo určující absolutní hodnotucelého čísla a z ∈ {0, 1} určuje jeho znaménko (0 znamená plus). Z množiny celých čísel vyjímámedvojici (1, 0), která by reprezentovala číslo „−0ÿ.Dále definujeme racionální čísla (zlomky) coby dvojice (a, b), kde a je celé číslo a b je nenulové

přirozené číslo nesoudělné s a. Tuto dvojici interpretujeme jako zlomek ab. Operace na celých a

racionálních číslech se definují klasicky, ale bylo by nudné to zde rozepisovat.

Poznámka. Při takto definovaném rozšiřování čísel je 42 coby přirozené číslo jiné než 42 cobycelé číslo, a to je jiné než 42 coby racionální číslo. Tento formalistický problém ale matematikynikdy příliš nevzrušoval a běžně jsou přirozená čísla chápána jako podmnožina celých a ta zasejako podmnožina racionálních, ačkoli tomu tak formálně vzato zcela není.

Předvedeme ještě elegantní definici nezáporných reálných čísel, kterou lze snadno rozšířit nadefinici všech reálných čísel. Bylo by možné je definovat pomocí nekonečného desetinného (či bi-nárního) rozvoje, avšak v takovém případě by bylo nesnadné například definovat násobení, prototo uděláme jinak.

Definice. Nechť Q+0 značí množinu nezáporných racionálních čísel. O množině R ⊂ Q+0 řekneme,

že to je nezáporné reálné číslo, pokud

(1) s každým prvkem obsahuje všechny menší, tedy (∀r ∈ R)(∀s ∈ Q+0 )(s < r ⇒ s ∈ R),

(2) nemá největší prvek,(3) R 6= Q

+0 .

Nezáporné reálné číslo tedy charakterizujeme výčtem všech racionálních čísel, která jsou menší.Racionální číslo q ∈ Q

+0 odpovídá reálnému číslu {r ∈ Q

+0 : r < q}. Operace v reálných číslech

definujeme následovně:

(i) pro nezáporná reálná čísla R, S definujeme R+ S = {s+ t : r ∈ R, s ∈ S},(ii) pro nezáporná reálná čísla R, S definujeme R · S = {r · s : r ∈ R, s ∈ S},(iii) pro nezáporná reálná čísla R, S definujeme R ≤ S jako R ⊂ S.

Množinu nezáporných reálných čísel značíme R+0 .

Poznámka. Tato konstrukce reálných čísel je nejbližší konstrukci pomocí Dedekindových řezů.Standardní konstrukce Dedekindovými řezy definuje rovnou všechna reálná čísla (nikoli jen nezá-porná). V takovém případě je třeba ještě zakázat prázdnou množinu a věnovat větší péči násobeníreálných čísel.

Cvičení 14. Dokaž, že množiny popsané v bodech (i), (ii), tedy součet a součin dvou nezápornýchčísel, odpovídají definici nezáporného reálného čísla.

Cvičení 15. Najdi nespočetnou množinu X ⊂ P(ω) tak, aby pro libovolné dva prvky x, y ∈ Xplatilo x ⊂ y nebo y ⊂ x.Z takto definovaných nezáporných reálných čísel plynou klíčové vlastnosti reálných čísel, které

popisují jejich vztah k číslům racionálním:

(i) Pro každá dvě různá nezáporná reálná čísla existuje racionální číslo, které leží mezi nimi.(hustota racionálních čísel)

(ii) Uvažme neprázdnou množinu čísel X ⊂ R+0 . Řekneme, že X je omezená číslem r ∈ R

+0 ,

pokud (∀x ∈ X)(x ≤ r). Pokud nějaké číslo r ∈ R+0 omezuje X, tak lze najít jedno

nejmenší možné číslo, které omezuje X (tímto číslem je⋃

X). (existence suprema)

36

Page 37: Matematický korespondenční seminář Milýpříteli! · těžiště trojúhelníka KNP. Poměr, v kterém těžiště dělí těžnici, je přitom dobře znám – kýžený výsledek

Korespondenční seminář, KAM MFF UK, Malostranské náměstí 25, 118 00 Praha 1

Nezáporná reálná čísla lze rozšířit na všechna reálná čísla obdobně jako přirozená čísla na celá.Vlastnost hustoty racionálních čísel i existence suprema v nich zůstane zachována.

Příklad. O množině X ⊂ R řekneme, že je diskrétní, pokud pro každé x ∈ X existuje otevřenýinterval20 I, který splňuje I ∩X = {x}. Platí, že každá diskrétní množina je spočetná.Důkaz. Uvažme x ∈ X a otevřený interval I = (a, b), který splňuje I ∩ X = {x}. Pak je možnénajít racionální číslo p uvnitř intervalu (a, x) a racionální číslo q uvnitř intervalu (x, b). Možnýchdvojic racionálních čísel (p, q) je jenom spočetně mnoho, protože |Q × Q| = ω. Na druhou stranulze zpětně z dvojice (p, q) rekonstruovat číslo x tím, že najdeme průnik intervalu (p, q) a množinyX. Proto je v množině X jenom spočetně mnoho čísel. �

Cvičení 16. Najdi funkci f :R×R→ ω takovou, že f(x, y) = f(y, z) nastává jedině pro x = y = z.

Nespočetný ordinál

Rozšíření přirozených čísel směrem k reálným byla odbočka demonstrující, že pomocí teorie množinlze stavět i známé matematické objekty. Nyní se vrátíme k výpravě do nekonečna a ještě dál asestrojíme nespočetné ordinální číslo.Nechť ω1 značí množinu všech spočetných ordinálních čísel. Určitě nemůže být ω1 spočetná –

to plyne ze stejného argumentu, z jakého On není množina a ω je nekonečná; musela by obsahovatsama sebe, což je ve sporu s fundovaností. Rovněž analogicky je ω1 nejmenší nespočetný ordinál.Nemusí ale být jasné, proč tato množina existuje, neboli proč není vlastní třídou.To plyne z axiomu nahrazení pro funkci typ. Začneme s množinou {ω}×P(ω×ω), čili množinou

všech dvojic (ω,U), kde U ⊂ ω × ω. V některých případech určuje U dobré uspořádání na ω,v takovém případě můžeme najít typ(ω,U). Množinu ω1 sestrojíme jako množinu těchto typů(existuje díky axiomu nahrazení), kterou nakonec ještě sjednotíme s ω (abychom tam dostali ikonečné ordinály).

Příklad. Neexistuje rostoucí funkce f :ω1 → R.

Důkaz. Pro spor předpokládejme takovou funkci f . Pak pro každé α ∈ ω1 najdeme racionálníčíslo g(α) v otevřeném intervalu (f(α), f(α + 1)). Funkce g je rostoucí, tedy prostá, a dostávámetak |ω1| ≤ |Q|, což je spor. �

Dále ω1 vykazuje řadu analogií s množinou přirozených čísel ω:

ω : Každý prvek n ∈ ω dělí ω na konečně mnoho menších a nekonečno větších prvků.ω1 : Každý prvek ω1 dělí ω1 na spočetně mnoho menších a nespočetně mnoho větších prvků.Proč? Menších prvků je spočetně, protože to jsou prvky spočetného ordinálu, a větších (ostatních)

prvků je nespočetně mnoho, protože ω1 sama je nespočetná.

ω : Každá množina X ⊂ ω je konečná nebo má stejný typ jako ω.ω1 : Každá množina X ⊂ ω1 je spočetná nebo má stejný typ jako ω1.Proč? Plyne z věty o porovnání typů z minulého dílu – buď typ(X) ∈ ω1, a tak je X spočetná,

nebo typ(X) = ω1.

ω : Množina X ⊂ ω je konečná právě tehdy, když ⋃X ∈ ω.ω1 : Množina X ⊂ ω1 je spočetná právě tehdy, když

X ∈ ω1.Proč? Když je X spočetná a každý prvek x ∈ X je spočetný ordinál, tak množina

X jespočetná, a tedy prvek ω1. Pro opačnou implikaci si stačí uvědomit X ⊂ (

X) + 1 ∈ ω1.

20Otevřený interval (a, b) je množina daná předpisem {x ∈ R : a < x < b} pro dané a, b ∈ R,a < b.

37

Page 38: Matematický korespondenční seminář Milýpříteli! · těžiště trojúhelníka KNP. Poměr, v kterém těžiště dělí těžnici, je přitom dobře znám – kýžený výsledek

Matematický korespondenční seminář 35. ročník (2015/2016), 2. komentáře

Aby to ale nevypadalo, že se ω1 chová úplně stejně jako přirozená čísla, nabízíme opět ilustračnípohádku.

Pohádka o nespočetném gamblerovi

Ve světě teorie množin, kde čas běží po ordinálních číslech, se jistý pan Loudal neustále hádal sesvou ženou. Sice před časem nalezl poklad, ale v teorii množin je času nekonečně (a ještě víc), atak už stihl skoro všechen majetek prosázet v loterii, za což ho Loudalka právě napomínala: „Mělbys s tím sázením skončit v konečným čase a čím dřív, tím líp. Nekonečně mincí se vyhrát nedá,to je dokázaný! Važ si tý jedný mince, co ti zbyla!ÿ „Ale na mne se jednou usměje štěstí a budememít nekonečně peněz,ÿ snil si Loudal svou.Navíc tou dobou do vesnice přivezli výherní automat. A nebyl to jen tak kdejaký hrací automat,

jaké možná znáte z obyčejného světa. Obyčejné automaty většinou sežerou minci a nic z toho.Kdykoli tento automat obdržel minci, vypadly z něho dvě nové mince stejné hodnoty. Od něčehotak lákavého Loudalka Loudala zadržet nedokázala. Hodil do něj mincim0, vypadly dvě další mincem1, m2, hodil do něj minci m1, vypadly další dvě mince m3, m4, vhodil mince m2, vypadly mincem5, m6. A takto Loudal pokračoval až do ω, kdy do automatu naházel všechny mince a žádná munezbyla.To nebylo to pravé ořechové, pomyslel si Loudal s jednou kapsou prázdnou a druhou vysypanou.

Loudalka ho přivítala zčásti vítězoslavně, zčásti pekelně naštvaně, ale unavený Loudal ji ani mocneposlouchal a šel spát. V noci dostal nápad. Hned vyskočil z postele a šel ještě za tmy na ryby,aby si vydělal nějakou tu kačku do automatu. Jakmile se mu to povedlo, vydělanou minci m0 hodildo automatu a vypadly opět mince m1, m2. Minci m1 si nechal a do automatu hodil až druhouvydělanou minci m2. Minci m3 si opět nechal a hodil do automatu m4, . . . Takto měl v čase ω odvhození první mince nekonečně mnoho mincí. „No vidíš,ÿ povídá ráno Loudalce, „ já věděl, že seto musí podařit.ÿDalšího dne pak automat vyměnili za nový. Nový automat, kdykoli dostane minci, vyhodí

zpátky ω mincí. „No teda,ÿ pomyslí si Loudal, „teď někomu stačí hodit do automatu jednu minci abude stejně bohatý jako já, jaká nespravedlnost!ÿA hned se mu v hlavě zrodil plán, že kdyby naautomatu hrál až do kroku ω1, mohl by vyhrátnespočetně mnoho mincí. Marně ho Loudalka na-pomínala, že v tom bude zase nějaká čertovina,akorát se ohradil: „Že dostanu nekonečně mnohomincí, tos mi taky nevěřila, a jakej jsem byl!ÿLoudal vzal svých ω mincí a začal je házet doautomatu. Opět se pokoušel si vybírat mince ši-kovně do zásoby, ale ať se snažil, jak se snažil,ještě dříve, než se dostal do kroku ω1, byl opětbez peněz. Nespočetně mnoho mincí pomocí ta-kového automatu totiž získat nelze. O tom, jakmu pak Loudalka vyhubovala, raději taktně po-mlčíme.A proč z automatu nelze získat nespočetně

mnoho mincí? Dokážeme, že už v některém spo-četném kroku Loudal nebude mít do automatuco hodit. Můžeme tedy pro jednoduchost před-pokládat, že se Loudal nerozhodl žádnou minciušetřit do kroku ω1 (když chceme dokázat, že sedo toho kroku stejně nedostane).Označme X0 množinu všech kroků, při nichž

vhodil do automatu některou z mincí, které měl

38

Page 39: Matematický korespondenční seminář Milýpříteli! · těžiště trojúhelníka KNP. Poměr, v kterém těžiště dělí těžnici, je přitom dobře znám – kýžený výsledek

Korespondenční seminář, KAM MFF UK, Malostranské náměstí 25, 118 00 Praha 1

na začátku. To je spočetná podmnožina ω1, tedy existuje ordinál α0 ∈ ω1, který je ostře větší nežvšechny prvky X0 (například můžeme zvolit α0 =

X0 + 1). Před α0 proběhlo spočetně mnohokroků a v každém z automatu vypadlo spočetně mnoho mincí. Celkem to je opět spočetně mnohovypadlých mincí. Označme X1 množinu všech kroků, během kterých Loudal hodil do automatuněkterou z nich. Opět najdeme α1, které je větší než všechny prvky X1, a opakováním postupusestrojíme Xn a αn pro každé přirozené číslo n. Nakonec definujeme spočetný ordinál α =

⋃{αn :n ∈ ω}. V kroku α už Loudalovi žádná mince nezbyla. Každou minci, kterou vydělal před krokemα, totiž vydělal před některým krokem αn. Jenže to znamená, že ji hodil do automatu před krokemαn+1.

ω1

0 α0 α1 α2 α3

α

X0

X1X2

Poznámka. Nepohádková verze tohoto tvrzení se nazývá Pressing down lemma a zní: „Nechťf je funkce ω1 → ω1. Předpokládejme, že pro všechny nenulové α ∈ ω1 platí f(α) < α. Pak senemůže stát, že by všechny množiny Xβ = {α ∈ ω1 : f(α) = β} byly spočetné.ÿ Jak toto tvrzenísouvisí s pohádkou? Z Pressing down lemmatu lze přímočaře dokázat nemožnost výhry ω1 mincí anaopak.Nejprve ukážeme, že kdyby se Loudal dostal až do kroku ω1, nemůže platit Pressing down lemma.

V každém kroku α si Loudal musí vzít minci z některého předchozího kroku, označíme jej f(α).Množina Xβ pro takto definovanou f pak obsahuje všechny kroky, ve kterých hodil do automatuněkterou minci vydělanou v kroku β. Je tedy spočetná, protože vydělaných mincí z každého krokuje spočetně.Nyní naopak mějme funkci f , pro kterou jsou všechnaXβ spočetná, a ukažme, že se Loudal může

dostat do kroku ω1. V kroku α vezme Loudal minci z kroku f(α). Navíc mějme zafixované očíslovánímnožiny Xf(α) a Loudal si vezme tolikátou minci z kroku f(α), kolikátý je prvek α ∈ Xf(α) v tomtoočíslování. Tím nebude potřebovat žádnou minci hodit do automatu vícekrát a vždy bude mít minci,kterou do automatu hodí. Dostane se tedy až do kroku ω1.

A ještě dál

Postup pro sestrojení ω1 coby množiny všech spočetných ordinálů lze zobecnit a definovat ωα proobecný ordinál α rekurzivním předpisem:

(i) ω0 = ω,(ii) ωα+1 je množina všech ordinálů β splňujících |β| ≤ |ωα|,(iii) ωα =

⋃{ωα′ : α′ < α} pro α limitní.Takto definované ordinály ωα mají všechny různou mohutnost, což dává odpověď na otázku,

kolik je různě velkých nekonečen. Různě velká nekonečna tvoří vlastní třídu, tedy je jich tolik, žese ani nevejdou do množiny.

Příklad. Pokud bychom chtěli sestrojit hodně velké nespočetné ordinální číslo (resp. DUMu),můžeme uvážit ordinály

ω0, ωω0 , ωωω0, ωωωω0

, . . .

a nakonec vzít jejich sjednocení.

39

Page 40: Matematický korespondenční seminář Milýpříteli! · těžiště trojúhelníka KNP. Poměr, v kterém těžiště dělí těžnici, je přitom dobře znám – kýžený výsledek

Matematický korespondenční seminář 35. ročník (2015/2016), 2. komentáře

Čokoládová výzva

Za následující čokoládovou úlohu na řešitele čeká geometrická nekonečná posloupnost čokolád.Ten, kdo ji vyřeší nejrychleji, dostane čokoládu. Ten, kdo ji vyřeší po něm, dostane půlku čokolády,další čtvrtku čokolády, atd. Tak s chutí do řešení! Úloha se týká toho, jak lze mohutnost ωncharakterizovat i bez použití rekurze či dobrých uspořádání.

Úloha. Nechť α je ordinál a n přirozené číslo. Označme [α]n množinu všech n-prvkových pod-množin α a [α]<ω množinu všech konečných podmnožin α. O zobrazení f : [α]n → [α]<ω řekneme,že je čokoládové, pokud existuje (n+ 1)-prvková množina X ⊂ α taková, že

(∀x ∈ X)(x 6∈ f(X \ {x})).

Dokaž, že následující dvě tvrzení jsou ekvivalentní.

(i) Všechna zobrazení f : [α]n → [α]<ω jsou čokoládová.(ii) α ≥ ωn.

Minulou čokoládovou výzvu vyhrál Danil Koževnikov s následující konstrukcí: Pro DUMy A,B definujeme mocninu AB jako množinu všech funkcí B → A, které jsou pouze na konečně mnohamístech nenulové. Dvě takové funkce f , g porovnáme tak, že se podíváme na nejvyšší prvek b ∈ B,pro který f(b) 6= g(b), a hodnota v tomto bodě rozhodne, která funkce je větší. Danil takto vymyslelmocnění DUM, které se běžně používá, a s přehledem by vyhrál už jen s DUMou ω(ω

ω).On však šel dál a zbytek byl mírně zamlžený, tak jej popíšeme pomocí ordinálních čísel a

standardní notace εα. Stejně jako u sčítání a násobení rozšíříme mocnění na ordinální čísla. Paktransfinitní rekurzí definujeme εα předpisem:

(i) ε0 = ω ∪ ωω ∪ ω(ωω) ∪ . . .,(ii) εα+1 = εα ∪ εεαα ∪ ε(ε

εαα)

α ∪ . . .,(iii) εα =

⋃{εβ : β < α} pro α limitní.DUMou, kterou Danil vsadil, je ε(ωω). Ještě poznamenejme, že v zadání předchozí čokoládové úlohybylo cílem popsat co možná největší spočetnou DUMu, takže i Danilova DUM je spočetná. Oprotitomu DUM popsaná v předchozí kapitole je nespočetná vzhledem k tomu, že už ω1 je nespočetná.

40

Page 41: Matematický korespondenční seminář Milýpříteli! · těžiště trojúhelníka KNP. Poměr, v kterém těžiště dělí těžnici, je přitom dobře znám – kýžený výsledek

Korespondenční seminář, KAM MFF UK, Malostranské náměstí 25, 118 00 Praha 1

Seznam symbolů a pojmů

Na této stránce jsou stručně uvedeny všechny důležité pojmy druhého dílu seriálu. U každéhopojmu z tohoto dílu je uvedeno, na které stránce byl definován.

str 27. Logické spojky ¬, ∧, ∨, ⇒, ⇔str 27. Kvantifikátory ∀, ∃str 27. Náležítko ∈str 27. Formule – výroky ve formálním jazycestr 28. Zkratky 6∈ (neleží), 6= (nerovná se), ⊂ (je podmnožinou)str 28. {x : . . .} – množina daná předpisemstr 28. {x0, . . . , xn} – množina daná výčtem prvkůstr 28. x ∩ y – průnik množin x a ystr 28. x ∪ y – sjednocení množin x a ystr 28. x \ y – množinový rozdíl x minus ystr 28. P(x) – potence množiny xstr 29. ∅ – prázdná množinastr 30.

X – sjednocení všech prvků množiny Xstr 31. (a, b) – uspořádaná dvojicestr 32. A×B – kartézský součinstr 32. Třídová funkcestr 32. Množinová funkcestr 32. Třída – souhrnnné označení pro všechny množiny s nějakou vlastnostístr 33. Vlastní třída – třída, která není množinoustr 33. DUM coby dvojice (D,U)str 33. typ(D,U) čili ordinální číslo (stručně ordinál) udávající typ DUMy (D,U)str 34. On – třída všech ordinálůstr 35. Ordinál nulový, izolovaný, limitnístr 35. Přirozené číslo neboli konečné ordinální číslostr 35. ω – první nekonečný ordinálstr 35. α+ β, α · β pro α, β ∈ On – operace na ordinálechstr 36. R+0 neboli množina všech nezáporných reálných číselstr 37. ω1 – první nespočetný ordinálstr 39. ωα – ordinální číslo α-té nekonečné mohutnosti

Důležité pojmy z minulého dílu: zobrazení (funkce), spočetno, nespočetno, porovnávání mohutností|A| ≤ |B|, |A| = |B|, DUM (dobře uspořádaná množina), operace na DUMách A + B, A · B,porovnávání DUM A < B, A ≃ B.

41

Page 42: Matematický korespondenční seminář Milýpříteli! · těžiště trojúhelníka KNP. Poměr, v kterém těžiště dělí těžnici, je přitom dobře znám – kýžený výsledek

Matematický korespondenční seminář 35. ročník (2015/2016), 2. komentáře

Návody

1. (∃y)(

(

¬(y ∈ x))

∧ (∀z)(

z ∈ x′ ⇔ (z = y ∨ z ∈ x))

)

.

2. Zvol z = {x}.3. a \ b = {z ∈ a : z 6∈ b}. Stačí tedy volit ϕ(z) jako z 6∈ b.4. (∀x)(∃y)(∀z)

(

z ∈ y ⇔ (∀u)(u ∈ z ⇒ u ∈ x))

.

5. (i) Existence množiny {a} je ve sporu s axiomem fundovanosti. (ii) Stejný argument s {a, b}.6. (Zatím není rozepsané kompletně)

(∃m)(∃e)(

(∀z)(

z 6∈ e) ∧ (e ∈ m) ∧ (∀x)(

x ∈ m⇒ (∃y ∈ m)(

∀z)(

(z ∈ y)⇔ (z ∈ x ∨ z = x))

)

)

.

7. (i) Axiom nekonečna vyžaduje existenci množiny, jejímž prvkem je prázdná množina. Tedyprázdná množina sama o sobě musí existovat. (ii) ψ(x, y) definujeme jako ϕ(x) ∧ (x = y). (iii)P(P(∅)) je dvouprvková množina {∅, {∅}}. Pak definujeme ψ(x, y) jako (x = ∅ ∧ y = s) ∨ (x ={∅} ∧ y = t) a dostaneme množinu {s, t}.8. Postupné kroky:

(i) ∅ ∈ P(∅), ∅ ∈ P(P(∅)),(ii) {∅} ∈ P(P(∅)), {∅} ∈ P(P(P(∅))) z (i),(iii) {∅, {∅}} ∈ P(P(P(∅))) z (i) a (ii),(iv) (∅, {∅}) = {{∅}, {∅, {∅}}} ∈ P(P(P(P(∅)))) z (ii) a (iii).

9. Dvojice (a, b) je jednoprvková množina právě tehdy, když a = b. V takovém případě je {a}jednoznačně určený prvek této množiny. Pokud je (a, b) dvouprvková, obsahuje jednu jednoprvkovoua jednu dvouprvkovou množinu. Prvek té jednoprvkové musí být a. Současně a musí být jednímz prvků dvouprvkové množiny, ten druhý pak musí být b.

10. Je třeba si vzpomenout na definici uspořádané dvojice. Součin A×B pak lze dostat vydělenímz množiny P(P(A ∪B)).11. Sporem: Kdyby to byla množina, tak z axiomu sjednocení existuje množina všech množin.Alternativa: Nechť množina A obsahuje všechny jednoprvkové množiny. Pak obsahuje i množinu{A}, což je sporu s fundovaností.12. To, že je rekurzivní předpis jednoznačně definován pro každou částečnou funkci, zaručujíaxiom nahrazení a extensionality. Pro využívání vlastnosti dobrého uspořádání vždy potřebujemeaxiom vydělení: Existuje prvek, který splňuje podmínku, vydělíme tedy množinu všech prvků, kteréji splňují, a v té najdeme nejmenší. Dále rozlišíme tři možnosti z důkazu v minulém dílu:

(i) Když x je nulový, stačí axiom existence.(ii) Když x je izolovaný, používáme axiomy dvojice a sjednocení pro rozšíření f o jeden prvek.Alternativně by stačilo popsat toto rozšířené zobrazení třídově a použít axiom nahrazenístejně jako v bodě (iii).

(iii) Když x je limitní, umíme (jednoznačně) popsat hodnotu pro všechna y < x. Stačí tedypoužít axiom nahrazení k tomu, že třídové zobrazení, jehož definiční obor je množina, lzereprezentovat množinou.

13. Dopředná implikace byla dokázaná v textu. Předpokládejme (i) a (ii) a dokažme, že X je or-dinál. Z (i) vyplyne přímočarou aplikací axiomu fundovanosti (zde se bez jeho využití neobejdeme),že X je dobře uspořádaná náležítkem. Důkaz se dokončí postupem z důkazu předchozího tvrzení.

14. (i) Když x < r + s, tak x = r′ + s, kde r′ = x − s < r. Z toho snadno odvodíme potřebnévlastnosti. (ii) Analogicky k (i).

15. Přejmenuj prvky nezáporných reálných čísel (coby množin) pomocí bijekce mezi Q+0 a ω.

16. Uvaž bijekci g:Q → ω. Pak lze volit f(x, x) = 0. Pro x < y pak f(x, y) = 2g(q) + 1,f(y, x) = 2g(q) + 2, kde q leží v intervalu (x, y).

42

Page 43: Matematický korespondenční seminář Milýpříteli! · těžiště trojúhelníka KNP. Poměr, v kterém těžiště dělí těžnici, je přitom dobře znám – kýžený výsledek

2. podzimní série – PoměryVýsledková listina

1.–6. Filip Bialas 3 GOpatovPH – – – 5 5 5 5 5 25 25,001.–6. František Couf 3 EKO GPraha 3 – 3 5 5 5 5 5 25 25,001.–6. Jakub Löwit 4 GČeskoliPH 3 3 3 5 5 5 5 5 25 25,001.–6. Jáchym Solecký 3 PORG PH 3 3 3 5 5 5 5 5 25 25,001.–6. Lucien Šíma 4 PORG PH 3 3 – 5 5 5 5 5 25 25,001.–6. Pavel Turek 3 GTomkovaOL 3 3 3 5 5 5 5 5 25 25,007. Danil Koževnikov 2 GKepleraPH 3 – – 5 5 5 5 4 24 + i 24,50

8.–9. Eleonora Krůtová 1 GJarošeBO 3 3 3 5 5 0 5 0 21 23,428.–9. Minh Duc Pham 1 NPorg 3 3 3 5 5 2 5 – 21 23,4210. Lucia Krajčoviechová 0 GJHroncaBA 3 3 3 5 5 – – – 19 23,1111. Lenka Kopfová 1 MendelG OP 3 2 3 5 5 5 – – 21 22,9612. Filip Čermák 2 MendelG OP 3 3 3 5 5 0 5 – 21 22,77

13.–17. Daniel Borák 1 GŠpitálsPH 3 3 3 5 5 0 – – 19 22,4313.–17. Viktória Brezinová 1 GAlejKošic 3 3 3 5 5 – – – 19 22,4313.–17. Matthew Dupraz 1 NPorg 3 3 3 5 5 0 – 1 19 22,4313.–17. Ondřej Krabec 1 G KomHavíř 3 3 3 5 5 – – – 19 22,4313.–17. Samuel Krajči 1 GAlejKošic 3 3 3 5 5 – – – 19 22,43

18. Pavel Hudec 2 GJarkovPH 3 2 3 2 5 5 5 – 21 22,3919. Vendula Kuchyňová 2 GMLerchaBO 3 3 3 – 5 4 5 – 20 + i 22,3420. Hedvika Ranošová 2 GBudějovPH 3 3 3 5 5 – 5 – 21 22,2321. Victoria María Nájares Romero 2 GZborovPH 3 3 3 5 3 5 5 – 21 22,18

22.–23. Alexandr Jankov 2 MatičníGOS 3 3 3 5 5 4 2 0 20 22,1722.–23. Ondřej Motlíček 3 G Šumperk 3 3 3 5 5 0 5 0 21 22,17

24. Radek Olšák 1 GMensaPH 3 3 3 5 5 – – – 19 21,9525.–26. Pavel Havlín 1 NPorg 3 3 3 4 5 – – – 18 21,8825.–26. Lucie Kundratová 1 G TGM Zlín 3 3 3 – – 0 5 4 18 21,88

27. Petr Gebauer 2 G Mělník 3 2 – 5 5 5 – – 20 21,6328. Václav Steinhauser 2 G Dačice 3 3 3 5 5 5 – 3 21 21,57

29.–32. Ondřej Buček 2 GJarošeBO 3 3 3 5 5 0 – – 19 21,5229.–32. Matúš Komora 2 GLettMart 3 3 3 5 5 0 1 – 19 21,5229.–32. Vojtěch Lengál 2 GZborovPH 3 3 3 5 5 – – 1 19 21,5229.–32. Lenka Vincenová 2 GTomkovaOL 3 3 3 5 5 – – – 19 21,52

33. Filip Matějka 2 GZborovPH 3 3 3 5 5 – – – 19 21,4934. Veronika Hladíková 3 GMikul23PL 3 3 3 5 5 5 – – 21 21,4835. Matěj Doležálek 1 G Humpolec 3 3 3 – – 4 – 4 17− i 21,1336. Kamila Kyzlíková 2 GZborovPH 3 3 3 5 5 0 – – 19 21,0837. David Królikowski 2 G Karviná 3 3 3 4 5 0 1 0 18 20,8438. Tomáš Konečný 3 GJirsíkaČB 3 3 3 5 5 – 5 – 21 + i 20,7339. Adam Mendl 0 GCoubTábor 3 3 3 – 5 0 – 2 16 20,7040. Matěj Kraft 1 GMikul23PL 3 3 3 5 2 – 1 – 16 20,67

43

Page 44: Matematický korespondenční seminář Milýpříteli! · těžiště trojúhelníka KNP. Poměr, v kterém těžiště dělí těžnici, je přitom dobře znám – kýžený výsledek

Matematický korespondenční seminář 35. ročník (2015/2016), 2. komentáře

41.–42. Jan Petr 3 GKepleraPH 3 3 3 5 5 – – – 19 20,6341.–42. Daniele Venier 3 LSciARoiti 3 3 3 5 5 – – – 19 20,63

43. Denisa Chytilová 2 GJŠkodyPŘ 3 3 3 4 3 – 5 – 18 20,3844. Vojtěch Lanz 2 GZborovPH 3 3 3 5 5 1 – 1 19 20,3645. Richard Hladík 3 GaOA MarLáz 3 3 3 5 5 – – – 19 20,2746. Ondřej Svoboda 3 GJarošeBO 3 3 3 5 5 – – – 19 20,22

47.–49. Martina Kalašová 1 GJHroncaBA 3 3 3 – 5 – 1 – 15 20,0047.–49. Erik Kočandrle 1 GMikul23PL 3 3 3 5 – – 1 – 15 20,0047.–49. Jana Pallová 1 3 3 3 3 3 – 1 – 15 20,00

50. Daniel Kopf 4 SlezkéG OP 3 3 3 5 5 5 – – 21 19,9751. Adéla Kostelecká 4 GLesníZlín 3 3 3 5 5 2 5 – 21 19,9152. Tomáš Domes 3 MendelG OP 3 3 3 5 5 – – – 19 19,6153. Tomáš Čelko 2 GPBystrica 3 3 – 5 3 0 – 2 16 19,37

54.–56. Jakub Domes 1 MendelG OP 3 3 3 – 5 – – – 14 19,2854.–56. Tomáš Drobil 1 G Dačice 3 3 3 3 2 – – – 14 19,2854.–56. Monika Machalová 1 GJHroncaBA 3 3 3 – 5 – – – 14 19,28

57. Martin Bakoš 2 GPBystrica 3 – – 5 5 0 – 3 16− i 19,1758.–59. Kateřina Nová 3 G Vimperk 3 3 3 5 5 – – – 19 19,1458.–59. Kristína Szabová 2 GVarŽilina 3 – 3 5 5 – – – 16 19,14

60. Marian Poljak 4 GJŠkodyPŘ 3 3 – 5 5 – 5 – 21 19,1361.–62. Samuel Baran 2 GRaymanaPV 3 3 3 – 5 – 1 – 15 18,5961.–62. Marie Dohnalová 3 GNadKavaPH 3 3 3 5 4 – – – 18 18,59

63. Zuzana Svobodová 4 G FrýdlNOs 3 – 3 5 5 5 – – 21 18,4964. Alžběta Neubauerová 2 GNadKavaPH 3 3 3 – 5 – 1 – 15 18,3265. Peter Súkeník 4 GOkrŽilina 3 3 3 4 5 – – – 18 18,00

66.–73. Klára Cihlářová 2 G Klatovy 3 3 3 5 – – – 0 14 17,7766.–73. Ondřej Dušek 2 PORG PH 3 3 3 – 5 – – – 14 17,7766.–73. Petr Ježek 2 GBNěmcovHK 3 3 3 – 5 – – 0 14 17,7766.–73. Adéla Seidelmannová 2 VOŠRychnovKn 3 3 3 5 – – – 0 14 17,7766.–73. Martin Spišák 2 GAlejKošic 3 3 3 0 5 – – 0 14 17,7766.–73. Martin Strnad 2 G Dobříš 3 3 3 – 5 – – – 14 17,7766.–73. Anna Šírová 2 GJilemnice 3 3 3 5 – – – – 14 17,7766.–73. Dominika Zumrová 2 SPŠ PansPH 3 3 3 5 – 0 – – 14 17,7774.–76. Kateřina Charvátová 1 GBNěmcovHK 3 3 3 – 2 – 1 – 12 17,7074.–76. Šárka Míchalová 1 G Kralupy 3 3 1 – 5 – – 0 12 17,7074.–76. Zdeněk Vostřel 1 3 3 3 – 2 – 1 – 12 17,70

77. Marco Souza de Joode 0 3 3 3 – – – 1 0 10 17,3678. Marek Pospíšil 3 GJatečníÚL 3 3 3 – 5 – 1 0 15 17,2779. Michal Töpfer 3 GJPekařeMB 3 3 3 – 5 2 – 0 16 17,0580. Anh Minh Tran 4 GJarošeBO 3 3 3 5 3 – – – 17 17,00

81.–83. David Krajíček 2 BG Ostrava 3 2 3 – 5 – – – 13 16,9081.–83. Veronika Roubínová 2 G Kadaň 3 2 3 – 5 0 – – 13 16,9081.–83. Tereza Vlčková 2 GJarošeBO 3 2 3 – 5 – – – 13 16,90

84. Jaroslav Paidar 2 SPŠMasarLI 3 3 1 5 2 – – – 14− i 16,8885.–86. Ondřej Bursa 1 GTNovákBO 3 3 0 – 5 – – 0 11 16,8385.–86. Michal Chudoba 1 GLitoměřPH 3 3 3 – 2 – – – 11 16,83

87. Jaromír Mielec 3 GVolgogrOS 3 3 3 5 5 – – – 19− i 16,7388. Jan Šorm 4 GJarošeBO 3 3 3 5 5 4 – – 20 16,60

89.–91. Jan Hrůza 3 G Kadaň 3 3 3 – 5 0 – – 14 16,3689.–91. Martin Pašen 3 GRaymanaPV 3 3 3 – 5 – 0 0 14 16,3689.–91. Filip Strakoš 3 G TGM Zlín 3 3 3 – – – 5 – 14 16,3692.–93. Slavomír Hanzely 4 GRaymanaPV 3 3 3 – 5 2 – – 16 16,00

44

Page 45: Matematický korespondenční seminář Milýpříteli! · těžiště trojúhelníka KNP. Poměr, v kterém těžiště dělí těžnici, je přitom dobře znám – kýžený výsledek

Korespondenční seminář, KAM MFF UK, Malostranské náměstí 25, 118 00 Praha 1

92.–93. Matej Kvorka 2 GŠkolDubni 3 3 3 3 – 0 – – 12 16,0094.–98. Anna Jandová 1 G Leg PB 3 3 3 – – – 1 – 10 15,8994.–98. Jaroslav Konečný 1 G Čelákov 3 3 3 – 1 – 0 – 10 15,8994.–98. Jaroslav Kortus 1 GMikul23PL 3 3 3 – 1 – – – 10 15,8994.–98. Timur Sibgatullin 1 PČGKarVary 3 3 3 – – – 1 0 10 15,8994.–98. Andrej Židek 1 GJKTyla HK 3 3 3 – – – 1 0 10 15,89

99. Daniel Pišťák 4 GZborovPH 3 – – 2 5 5 5 – 20 15,87100. Vladimír Lukačko 2 GVarŽilina 3 2 3 4 – – – – 12 15,60101. Filip Oplt 3 GBudějovPH 3 3 3 – – – 1 3 13 15,43102. Aleš Krčil 3 G Humpolec 3 3 3 5 – 0 – 0 14 15,13103. Nikola Kalábová 2 FSG Pirna 3 2 0 0 5 – 1 0 11 15,05

104.–111. Alexander Csizmár 1 GMikul23PL 3 3 3 – – – – – 9 14,89104.–111. Pavel Čácha 1 GMikul23PL 3 3 3 – – – – – 9 14,89104.–111. Josef Král 1 MendelG OP 3 3 3 – – – – – 9 14,89104.–111. Anna Mírková 1 G LPika PL 3 3 3 – – – – – 9 14,89104.–111. Dominika Mokroszová 1 G FrýdČTěš 3 3 3 – – – – – 9 14,89104.–111. Michal Poft 1 G Teplice 3 2 3 – – 0 1 0 9 14,89104.–111. Antonín Štrpka 1 G Šumperk 3 3 3 – – – – 0 9 14,89104.–111. Linh Giang Tran 1 GMikul23PL 3 3 3 – – – – – 9 14,89112.–113. Petra Šnoblová 2 GJatečníÚL 3 3 3 – – – 1 – 10 14,05112.–113. Evžen Wybitul 2 3 3 3 0 – – 1 0 10 14,05114.–116. Kristýna Lhoťanová 1 G RožnRadh 3 3 1 – – 0 1 0 8 13,81114.–116. Lubomír Smrček 1 3 – – 0 4 – 1 0 8 13,81114.–116. Kamila Ženatá 1 GNeumannŽR 3 – 3 – 1 – 1 – 8 13,81

117. Nodari Gogatishvili 2 GZborovPH 3 3 3 – – 0 1 – 10 13,57118. Filip Chudoba 2 PORG PH 3 3 3 – 1 – – 0 10 13,54119. Johana Dvořáková 1 G Trutnov 3 3 2 – – – – – 8− i 13,51

120.–122. Matěj Coufal 3 G HavlBrod 3 3 3 – 1 – 1 0 11 13,47120.–122. Jiří Češka 3 CMGProstěj 3 3 3 – 2 0 – 0 11 13,47120.–122. Jan Klásek 3 SlovanG OL 3 3 3 – 1 – 1 – 11 13,47

123. Adam Doubrava 0 GMasarykKM 3 3 – – – – 1 – 7 13,34124. Zuzana Tréglová 3 G Žatec 3 1 3 – 5 – – – 12 13,06

125.–126. Martin Hubata 0 GMikul23PL 3 3 – – – – – – 6 13,03125.–126. Anna Musilová 0 PORG PH 3 3 – – – – – – 6 13,03127.–129. Jáchym Bareš 2 GTomkovaOL 3 3 3 – – – – – 9 13,00127.–129. Daniel Ridzoň 2 GKepleraPH 3 3 3 – – – – – 9 13,00127.–129. John Richard Ritter 2 G MasNámTŘ 3 3 3 – – – – – 9 13,00

130. Vít Kalisz 4 FSG Pirna 3 3 2 2 5 0 – – 15 12,87131.–135. Šimon Chvátil 1 GBNěmcovHK 3 3 1 0 – – – – 7 12,64131.–135. Alžběta Manová 1 G UherBrod 3 3 – – – – 1 – 7 12,64131.–135. Filip Müller 1 GMikul23PL 3 3 1 – – – – – 7 12,64131.–135. Karel Müller 1 GMikul23PL 3 3 1 – – – – – 7 12,64131.–135. Petr Zahradník 1 GŠmejkalÚL 3 3 1 – – – – – 7 12,64

136. Marek Malý 3 G Neratov 3 3 – – 5 – – – 11 12,57137.–139. Barbora Lišková 3 GJPekařeMB 3 3 3 – – – 1 – 10 12,45137.–139. Jana Menšíková 3 3 3 3 – – – 1 – 10 12,45137.–139. David Neugebauer 3 SlezkéG OP 3 3 – – – 0 – 4 10 12,45

140. Martina Šmehylová 2 GHlinŽilina 3 3 2 – – – – – 8 11,89141.–145. Tomáš Hampl 3 GDukelBR 3 3 3 – – – – 0 9 11,39141.–145. Samuel Hrubý 3 GFraŽilina 3 3 2 – – – 1 0 9 11,39141.–145. Vojtěch Jílek 3 VOŠKutHora 3 3 3 – – – – 0 9 11,39141.–145. Oldřich Kos 3 GKepleraPH 3 3 3 – – – – – 9 11,39

45

Page 46: Matematický korespondenční seminář Milýpříteli! · těžiště trojúhelníka KNP. Poměr, v kterém těžiště dělí těžnici, je přitom dobře znám – kýžený výsledek

Matematický korespondenční seminář 35. ročník (2015/2016), 2. komentáře

141.–145. Dan Raffl 3 GVoděraPH 3 3 3 – – – – 0 9 11,39146.–151. Denisa Jandová 1 GMikul23PL 3 3 – – – – – – 6 11,38146.–151. Tereza Lukášová 1 GMikul23PL 3 3 – – – – – – 6 11,38146.–151. Ondřej Meduna 1 GMikul23PL 3 3 – – – – – – 6 11,38146.–151. Magdaléna Sejkorová 1 GUKlafárŽR 3 3 – – – – – – 6 11,38146.–151. Monika Suchá 1 GMikul23PL 3 3 – – – – – – 6 11,38146.–151. Martin Zimen 1 GJMasar JI 3 3 – – – – – – 6 11,38

152. Valerie Skopalová 4 G VysMýto 3 3 1 3 1 0 1 0 11 11,00153. Anh Le Hoang 4 GJarošeBO – – 3 – 5 0 5 0 13 10,81

154.–157. Barbora Hálková 2 SPŠStav LI 3 3 1 – – – – – 7 10,72154.–157. Andrej Horník 2 GAnMeTr 3 3 – – – 0 1 0 7 10,72154.–157. Hana Jirovská 2 NPorg 3 1 3 0 – – – – 7 10,72154.–157. Jakub Zápotocký 2 GOpatovPH 3 3 0 – – – 1 – 7 10,72158.–159. Karolína Slaběňáková 0 ZŠ ValKlob 3 – 1 – – 0 0 – 4 10,11158.–159. Jakub Ucháč 0 ZŠVranéNVl 3 0 – – – – 1 – 4 10,11160.–162. Magdaléna Bártová 1 GDašickáPA 3 – – – 1 – 1 – 5 10,00160.–162. Jakub Čurda 1 PORG PH 3 2 – – – – – – 5 10,00160.–162. Kateřina Vaňková 4 GJarošeBO 3 3 3 – – – 1 0 10 10,00163.–166. Kateřina Čížková 2 G Rokycany 3 3 – – – 0 – – 6 9,48163.–166. Aneta Němcová 2 GBoskovice 3 3 – – – – – – 6 9,48163.–166. Krystyna Waniová 2 G HavlČTěš 3 3 – – – – – 0 6 9,48163.–166. Radek Wágner 2 GMikul23PL 3 3 – – – – – – 6 9,48

167. Barbora Mouleová 2 G Plasy 3 – 3 – – – – – 6 9,22168.–170. Vojtěch Janků 4 CMGPgBrno 3 3 3 0 – – – 0 9 9,00168.–170. Přemysl Kopečný 4 ČRG ČesBuď 3 3 1 0 1 0 1 0 9 9,00168.–170. Peter Macko 4 GJHroncaBA 3 3 3 – – – – – 9 9,00171.–172. Karolína Tulingerová 1 AkademG PH 3 – – – 1 – – – 4 8,48171.–172. Kateřina Žáková 1 SlovanG OL 3 – – – – – 1 – 4 8,48

173. Marek Murin 4 GJHroncaBA 3 3 3 – – – – – 9 8,45174. Jan Antonín Musil 0 PORG PH 3 – – – – – – – 3 8,34

175.–180. Adam Dobrovič 3 GTajBanBys 3 3 – – – – – – 6 8,00175.–180. Jan Dopita 3 GBudějovPH 3 3 – – – – – – 6 8,00175.–180. Ľubica Hladká 3 GTajBanBys 3 3 – – – – – – 6 8,00175.–180. Leoš Smetana 3 G Jaroměř 3 3 0 – – – – – 6 8,00175.–180. Kateřina Škorvánková 3 G Rokycany 3 3 – – – – – – 6 8,00175.–180. Zuzana Urbanová 3 GUBalvanJN 3 3 – – – – – – 6 8,00

181. Jana Řežábková 4 PORG PH 3 3 3 – – – – – 9 7,73182.–187. Monika Berková 1 GKepleraPH 3 – – – – – – – 3 6,79182.–187. Jan Česnek 1 GJarošeBO 3 – – – – – – – 3 6,79182.–187. Vít Gaďurek 1 PORG PH 3 – – – – – – – 3 6,79182.–187. Šimon Hutař 1 PORG PH 3 – – – – – – 0 3 6,79182.–187. Radim Novotný 1 GMikul23PL 3 – – – – – – – 3 6,79182.–187. Kateřina Šauerová 1 GPSJazykHK 3 – – – – – – – 3 6,79

188. Adam Španěl 4 3 3 – – – – – – 6 6,00189. Jana Řičicová 3 GCoubTábor 3 – – – – – 1 0 4 5,53190. Matúš Varhaník 2 G Bytča 3 – – – – – – 0 3 5,27191. Jan Dittrich 4 GJarošeBO 3 3 – – – – – – 6 5,24192. Marie Freibergová 3 G Děčín 0 3 – – – – – – 3 4,23193. Klára Stefanová 4 GBNěmcovHK – 3 – – – – – – 3 3,00194. Jakub Ševčík 4 GKukučPopr 3 – – – – – – – 3 2,55195. Filip Pastierovič 3 G ĽŠtúraZV 0 1 – – – – – – 1 1,47

46

Page 47: Matematický korespondenční seminář Milýpříteli! · těžiště trojúhelníka KNP. Poměr, v kterém těžiště dělí těžnici, je přitom dobře znám – kýžený výsledek

3. podzimní série – Úlohy na šachovniciVýsledková listina

1.–2. Filip Bialas 3 GOpatovPH – – – 5 5 5 5 5 25 25,001.–2. Pavel Turek 3 GTomkovaOL 3 3 3 5 5 5 5 5 25 25,003. Samuel Krajči 1 GAlejKošic 3 3 – 5 5 5 5 – 23 24,274. Lenka Kopfová 1 MendelG OP – – 3 5 5 5 5 – 23 24,045. Slavomír Hanzely 4 GRaymanaPV – – – 5 5 5 5 4 24 24,006. Jan Petr 3 GKepleraPH 3 – 3 5 5 5 5 – 23 + i 23,827. Pavel Hudec 2 GJarkovPH 3 3 3 5 5 5 5 – 23 23,748. Petr Gebauer 2 G Mělník 3 3 – 5 5 5 5 – 23 23,729. Richard Hladík 3 GaOA MarLáz 3 3 3 5 5 5 5 – 23 + i 23,70

10.–12. Ondřej Motlíček 3 G Šumperk 3 3 3 5 5 5 5 3 23 23,6310.–12. Jáchym Solecký 3 PORG PH 3 3 3 5 5 5 5 – 23 23,6310.–12. Václav Volhejn 3 GKepleraPH 3 3 3 1 5 5 5 5 23 23,63

13. Danil Koževnikov 2 GKepleraPH 3 – – 1 5 5 5 5 23 23,6114. Václav Steinhauser 2 G Dačice 3 3 0 4 5 5 5 4 23 23,3115. Veronika Hladíková 3 GMikul23PL 3 – 3 5 5 5 5 – 23 23,2616. Tomáš Domes 3 MendelG OP 3 3 3 5 5 5 5 – 23 23,2417. Radek Olšák 1 GMensaPH 3 3 – – 5 5 5 – 21 + i 23,2318. Filip Čermák 2 MendelG OP 3 2 3 5 5 5 – – 21 22,7719. Tomáš Konečný 3 GJirsíkaČB 3 – 3 5 5 5 5 – 23 22,6920. Ondřej Krabec 1 G KomHavíř 3 3 3 5 5 – – – 19 22,4321. Marian Poljak 4 GJŠkodyPŘ 3 – 3 5 5 5 5 – 23 + i 22,3122. Lucien Šíma 4 PORG PH 3 – 3 5 5 5 5 – 23 22,2923. František Couf 3 EKO GPraha 3 – 3 5 5 2 5 5 23 + i 22,24

24.–27. Šimon Chvátil 1 GBNěmcovHK 3 2 3 3 5 0 4 – 18 21,8824.–27. Lucie Kundratová 1 G TGM Zlín 3 3 3 1 – 5 4 – 18 21,8824.–27. Jana Pallová 1 3 3 3 – 5 4 – – 18 21,8824.–27. Minh Duc Pham 1 NPorg 1 3 3 2 5 5 – – 18 21,88

28. Jan Šorm 4 GJarošeBO 3 3 3 5 5 5 5 – 23 + i 21,6729. Lucia Krajčoviechová 0 GJHroncaBA 3 – 3 3 5 2 – – 16 21,6430. Michal Töpfer 3 GJPekařeMB 3 2 3 5 5 5 – – 21 21,5931. Vojtěch Lengál 2 GZborovPH 3 3 3 5 5 1 – – 19 21,52

32.–33. Matouš Trnka 3 GJarošeBO 3 3 – 5 5 4 3 – 20 21,4132.–33. Daniele Venier 3 LSciARoiti 3 3 3 4 – 5 5 3 20 21,4134.–35. Jakub Domes 1 MendelG OP 3 – 3 3 5 – – 3 17 21,3034.–35. Timur Sibgatullin 1 PČGKarVary 3 3 3 2 0 1 5 3 17 21,30

36. Adam Španěl 4 3 2 3 – 5 5 5 – 21 + i 21,2737. Daniel Pišťák 4 GZborovPH 3 – – 5 5 5 5 – 23 20,7338. Adam Mendl 0 GCoubTábor 3 – 3 – 5 5 – – 16 20,70

39.–41. Matěj Doležálek 1 G Humpolec 3 – 3 – – 5 5 – 16 20,67

47

Page 48: Matematický korespondenční seminář Milýpříteli! · těžiště trojúhelníka KNP. Poměr, v kterém těžiště dělí těžnici, je přitom dobře znám – kýžený výsledek

Matematický korespondenční seminář 35. ročník (2015/2016), 2. komentáře

39.–41. Pavel Havlín 1 NPorg 3 2 3 3 – 5 – – 16 20,6739.–41. Dominika Mokroszová 1 G FrýdČTěš 3 2 3 3 5 1 1 – 16 20,67

42. Jakub Löwit 4 GČeskoliPH 3 2 3 5 5 5 5 – 23 20,4243. Ondřej Svoboda 3 GJarošeBO 3 3 3 5 5 – – – 19 20,22

44.–46. Alexandr Jankov 2 MatičníGOS 3 – 0 3 5 1 3 3 17 20,1244.–46. Petr Ježek 2 GBNěmcovHK 3 3 3 – 5 1 – 3 17 20,1244.–46. Matúš Komora 2 GLettMart 3 2 3 5 4 2 – 0 17 20,12

47. Daniel Kopf 4 SlezkéG OP 3 – 3 5 5 – 5 – 21 19,9748. Adéla Kostelecká 4 GLesníZlín 3 2 – 5 5 3 5 – 21 19,9149. Alžběta Neubauerová 2 GNadKavaPH 3 2 3 4 – 5 – – 17 19,9050. David Neugebauer 3 SlezkéG OP 3 3 3 3 5 2 4 – 18 19,8351. Ondřej Dušek 2 PORG PH 3 3 3 1 5 2 – – 16 19,37

52.–54. Daniel Borák 1 GŠpitálsPH 0 – 3 – 5 1 2 3 14 19,2852.–54. Viktória Brezinová 1 GAlejKošic 3 3 3 5 – – – – 14 19,2852.–54. Matěj Kraft 1 GMikul23PL 3 2 3 2 4 – 1 – 14 19,28

55. Kateřina Nová 3 G Vimperk 3 – 3 4 5 4 – – 19 19,1456. Veronika Roubínová 2 G Kadaň 3 2 3 2 5 – – – 15 + i 18,8057. Vojtěch Lanz 2 GZborovPH 3 3 3 3 5 3 – – 17 18,66

58.–61. Ondřej Buček 2 GJarošeBO 3 3 3 – 5 1 – – 15 18,5958.–61. Marie Dohnalová 3 GNadKavaPH 3 3 3 4 – 5 – – 18 18,5958.–61. David Królikowski 2 G Karviná 3 2 3 2 5 2 0 – 15 18,5958.–61. Anna Šírová 2 GJilemnice 3 – 3 3 5 – 1 – 15 18,59

62. Marco Souza de Joode 0 3 3 3 2 – – – – 11 + i 18,4563. Hedvika Ranošová 2 GBudějovPH 3 3 0 5 5 – – – 16 18,2764. Jaromír Mielec 3 GVolgogrOS 3 2 3 5 5 4 – – 20 18,2565. Filip Chudoba 2 PORG PH 3 2 3 2 5 – – – 15 18,1866. Victoria María Nájares Romero 2 GZborovPH 3 2 3 3 5 2 1 – 16 18,1567. Kateřina Charvátová 1 GBNěmcovHK 3 2 3 3 0 1 1 0 12 17,7068. Vít Kalisz 4 FSG Pirna 3 3 3 3 4 5 4 – 19 17,2869. Lenka Vincenová 2 GTomkovaOL 3 3 0 2 4 – 1 – 13 + i 17,14

70.–72. Jáchym Bareš 2 GTomkovaOL 3 1 3 3 – – 3 – 13 16,9070.–72. Tomáš Čelko 2 GPBystrica 3 – 3 0 4 – 3 – 13 16,9070.–72. Vendula Kuchyňová 2 GMLerchaBO 3 2 3 – 5 – – – 13 16,9073.–76. Ondřej Bursa 1 GTNovákBO 3 2 3 3 – – – – 11 16,8373.–76. Martina Kalašová 1 GJHroncaBA 3 1 3 3 – – 1 – 11 16,8373.–76. Josef Král 1 MendelG OP 3 3 3 2 – – 0 – 11 16,8373.–76. Kristýna Lhoťanová 1 G RožnRadh 3 2 3 2 1 1 – 0 11 16,83

77. Denisa Chytilová 2 GJŠkodyPŘ 1 2 0 2 – 5 – 3 13 16,2378. Tomáš Drobil 1 G Dačice 3 2 3 2 – – – – 10 15,8979. Marek Pospíšil 3 GJatečníÚL 3 2 3 3 – 2 1 – 13 + i 15,6880. Martin Pašen 3 GRaymanaPV 3 2 3 – – 1 4 – 13 15,4381. Kamila Kyzlíková 2 GZborovPH 3 2 3 3 0 1 – – 12 15,2382. Aleš Krčil 3 G Humpolec 3 3 3 4 – – 1 – 14 15,13

83.–85. Vojtech Filipi 1 GDašickáPA 3 2 3 0 – – 1 – 9 14,8983.–85. Martin Simet 1 GMikul23PL 3 0 3 – 1 1 1 – 9 14,8983.–85. Martin Zimen 1 GJMasar JI – 3 3 3 – – – – 9 14,89

86. Zuzana Tréglová 3 G Žatec 3 3 0 1 4 – 1 – 12− i 14,2087. Dominika Zumrová 2 SPŠ PansPH 3 2 3 – – 1 1 – 10 14,0588. Zuzana Šraierová 1 GČeskoliPH 3 0 3 2 – – – 0 8 13,8189. Marek Malý 3 G Neratov 3 2 3 4 – – – – 12 13,58

90.–91. Vojtěch Jílek 3 VOŠKutHora 3 1 3 – 4 – – – 11 13,4790.–91. Dan Raffl 3 GVoděraPH 3 1 3 3 1 – – – 11 13,47

48

Page 49: Matematický korespondenční seminář Milýpříteli! · těžiště trojúhelníka KNP. Poměr, v kterém těžiště dělí těžnici, je přitom dobře znám – kýžený výsledek

Korespondenční seminář, KAM MFF UK, Malostranské náměstí 25, 118 00 Praha 1

92.–93. Anna Musilová 0 PORG PH 3 – 3 – – – – – 6 13,0392.–93. Jakub Ucháč 0 ZŠVranéNVl 3 2 0 – – 1 – – 6 13,0394.–95. Nikola Kalábová 2 FSG Pirna 3 – 3 2 – 1 – – 9 13,0094.–95. Martin Spišák 2 GAlejKošic 3 2 0 – – – 4 – 9 13,00

96. Kristína Szabová 2 GVarŽilina 3 – 3 3 – – – – 9 12,6997.–103. Michal Chudoba 1 GLitoměřPH 3 1 3 – – – – – 7 12,6497.–103. Duc Long Hoang 1 GMikul23PL 3 0 3 – 0 – 1 – 7 12,6497.–103. Eleonora Krůtová 1 GJarošeBO 3 1 3 – – – – – 7 12,6497.–103. Ondřej Meduna 1 GMikul23PL 3 1 3 – – – – – 7 12,6497.–103. Anna Mírková 1 G LPika PL 1 1 3 2 – – – 0 7 12,6497.–103. David Šnajdr 1 GMikul23PL 3 1 3 – 0 0 0 0 7 12,6497.–103. Kateřina Žáková 1 SlovanG OL 3 – 3 – – – 1 – 7 12,64

104. Jiří Češka 3 CMGProstěj 3 2 3 – – 2 – – 10 12,45105. Jaroslav Paidar 2 SPŠMasarLI 3 3 3 – – – 0 – 9 12,23106. Adam Doubrava 0 GMasarykKM 3 3 – – – – – – 6 12,09107. Andrej Horník 2 GAnMeTr 3 – 3 2 – – – – 8 11,89108. Filip Matějka 2 GZborovPH 3 2 3 – – – – – 8 11,84109. Barbora Lišková 3 GJPekařeMB 3 – 3 3 – – – – 9 11,39

110.–112. Alexander Csizmár 1 GMikul23PL 3 0 3 – 0 – – – 6 11,38110.–112. Pavel Čácha 1 GMikul23PL 3 0 3 – – – – – 6 11,38110.–112. Zdeněk Vostřel 1 3 – 0 – – 2 1 – 6 11,38113.–115. Adéla Seidelmannová 2 VOŠRychnovKn 3 0 3 1 – – – 0 7 10,72113.–115. Martina Šmehylová 2 GHlinŽilina 3 – 3 – 0 – 1 – 7 10,72113.–115. Krystyna Waniová 2 G HavlČTěš 3 1 3 – – – – – 7 10,72

116. Barbora Mouleová 2 G Plasy 3 – 3 – – – 1 – 7 10,45117. Samuel Hrubý 3 GFraŽilina 3 2 3 – – – – – 8 10,30118. Kateřina Pařízková 1 MatičníGOS 3 2 – – – – – – 5 10,00119. Matej Kvorka 2 GŠkolDubni 3 0 3 – – – – – 6 9,48

120.–121. Jan Klásek 3 SlovanG OL 3 1 0 2 – 1 0 – 7 9,17120.–121. Jana Menšíková 3 3 – 3 – – – 1 – 7 9,17122.–123. Vít Gaďurek 1 PORG PH 3 – 0 – 1 – – – 4 8,48122.–123. Alžběta Manová 1 G UherBrod – 2 – 2 – – – – 4 8,48

124. Jan Antonín Musil 0 PORG PH 3 – – – – – – – 3 8,34125.–128. Michael Borák 3 GKepleraPH 3 – 3 – – – – – 6 8,00125.–128. Jan Hrůza 3 G Kadaň 3 – 3 – – – – – 6 8,00125.–128. Michaela Macáková 4 GOhradníPH 3 2 3 – – – – – 8 8,00125.–128. Valerie Skopalová 4 G VysMýto 3 2 3 – – – – – 8 8,00

129. Lucie Hronová 3 GJarošeBO 3 1 0 2 – – – – 6 7,38130. Natálie Mikolajová 4 G HavlBrod 3 – 0 2 – – 1 1 7 7,00

131.–139. Magdaléna Bártová 1 GDašickáPA 3 – – – – – – – 3 6,79131.–139. Jan Česnek 1 GJarošeBO 3 – – – – – – – 3 6,79131.–139. Erik Kočandrle 1 GMikul23PL 3 0 0 – – – – – 3 6,79131.–139. Tereza Lukášová 1 GMikul23PL 3 – 0 – – – – – 3 6,79131.–139. Filip Müller 1 GMikul23PL 3 – 0 – – – – – 3 6,79131.–139. Filip Oplt 3 GBudějovPH 3 2 0 – – – – – 5 6,79131.–139. Anna Šebestíková 1 GCON ČesBuď 3 – – – – – – – 3 6,79131.–139. Samuel Štrunc 1 GMikul23PL 3 – 0 – – – – – 3 6,79131.–139. Linh Giang Tran 1 GMikul23PL 3 – – – – – – – 3 6,79140.–141. David Krajíček 2 BG Ostrava 3 – 0 – – – – – 3 5,27140.–141. Jakub Zápotocký 2 GOpatovPH 3 – 0 – – – – – 3 5,27

142. Jan Dittrich 4 GJarošeBO 3 – 3 – – – – – 6 5,24143.–144. Filip Strakoš 3 G TGM Zlín 3 – 0 – – – – – 3 4,23

49

Page 50: Matematický korespondenční seminář Milýpříteli! · těžiště trojúhelníka KNP. Poměr, v kterém těžiště dělí těžnici, je přitom dobře znám – kýžený výsledek

Matematický korespondenční seminář 35. ročník (2015/2016), 2. komentáře

143.–144. Kateřina Škorvánková 3 G Rokycany 3 – – – – – – – 3 4,23145. Hana Jirovská 2 NPorg – – 1 – – – 1 – 2 3,65146. Filip Pastierovič 3 G ĽŠtúraZV – – – 1 0 1 – – 2 2,88147. Denisa Jandová 1 GMikul23PL – 1 – – – – – – 1 2,67148. Kateřina Vaňková 4 GJarošeBO 1 – 0 – – – – – 1 1,00

149.–152. Vojtěch Kantor 0 GNerudCheb 0 0 0 – 0 – – – 0 0,00149.–152. Vladimír Lukačko 2 GVarŽilina – – 0 – – – – – 0 0,00149.–152. Aneta Němcová 2 GBoskovice – – 0 – – – – 0 0 0,00149.–152. Evžen Wybitul 2 – – 0 – – – – – 0 0,00

50

Page 51: Matematický korespondenční seminář Milýpříteli! · těžiště trojúhelníka KNP. Poměr, v kterém těžiště dělí těžnici, je přitom dobře znám – kýžený výsledek

1. seriálová série – Do nekonečna a ještě dálVýsledková listina

1.–2. Filip Bialas 3 GOpatovPH 5 5 5 15 + i 15,001.–2. Pavel Turek 3 GTomkovaOL 5 5 5 15 + i 15,003. Danil Koževnikov 2 GKepleraPH 5 5 4 14 + i 14,504. Radek Olšák 1 GMensaPH 5 5 0 10 12,275. Petr Gebauer 2 G Mělník 5 5 – 10 + i 11,656. Lenka Kopfová 1 MendelG OP 4 5 – 9 + i 11,587. Richard Hladík 3 GaOA MarLáz 5 5 – 10 + i 11,198. Jáchym Solecký 3 PORG PH 4 5 – 9 + i 10,609. Hedvika Ranošová 2 GBudějovPH 5 3 – 8 9,5310. Lucien Šíma 4 PORG PH 5 5 – 10 + i 9,13

11.–12. Matěj Doležálek 1 G Humpolec 5 – – 5 8,5111.–12. Jakub Domes 1 MendelG OP 5 – – 5 8,51

13. Filip Čermák 2 MendelG OP 5 1 – 6 8,4014. Marian Poljak 4 GJŠkodyPŘ 5 5 – 10 8,3215. Jakub Löwit 4 GČeskoliPH 5 5 1 11 + i 8,2616. Victoria María Nájares Romero 2 GZborovPH 1 5 – 6 + i 7,7517. Slavomír Hanzely 4 GRaymanaPV 5 2 0 7 7,0018. Daniel Kopf 4 SlezkéG OP 5 3 – 8 6,9619. Pavel Hudec 2 GJarkovPH 5 – – 5 6,7220. Václav Volhejn 3 GKepleraPH – 5 – 5 + i 6,6921. Václav Steinhauser 2 G Dačice 1 5 0 6 6,6622. Tomáš Domes 3 MendelG OP 5 1 – 6 6,5123. Matouš Trnka 3 GJarošeBO 5 – – 5 6,4024. Tomáš Konečný 3 GJirsíkaČB – 5 – 5 + i 4,7425. Ondřej Krabec 1 G KomHavíř – 2 – 2 4,4326. Denisa Chytilová 2 GJŠkodyPŘ 2 – – 2 3,1427. Daniel Pišťák 4 GZborovPH 5 – – 5 2,67

28.–30. Viktória Brezinová 1 GAlejKošic 0 1 – 1 2,5128.–30. Kateřina Charvátová 1 GBNěmcovHK – 1 – 1 2,5128.–30. Lucie Kundratová 1 G TGM Zlín – 1 – 1 2,51

31. Jaromír Mielec 3 GVolgogrOS 3 – – 3 2,1732. Alexandr Jankov 2 MatičníGOS – 1 – 1 1,85

33.–35. Ondřej Motlíček 3 G Šumperk – 1 0 1 1,4533.–35. Filip Pastierovič 3 G ĽŠtúraZV 0 1 0 1 1,4533.–35. Daniele Venier 3 LSciARoiti – 1 0 1 1,45

36. Michal Töpfer 3 GJPekařeMB 1 – 0 1 1,1937. Veronika Hladíková 3 GMikul23PL 1 0 – 1 1,1538. Adéla Kostelecká 4 GLesníZlín – 1 – 1 0,76

39.–44. Marie Dohnalová 3 GNadKavaPH – – 0 0 0,0039.–44. Vojtech Filipi 1 GDašickáPA 0 – – 0 0,0039.–44. Samuel Krajči 1 GAlejKošic 0 – – 0 0,0039.–44. Aleš Krčil 3 G Humpolec 0 – – 0 0,0039.–44. Kateřina Vaňková 4 GJarošeBO 0 0 – 0 0,0039.–44. Martin Zimen 1 GJMasar JI – 0 – 0 0,00

51

Page 52: Matematický korespondenční seminář Milýpříteli! · těžiště trojúhelníka KNP. Poměr, v kterém těžiště dělí těžnici, je přitom dobře znám – kýžený výsledek

Výsledky po 3. podzimní sérii

1. Filip Bialas 3 GOpatovPH 25 25 25 – 15 90,00 5402. Pavel Turek 3 GTomkovaOL 23 25 25 – 15 88,20 7153. Danil Koževnikov 2 GKepleraPH 24 25 24 – 15 86,93 3204. Petr Gebauer 2 G Mělník 24 22 24 – 12 80,90 2485. Lenka Kopfová 1 MendelG OP 22 23 24 – 12 80,51 2406. Jáchym Solecký 3 PORG PH 21 25 24 – 11 79,86 807. Jakub Löwit 4 GČeskoliPH 25 25 20 – 8 78,68 7138. Richard Hladík 3 GaOA MarLáz 22 20 24 – 11 77,08 1489. Lucien Šíma 4 PORG PH 20 25 22 – 9 76,16 25310. Radek Olšák 1 GMensaPH 18 22 23 – 12 75,92 18711. Pavel Hudec 2 GJarkovPH 23 22 24 – 7 75,43 22912. Filip Čermák 2 MendelG OP 20 23 23 – 8 74,03 8113. Václav Steinhauser 2 G Dačice 20 22 23 – 7 71,81 44114. Ondřej Krabec 1 G KomHavíř 21 22 22 – 4 70,13 7015. Samuel Krajči 1 GAlejKošic 23 22 24 – 0 69,27 6916. Tomáš Domes 3 MendelG OP 20 20 23 – 7 68,97 23417. Marian Poljak 4 GJŠkodyPŘ 19 19 22 – 8 68,89 35618. Matěj Doležálek 1 G Humpolec 19 21 21 – 9 68,83 6919. Lucie Kundratová 1 G TGM Zlín 22 22 22 – 3 68,70 6920. Jakub Domes 1 MendelG OP 19 19 21 – 9 68,37 6821. Slavomír Hanzely 4 GRaymanaPV 21 16 24 – 7 68,00 6822. Victoria María Nájares Romero 2 GZborovPH 20 22 18 – 8 67,91 29223. František Couf 3 EKO GPraha 20 25 22 – – 67,62 60524. Minh Duc Pham 1 NPorg 22 23 22 – – 67,18 6725. Lucia Krajčoviechová 0 GJHroncaBA 22 23 22 – – 66,92 6726. Veronika Hladíková 3 GMikul23PL 21 21 23 – 1 66,47 22527. Ondřej Motlíček 3 G Šumperk 19 22 24 – 1 66,25 6628. Jan Petr 3 GKepleraPH 21 21 24 – – 65,86 6629. Hedvika Ranošová 2 GBudějovPH 16 22 18 – 10 65,60 27230. Viktória Brezinová 1 GAlejKošic 21 22 19 – 3 64,89 6531. Tomáš Konečný 3 GJirsíkaČB 16 21 23 – 5 64,55 36032. Alexandr Jankov 2 MatičníGOS 17 22 20 – 2 61,04 6133. Jana Pallová 1 19 20 22 – – 60,96 6134. Daniele Venier 3 LSciARoiti 17 21 21 – 1 60,10 6035. Vojtěch Lanz 2 GZborovPH 20 20 19 – – 59,38 33836. Daniel Kopf 4 SlezkéG OP 12 20 20 – 7 59,17 19837. Adam Mendl 0 GCoubTábor 18 21 21 – – 59,15 16738. Marie Dohnalová 3 GNadKavaPH 22 19 19 – 0 58,82 23339. Denisa Chytilová 2 GJŠkodyPŘ 19 20 16 – 3 58,78 167

40.–41. Matúš Komora 2 GLettMart 17 22 20 – – 58,54 5940.–41. Daniel Borák 1 GŠpitálsPH 17 22 19 – – 58,54 59

42. Pavel Havlín 1 NPorg 16 22 21 – – 58,44 5843. Adéla Kostelecká 4 GLesníZlín 18 20 20 – 1 58,14 20744. Vojtěch Lengál 2 GZborovPH 15 22 22 – – 58,09 5845. Kateřina Nová 3 G Vimperk 19 19 19 – – 57,68 271

52

Page 53: Matematický korespondenční seminář Milýpříteli! · těžiště trojúhelníka KNP. Poměr, v kterém těžiště dělí těžnici, je přitom dobře znám – kýžený výsledek

Korespondenční seminář, KAM MFF UK, Malostranské náměstí 25, 118 00 Praha 1

46. Kamila Kyzlíková 2 GZborovPH 20 21 15 – – 56,65 17547. Martina Kalašová 1 GJHroncaBA 19 20 17 – – 56,31 5648. Jan Šorm 4 GJarošeBO 18 17 22 – – 56,30 54149. Vendula Kuchyňová 2 GMLerchaBO 17 22 17 – – 56,14 5650. Tomáš Čelko 2 GPBystrica 19 19 17 – – 55,64 5651. Ondřej Buček 2 GJarošeBO 15 22 19 – – 55,16 5552. Eleonora Krůtová 1 GJarošeBO 19 23 13 – – 54,58 5553. Jaromír Mielec 3 GVolgogrOS 17 17 18 – 2 54,19 49854. Michal Töpfer 3 GJPekařeMB 14 17 22 – 1 53,99 19455. Daniel Pišťák 4 GZborovPH 15 16 21 – 3 53,78 63856. Matěj Kraft 1 GMikul23PL 14 21 19 – – 53,76 5457. Veronika Roubínová 2 G Kadaň 18 17 19 – – 53,47 5358. Petr Ježek 2 GBNěmcovHK 15 18 20 – – 52,94 5359. Kateřina Charvátová 1 GBNěmcovHK 15 18 18 – 3 52,80 5360. Václav Volhejn 3 GKepleraPH 22 – 24 – 7 52,49 5261. Kristína Szabová 2 GVarŽilina 20 19 13 – – 51,74 9762. Lenka Vincenová 2 GTomkovaOL 13 22 17 – – 51,66 5263. Martin Pašen 3 GRaymanaPV 20 16 15 – – 51,62 5264. Ondřej Dušek 2 PORG PH 14 18 19 – – 51,19 5165. Alžběta Neubauerová 2 GNadKavaPH 13 18 20 – – 51,17 99

66.–67. Šimon Chvátil 1 GBNěmcovHK 16 13 22 – – 50,41 5066.–67. Anna Šírová 2 GJilemnice 14 18 19 – – 50,41 50

68. Filip Matějka 2 GZborovPH 17 21 12 – – 50,18 5869. Ondřej Svoboda 3 GJarošeBO 10 20 20 – – 50,08 13070. Jaroslav Paidar 2 SPŠMasarLI 20 17 12 – – 48,72 15971. Ondřej Bursa 1 GTNovákBO 15 17 17 – – 48,55 4972. Dominika Mokroszová 1 G FrýdČTěš 13 15 21 – – 48,20 4873. Dominika Zumrová 2 SPŠ PansPH 16 18 14 – – 47,82 4874. Martin Spišák 2 GAlejKošic 17 18 13 – – 47,67 4875. Vít Kalisz 4 FSG Pirna 17 13 17 – – 47,43 23376. Marek Pospíšil 3 GJatečníÚL 14 17 16 – – 47,42 4777. Tomáš Drobil 1 G Dačice 11 19 16 – – 46,55 4778. Marco Souza de Joode 0 11 17 18 – – 46,35 4679. Adam Španěl 4 18 6 21 – – 45,27 4580. Jáchym Bareš 2 GTomkovaOL 15 13 17 – – 44,95 4581. David Neugebauer 3 SlezkéG OP 12 12 20 – – 44,73 4582. Timur Sibgatullin 1 PČGKarVary 7 16 21 – – 43,98 4483. Aleš Krčil 3 G Humpolec 13 15 15 – 0 43,42 18484. Kristýna Lhoťanová 1 G RožnRadh 13 14 17 – – 43,28 4385. Matouš Trnka 3 GJarošeBO 15 – 21 – 6 43,24 4386. Zdeněk Vostřel 1 14 18 11 – – 42,89 4387. Matej Kvorka 2 GŠkolDubni 16 16 9 – – 41,48 4188. Nikola Kalábová 2 FSG Pirna 13 15 13 – – 41,05 4189. Michal Chudoba 1 GLitoměřPH 11 17 13 – – 40,85 4190. Filip Chudoba 2 PORG PH 9 14 18 – – 40,71 11591. Adéla Seidelmannová 2 VOŠRychnovKn 12 18 11 – – 40,38 40

92.–93. Pavel Čácha 1 GMikul23PL 14 15 11 – – 40,08 4092.–93. Martin Zimen 1 GJMasar JI 14 11 15 – 0 40,08 40

94. David Królikowski 2 G Karviná – 21 19 – – 39,43 3995. Jiří Češka 3 CMGProstěj 13 13 12 – – 39,39 3996. Anna Musilová 0 PORG PH 13 13 13 – – 39,09 3997. Peter Súkeník 4 GOkrŽilina 21 18 – – – 39,00 3998. Tereza Vlčková 2 GJarošeBO 22 17 – – – 38,42 3899. Erik Kočandrle 1 GMikul23PL 11 20 7 – – 38,17 38100. Martin Bakoš 2 GPBystrica 19 19 – – – 37,76 38101. Filip Oplt 3 GBudějovPH 15 15 7 – – 37,65 38102. Adam Doubrava 0 GMasarykKM 12 13 12 – – 37,52 95

53

Page 54: Matematický korespondenční seminář Milýpříteli! · těžiště trojúhelníka KNP. Poměr, v kterém těžiště dělí těžnici, je přitom dobře znám – kýžený výsledek

Matematický korespondenční seminář 35. ročník (2015/2016), 2. komentáře

103. Andrej Horník 2 GAnMeTr 14 11 12 – – 36,66 37104. Marek Malý 3 G Neratov 11 13 14 – – 36,65 139105. Dan Raffl 3 GVoděraPH 11 11 13 – – 36,25 36106. David Krajíček 2 BG Ostrava 14 17 5 – – 36,22 36107. Jakub Ucháč 0 ZŠVranéNVl 13 10 13 – – 36,17 36108. Vojtěch Jílek 3 VOŠKutHora 10 11 13 – – 35,16 35109. Anna Mírková 1 G LPika PL 7 15 13 – – 34,32 34110. Monika Machalová 1 GJHroncaBA 15 19 – – – 34,17 34111. Barbora Lišková 3 GJPekařeMB 10 12 11 – – 34,14 34112. Alžběta Manová 1 G UherBrod 13 13 8 – – 33,76 34113. Alexander Csizmár 1 GMikul23PL 7 15 11 – – 33,06 33114. Jan Klásek 3 SlovanG OL 10 13 9 – – 32,94 33115. Šárka Míchalová 1 G Kralupy 15 18 – – – 32,59 33116. Kateřina Žáková 1 SlovanG OL 11 8 13 – – 32,50 33117. Jan Hrůza 3 G Kadaň 8 16 8 – – 32,36 32118. Samuel Hrubý 3 GFraŽilina 10 11 10 – – 31,99 32119. Evžen Wybitul 2 18 14 0 – – 31,82 32120. Josef Král 1 MendelG OP – 15 17 – – 31,72 32121. Linh Giang Tran 1 GMikul23PL 10 15 7 – – 31,68 32122. Valerie Skopalová 4 G VysMýto 12 11 8 – – 31,00 31123. Filip Müller 1 GMikul23PL 11 13 7 – – 30,81 31124. John Richard Ritter 2 G MasNámTŘ 18 13 – – – 30,77 31125. Zuzana Tréglová 3 G Žatec 17 13 – – – 30,01 180126. Tereza Lukášová 1 GMikul23PL 11 11 7 – – 29,55 30127. Hana Jirovská 2 NPorg 15 11 4 – – 29,42 29128. Martina Šmehylová 2 GHlinŽilina 7 12 11 – – 29,37 29129. Vladimír Lukačko 2 GVarŽilina 14 16 0 – – 29,23 90130. Filip Strakoš 3 G TGM Zlín 8 16 4 – – 28,59 29131. Anh Minh Tran 4 GJarošeBO 10 17 – – – 27,27 27132. Daniel Ridzoň 2 GKepleraPH 14 13 – – – 27,05 27133. Krystyna Waniová 2 G HavlČTěš 7 9 11 – – 26,96 27134. Vít Gaďurek 1 PORG PH 11 7 8 – – 26,65 27135. Antonín Štrpka 1 G Šumperk 11 15 – – – 26,27 26136. Zuzana Urbanová 3 GUBalvanJN 18 8 – – – 26,15 26137. Jana Menšíková 3 4 12 9 – – 25,85 26138. Kateřina Čížková 2 G Rokycany 16 9 – – – 25,48 25139. Jakub Zápotocký 2 GOpatovPH 9 11 5 – – 25,47 25140. Denisa Jandová 1 GMikul23PL 11 11 3 – – 25,43 25141. Lubomír Smrček 1 11 14 – – – 25,19 25142. Marek Murin 4 GJHroncaBA 16 8 – – – 24,91 65143. Matěj Coufal 3 G HavlBrod 11 13 – – – 24,86 25144. Barbora Mouleová 2 G Plasy 5 9 10 – – 24,74 65145. Jana Řežábková 4 PORG PH 17 8 – – – 24,52 133146. Adam Dobrovič 3 GTajBanBys 16 8 – – – 24,36 24147. Ondřej Meduna 1 GMikul23PL – 11 13 – – 24,02 24148. Monika Suchá 1 GMikul23PL 11 11 – – – 22,76 23149. Andrej Židek 1 GJKTyla HK 7 16 – – – 22,68 23150. Petr Zahradník 1 GŠmejkalÚL 10 13 – – – 22,64 23151. Peter Macko 4 GJHroncaBA 13 9 – – – 22,46 22152. Matthew Dupraz 1 NPorg – 22 – – – 22,43 22153. Zuzana Svobodová 4 G FrýdlNOs 4 18 – – – 22,13 428154. Jan Česnek 1 GJarošeBO 8 7 7 – – 22,06 22155. Vojtěch Janků 4 CMGPgBrno 13 9 – – – 22,00 22156. Tomáš Hampl 3 GDukelBR 10 11 – – – 21,69 22157. Ľubica Hladká 3 GTajBanBys 13 8 – – – 21,47 21

158.–159. Jakub Čurda 1 PORG PH 11 10 – – – 21,38 21158.–159. Kateřina Pařízková 1 MatičníGOS 11 – 10 – – 21,38 21

54

Page 55: Matematický korespondenční seminář Milýpříteli! · těžiště trojúhelníka KNP. Poměr, v kterém těžiště dělí těžnici, je přitom dobře znám – kýžený výsledek

Korespondenční seminář, KAM MFF UK, Malostranské náměstí 25, 118 00 Praha 1

160. Radek Wágner 2 GMikul23PL 12 9 – – – 21,37 21161. Matúš Varhaník 2 G Bytča 15 5 – – – 20,32 20162. Barbora Hálková 2 SPŠStav LI 9 11 – – – 20,20 20163. David Šnajdr 1 GMikul23PL 7 – 13 – – 19,43 19164. Tomáš Jurčo 1 G Prachati 19 – – – – 19,28 19165. Samuel Baran 2 GRaymanaPV – 19 – – – 18,59 19166. Nodari Gogatishvili 2 GZborovPH 5 14 – – – 18,50 89167. Radim Novotný 1 GMikul23PL 11 7 – – – 18,17 18

168.–169. Klára Cihlářová 2 G Klatovy – 18 – – – 17,77 18168.–169. Martin Strnad 2 G Dobříš – 18 – – – 17,77 18

170. Soňa Burešová 2 GHeyrovPH 17 – – – – 16,90 17171. David Vojáček 1 GSOŠ FrMýs 17 – – – – 16,83 17172. Magdaléna Bártová 1 GDašickáPA – 10 7 – – 16,79 17173. Jan Antonín Musil 0 PORG PH – 8 8 – – 16,68 17174. Kateřina Škorvánková 3 G Rokycany 4 8 4 – – 16,46 16175. David Žáček 3 GZborovPH 16 – – – – 16,36 16

176.–177. Jan Dopita 3 GBudějovPH 8 8 – – – 16,00 16176.–177. Leoš Smetana 3 G Jaroměř 8 8 – – – 16,00 16178.–181. Anna Jandová 1 G Leg PB – 16 – – – 15,89 16178.–181. Jaroslav Konečný 1 G Čelákov – 16 – – – 15,89 16178.–181. Jaroslav Kortus 1 GMikul23PL – 16 – – – 15,89 16178.–181. Radovan Picek 1 GBalbínaHK 16 – – – – 15,89 16

182. Marie Freibergová 3 G Děčín 11 4 – – – 15,62 16183.–184. Jan Lindauer 2 PČGKarVary 15 – – – – 15,05 15183.–184. Pavel Nedělník 2 GJarošeBO 15 – – – – 15,05 15

185. Jan Sedlák 4 GOPavla PH 15 – – – – 15,00 15186.–188. Vojtech Filipi 1 GDašickáPA – – 15 – 0 14,89 15186.–188. Michal Poft 1 G Teplice – 15 – – – 14,89 15186.–188. Martin Simet 1 GMikul23PL – – 15 – – 14,89 15

189. Marián Okál 2 SŠNvh 15 – – – – 14,87 41190. Anh Le Hoang 4 GJarošeBO 4 11 – – – 14,54 205191. Zuzana Tréglová 3 G Žatec – – 14 – – 14,20 14

192.–193. Veronika Blovská 2 GMikul23PL 14 – – – – 14,05 14192.–193. Petra Šnoblová 2 GJatečníÚL – 14 – – – 14,05 14

194. Henrieta Micheľová 4 GAlejKošic 14 – – – – 14,00 14195.–196. Zuzana Šraierová 1 GČeskoliPH – – 14 – – 13,81 14195.–196. Kamila Ženatá 1 GNeumannŽR – 14 – – – 13,81 14197.–198. Šimon Hutař 1 PORG PH 7 7 – – – 13,58 14197.–198. Kateřina Šauerová 1 GPSJazykHK 7 7 – – – 13,58 14

199. Dominik Krasula 3 G Krnov 14 – – – – 13,57 469200. Johana Dvořáková 1 G Trutnov – 14 – – – 13,51 14201. Jan Šuta 3 GJŠkodyPŘ 13 – – – – 13,47 13202. Jan Dittrich 4 GJarošeBO 3 5 5 – – 13,05 91203. Martin Hubata 0 GMikul23PL – 13 – – – 13,03 13204. Ákos Záhorský 2 G VJM Šahy 13 – – – – 13,00 13

205.–206. Duc Long Hoang 1 GMikul23PL – – 13 – – 12,64 13205.–206. Karel Müller 1 GMikul23PL – 13 – – – 12,64 13

207. Filip Pastierovič 3 G ĽŠtúraZV 7 1 3 – 1 12,59 13208. Matěj Žídek 4 G FrýdlNOs 12 – – – – 12,00 12209. Štefan Hollán 2 G Bytča 12 – – – – 11,54 63210. Oldřich Kos 3 GKepleraPH – 11 – – – 11,39 11

211.–218. Vítek Dragoun 1 GMikul23PL 11 – – – – 11,38 11211.–218. Adam Hlas 1 GMikul23PL 11 – – – – 11,38 11211.–218. Klára Holešovská 1 GJNerudyPH 11 – – – – 11,38 11211.–218. Dominika Jurčová 1 G Prachati 11 – – – – 11,38 11211.–218. Jakub Kropš 1 11 – – – – 11,38 11211.–218. Tomáš Pishovaký 1 RGZS Prost 11 – – – – 11,38 11

55

Page 56: Matematický korespondenční seminář Milýpříteli! · těžiště trojúhelníka KNP. Poměr, v kterém těžiště dělí těžnici, je přitom dobře znám – kýžený výsledek

Matematický korespondenční seminář 35. ročník (2015/2016), 2. komentáře

211.–218. Magdaléna Sejkorová 1 GUKlafárŽR – 11 – – – 11,38 11211.–218. Veronika Straková 1 G Benešov 11 – – – – 11,38 11

219. Samuel Karaba 2 SŠNvh 11 – – – – 11,31 95220. Kateřina Vaňková 4 GJarošeBO – 10 1 – 0 11,00 11221. Karolína Slaběňáková 0 ZŠ ValKlob – 10 – – – 10,11 10

222.–223. Přemysl Kopečný 4 ČRG ČesBuď 1 9 – – – 10,00 10222.–223. Hana Kučerová 1 GKřenováBO 10 – – – – 10,00 10224.–225. Šárka Altmanová 2 G Rokycany 9 – – – – 9,48 9224.–225. Aneta Němcová 2 GBoskovice – 9 0 – – 9,48 9

226. Jakub Novotný 3 GTNovákBO 9 – – – – 9,17 9227.–228. Michaela Brabcová 4 G Jírov ČB 9 – – – – 9,00 9227.–228. Adam Kutnar 4 NPorg 9 – – – – 9,00 9229.–232. Ha Mi Dao 1 GŠmejkalÚL 8 – – – – 8,48 8229.–232. Vadim Kablukov 1 GBNěmcovHK 8 – – – – 8,48 8229.–232. Veronika Řehulková 1 MendelG OP 8 – – – – 8,48 8229.–232. Karolína Tulingerová 1 AkademG PH – 8 – – – 8,48 8

233. Jana Řičicová 3 GCoubTábor 3 6 – – – 8,41 8234. Tomáš Flídr 0 GMasarykKM 8 – – – – 8,34 8235. Marie Kaiserová 2 CírkGPlzeň 8 – – – – 8,17 8

236.–237. Michael Borák 3 GKepleraPH – – 8 – – 8,00 8236.–237. Michaela Macáková 4 GOhradníPH – – 8 – – 8,00 8

238. Jakub Ševčík 4 GKukučPopr 5 3 – – – 7,77 89239. Lucie Hronová 3 GJarošeBO – – 7 – – 7,38 91

240.–241. Lukáš Belza 4 7 – – – – 7,00 7240.–241. Natálie Mikolajová 4 G HavlBrod – – 7 – – 7,00 7242.–245. Monika Berková 1 GKepleraPH – 7 – – – 6,79 7242.–245. Tereza Prokopová 1 GJHroncaBA 7 – – – – 6,79 7242.–245. Anna Šebestíková 1 GCON ČesBuď – – 7 – – 6,79 7242.–245. Samuel Štrunc 1 GMikul23PL – – 7 – – 6,79 7

246. Anežka Soukupová 2 SPŠchemBrno 7 – – – – 6,59 37247.–249. Agáta Brozová 4 6 – – – – 6,00 6247.–249. Haštal Hapka 4 GBudějovPH 6 – – – – 6,00 6247.–249. Lukáš Vik 4 6 – – – – 6,00 6

250. Tomáš Faikl 2 GDašickáPA 5 – – – – 5,27 5251. Matěj Šarboch 3 GKepleraPH 4 – – – – 4,23 4252. Eva Klimentová 3 GJarošeBO 4 – – – – 4,16 19253. Kateřina Jakubíková 4 G Vítkov 4 – – – – 4,00 4

254.–256. Zsófia Kálosi 4 GHSelyhoKM 3 – – – – 3,00 3254.–256. Michal Porubsky 4 GCyMeNitra 3 – – – – 3,00 3254.–256. Klára Stefanová 4 GBNěmcovHK – 3 – – – 3,00 3

257. Veronika Úlovcová 3 CírkGPlzeň 1 – – – – 1,47 1258.–259. Vojtěch Kantor 0 GNerudCheb – – 0 – – 0,00 0258.–259. Oliver Šmakal 0 BGJirsíkČB 0 – – – – 0,00 0

adresa: Korespondenční seminářKAM MFF UK

Malostranské náměstí 25

118 00 Praha 1

web: http://mks.mff.cuni.cz/e-mail: [email protected]